Section 4 Module 3 - Kiddie Tax, Charitable Contributions, AMT, Imputed Interest

¡Supera tus tareas y exámenes ahora con Quizwiz!

Which of the following may enable a limited partnership to provide specific tax advantages (subject to certain limits)? It allows for special allocations. It is not subject to the at-risk rules. Its distributions may trigger the alternative minimum tax. A)I only B)II and III C)I and III D)I and II

1 only The partnership business form allows for special allocations. Partners are still subject to at-risk rules and AMT. LO 7.2.2

John Prentice owns 100 shares of Quantum Inc., a publicly held corporation. He receives a stock dividend of 10 shares of Quantum Inc. when the fair market value (FMV) of the stock is $10 per share. John originally paid $20 per share for the 100 shares of stock. Approximately what is John's new basis in each share of stock after the stock dividend?

18 The effect of the stock dividend is to "dilute" the basis in the previously owned shares by establishing basis in the new shares. John now owns 110 shares with an aggregate basis of $2,000. This equals an approximate basis of $18 per share.

Nancy is a single taxpayer and 67 years old. She has the following itemized deductions: Home mortgage interest (first mortgage)$15,950State income taxes$3,120Property taxes$1,480Charitable contributions$2,000Gambling losses$1,500Unreimbursed employee business expenses$4,600Tax return preparation fee$400Medical expenses$18,980 Nancy's AGI for 2021 is $250,000. Included in the AGI is $500 of gambling winnings. What amount of Nancy's itemized deductions would be allowed for purposes of the AMT?

18680 Of the itemized deductions listed, only the qualifying home mortgage interest of $15,950, the charitable contributions of $2,000, and the gambling losses to the extent of winnings of $500, and the $230 of medical expenses are allowable for purposes of the AMT. The medical expenses are deductible to the extent they exceed 7.5% of AGI for both regular and AMT purposes.

Which of the following are adjustments to gross income (above-the-line deductions)? Medical expenses Capital losses Deductible IRA contributions

2 and 3

Which of the following forms of business may be classified as direct participation programs? General partnership Limited partnership S corporation Closely held C corporation A)III and IV B)IV only C)I, II, and III D)I and II

C) I, II, and III The tax advantages provided by direct participation programs are founded upon the principle that most types of business organizations function as tax conduits; therefore, a closely held C corporation cannot qualify as there is no flow through of gains and losses. LO

Which one of the following objectives of the federal taxation system would include keeping prices stable? A) Social objective B) Economic objective C) Writing tax code D) Revenue raising

Economic Objective The first, and perhaps most important, goal of the economic objective is price stability. As a social objective, JGTRRA and TCJA significantly increased the Section 179 expense limit and the amount of depreciation deductions (bonus depreciation) that may be claimed in the first year in an attempt to stimulate purchases of business assets. Revenue raising through corporate, individual, and payroll taxes is an important objective of the federal taxation system.

Jill purchased stock four years ago for $15,000. The stock had a fair market value (FMV) of $13,000 on the last day of the prior year. If Jill sells the stock for $16,000 six months later, what is her recognized gain or loss?

C) $1,000 long-term capital gain (LTCG) Because Jill sold the stock after four years, her holding period is long term and her gain is $1,000 ($16,000 − $15,000). LO 2.1.3

Which of the following taxpayers is subject to the passive activity loss rules? Individuals Estates Trusts A)I only B)I and II C)II and III D)I, II, and III

D) I, II, and III

Jasmine and Luke, a married couple, bought 100 shares of Mutual Fund B for $3,200 on March 2 of this year. On December 19 of this year, they sold the 100 shares of Mutual Fund B for $3,500. They used the proceeds to purchase a trailer for its FMV of $3,500 and immediately donated it to their church to use for special events. What are the tax consequences of this sale? A)There is no recognized gain on the sale because the proceeds were used for a donation. B)The couple has donated appreciated property. C)The couple has a short-term capital gain of $300 on the sale. D)The couple has a long-term capital gain.

The couple has a short-term capital gain of $300 on the sale The couple have a recognized short-term capital gain of $300. Separately, they have donated property with a value of $3,500 to the church

Which one of the following is accurate regarding the Lifetime Learning Credit?

The maximum credit of $2,000 is per tax return.

Which of the following best describes the marginal tax rate?

The rate which is paid on the last taxable dollar

Upon the disposition of a passive activity interest by gift, the suspended losses are A)added to the basis of the interest. B)deductible in full. C)deductible to the extent the losses exceed any increase in the fair market value of the activity. D)completely lost.

added to the basis of the interest. Upon the disposition of a passive activity by gift, the suspended losses are added to the basis of the activity.

The effective tax rate is calculated by dividing the calculated tax by

B) taxable income

In the current tax year, Fay has short-term capital loss carryovers of $5,000 and long-term capital loss carryovers of $40,000, both carried over from the previous year. Her net short-term gain for this year is $6,000, and her net long-term gain for this year is $5,000. How much of her gain for this year will be taxable? A) $5,000 B) $1,000 C) $6,000 D) $0

$0 Fay can apply her short-term capital loss carryover to all current short-term capital gains, which results in a net short-term capital gain for this year of $1,000 ($6,000 gain − $5,000 carryover). She is then left with a net long-term capital loss of $35,000 ($5,000 gain − $40,000 carryover). To calculate net capital gains for the year, aggregate the long-term and short-term gains or losses, which in this case equals $35,000 long-term loss − $1,000 short-term gain, or a $34,000 net capital loss. She has no net gain and, as such, pays no taxes on any of the capital transactions she made this year. LO 6.2.1

During the current tax year, Jim has $10,000 of passive income from a publicly traded limited partnership. He also has a nonpublicly traded limited partnership that will generate a $10,000 passive loss. How much of this passive loss, if any, is deductible by Jim during the current tax year? A)$0 B)$1,000 C)$10,000 D)$6,500

$0 Income from a publicly traded limited partnership may not be offset by any other passive losses. Remember that income and losses from nonpublicly traded partnerships may be used to offset each other. LO 7.2.2

Paula purchased an interest in a master limited partnership (MLP) that produced a loss of $7,000 this year. She also purchased a real estate limited partnership (RELP) that generated $10,000 of passive income this year. How much, if any, of the passive loss from the MLP could be used to offset Paula's income from the RELP in the current year? A)$3,000 B)$7,000 C)$0 D)$3,500

$0 The answer is $0. Paula's losses from the MLP cannot be used to offset income from the RELP. MLP losses may only be used to offset income from the same MLP. LO 7.2.2

Cara has a basis of $6,000 in a classic Mercedes that she purchased several years ago. This year, she sold the Mercedes to a business associate for $18,000. The buyer made the first of six annual installments of $3,000 this year. What is the amount of gain recognized in the current year? A)$3,000 B)$2,000 C)$12,000 D)$1,000

$2,000 The gross profit percentage is the profit on the sale ($12,000) divided by the contract, or sale, price of $18,000, or 66.67%. The gross profit percentage is multiplied by the payment received in the current year ($3,000) to give a gain recognized of $2,000. LO 6.2.4

Phillip's classic automobile was completely destroyed in an earthquake that was declared a federal disaster. Unfortunately, his insurance paid only $12,000, while the fair market value was $22,000. His basis in the automobile was $17,000. Phillip's AGI is $35,000. What is the amount, if any, of Phillip's deductible casualty loss? A)$1,500 B)$1,400 C)$6,000 D)$0

$1,400 The deduction is based on the lesser of basis or the decrease in FMV—reduced by the insurance, the $100 floor per occurrence, and 10% of AGI. Lesser of decrease in FMV ($22,000) or adjusted basis ($17,000) $17,000Less insurance coverage(12,000)$5,000Less $100 floor(100)$4,900Less 10% of AGI(3,500)Deductible loss on Schedule A$1,400 LO 6.2.3 PREV

Six years ago, Holly purchased an office building for her retail business at a cost of $200,000. She paid $5,000 in legal fees associated with the acquisition. She paid $10,000 to remodel the interior to make the building suit her purpose. She has paid $18,000 for property taxes and $20,000 for utilities during the five years. She has taken cost recovery deductions of $35,000. She also paid $40,000 interest on the mortgage note. What is Holly's current adjusted basis in the warehouse? A)$180,000 B)$220,000 C)$165,000 D)$170,000

$180,000 The $200,000 cost is increased by the capitalized costs: legal fees of $5,000 and improvements of $10,000. The cost recovery deductions reduce the adjusted basis to $180,000. The property taxes, utilities, and interest are currently deductible items and therefore do not affect basis.

Caleb earns a salary of $190,000. This year, he also received dividends and interest of $60,000. Caleb had previously invested $50,000 to purchase a 15% interest in a passive activity. Operations of the activity this year resulted in a loss of $400,000, of which Caleb's share is $60,000. How is Caleb's loss for the current year characterized for income tax purposes? A)$10,000 is suspended under the at-risk rules, and $50,000 is suspended under the passive activity loss rules. B)$60,000 is suspended under the passive activity loss rules. C)$60,000 is suspended under the at-risk rules. D)$50,000 is suspended under the at-risk rules, and $10,000 is suspended under the passive activity loss rules.

$10,000 is suspended under the at-risk rules, and $50,000 is suspended under the passive activity loss rules. Caleb invested $50,000 in the passive activity which becomes his at-risk amount. Because his share of the loss from the activity is $60,000, Caleb will be allowed to deduct only $50,000, which is his amount at risk. In addition, $10,000 of the loss ($60,000 total - $50,000 deductible under at-risk rules) has been suspended because of the at-risk rules and must be carried forward until Caleb either has $10,000 of income from the passive activity or invests $10,000 in the activity. Caleb has a $50,000 loss after applying the at-risk rules, but he is still not permitted a deduction for the loss because he has no passive income to offset the passive activity loss.

In 2021, Glen, a single taxpayer, has the following itemized deductions: Home mortgage interest (first mortgage)$7,950Home mortgage interest (home equity loan)$4,350State income taxes$4,120Property taxes$1,880Charitable contributions$2,600Gambling losses (to extent of winnings)$1,200 The home equity loan was incurred to purchase what Glen describes as a "midlife crisis" sports car. What amount of itemized deductions, if any, would be allowed for purposes of the AMT? A)$17,750 B)$7,950 C)$11,750 D)$16,100

$11,750 Of the itemized deductions listed, only the qualifying home mortgage interest of $7,950, the charitable contributions of $2,600, and the gambling losses to the extent of winnings of $1,200 are allowable for purposes of the alternative minimum tax. For both regular income tax and AMT purposes, the interest on the home equity loan is only deductible if used for acquisition or renovation of the principal residence and/or one other residence. The taxes are not deductible for AMT purposes.

Jack operates a cosmetic manufacturing business as an S corporation. During 2021, the business placed in service $33,000 of new property eligible for limited expensing under Section 179. If taxable income before cost recovery is $12,750, the maximum amount that the business can elect this year under Section 179 is A)$25,000. B)$12,750. C)$8,000. D)$33,000.

$12,750. The expensing election is limited to taxable income before the expensing election is calculated; therefore, $12,750 is the maximum Jack's business can elect under Section 179. The remaining amount of $12,250 is carried forward until the income of the business is sufficient to use up the remaining Section 179 expense election.

An office building with an adjusted tax basis of $120,000 was destroyed by fire on January 2 of last year. On January 15 of the current year, the insurance company paid the owner $200,000. The owner reinvested $190,000 in a new office building. What is the basis of the new building under Section 1033 (the involuntary conversion rules)? A)$120,000 B)$190,000 C)$110,000 D)$180,000

$120,000 Gain realized equals $80,000, and gain recognized equals $10,000 ($200,000 amount realized − $190,000 amount reinvested). This is a net postponed gain of $70,000 ($80,000 − $10,000). The basis of the new building equals $120,000 ($190,000 − $70,000).

In April of the current year, Mel sold his principal residence for a total price of $408,000. Mel sold the house due to a job transfer out of state. He received the house as a gift 18 months ago when its fair market value was $375,000, and the donor's basis was $50,000. He completed a room addition for $20,000. He paid realtor commissions of $23,000 on the sale. What amount, if any, must be recognized on the sale of Mel's residence? A)$315,000 B)$127,500 C)$0 D)$13,000

$127,500 Explanation Sale price$408,000Selling expenses($23,000)Total amount realized$385,000Less adjusted basis($70,000)Gain realized$315,000Gain recognized:Gain realized$315,000Less allowable exclusion($187,500)Gain recognized$127,500 The allowable exclusion is the full exclusion of $250,000 multiplied by 75%—18 months of use divided by the 24-month requirement. The basis is the $50,000, increased by the improvement of $20,000. The donor's basis is used as the carryover basis. The gain recognized is the amount of gain that is subject to tax. LO 6.2.5

Four years ago, David purchased a condominium to use as a rental property. The cost of the property was $165,000. He paid an attorney $1,800 to draft the legal documents required for the purchase of the property. He paid $6,500 to replace the wiring and patio before he could rent the property. Last year, he paid $2,200 to repair damage caused by the former tenants. He has claimed cost recovery deductions of $24,600. What is David's adjusted basis in the condo? A)$148,700 B)$150,900 C)$140,400 D)$142,200

$148,700 The cost of the condominium of $165,000 is increased by the capitalized costs. The capitalized costs would include $1,800 for the attorney's fees and $6,500 for improvements. Even if the $6,500 for wiring and patio replacement were considered repairs, they would be capitalized in this situation because they were incurred before placing the property in service (making it available to rent).This amount would be reduced by the cost recovery deductions of $24,600, to leave an adjusted basis of $148,700. The repairs of $2,200 represent a current deduction and therefore do not impact the basis of the property.

Jim's 2020 income tax return, which was for a full year, showed an AGI of $185,000 and a tax liability of $35,000. He estimates his 2021 tax to be $40,000 and his total wage withholding to be $10,000. What minimum amount, if any, of estimated tax payments must Jim pay (in equal quarterly installments) for 2021? A)$0 B)$26,000 C)$28,500 D)$36,000

$26,000 The safe harbor for avoiding the underpayment penalty is the lesser of 90% of the current year tax liability or 110% of the prior-year tax liability if the prior year's AGI is over $150,000. 90% of the current year tax liability is $36,000; 110% of the prior-year tax liability is $38,500. The smaller of these numbers, reduced by the $10,000 withholding, equals $26,000.

Your client is contemplating the sale of some of her holdings in her employer's stock. The stock was acquired in four separate purchases as follows: Purchase DateSharesCost Per ShareCostJune 1, 1996200$10$2,000June 1, 1998200$18$3,600June 1, 2000200$12$2,400June 1, 2006200$20$4,000Total800$12,000 What is the least amount of gain she would be required to report if she sold 500 shares for $12,500?

$3,700 To minimize the taxable gain, we would choose the shares with the highest cost basis. Thus we would sell the 200 shares that have a basis of $20 each, 200 of the $18 basis shares, and 100 of the $12 basis shares. This gives an aggregate basis of $8,800, resulting in a (long-term capital) gain of $3,700. The ability to use specific identification was not impacted by the Tax Cuts and Jobs Act (TCJA).

During the current tax year, Meghan expects to have the following items of income: Distributive share of general partnership self-employment income$25,000Distributive share of S corporation income$17,000Sole proprietorship net income$1,500 How much self-employment tax must Meghan pay for the 2021 tax year (round your answer to the nearest dollar)? A)$4,974 B)$3,744 C)$4,055 D)$200 PREV

$3,744 The partnership and sole proprietorship income are included. The S corp distribution is treated as a dividend not subject to SE tax. ($26,500 x .9235) x .153 = $3,744

Frank, a single taxpayer, owned a warehouse that he rented as commercial property. He acquired the property several years ago for $196,000. He used the straight-line method of cost recovery, which totaled $35,000. Frank sold the property in February of the current year for $230,000. Frank is single, and has taxable income (not including the real estate gain) of $475,000. What is the amount and nature of the gain on the sale? A) $34,000 Section 1231 gain; $35,000 ordinary income B) $7,000 ordinary income C) $69,000 ordinary income D) $35,000 unrecaptured Section 1250 gain; $34,000 long-term capital gain

$35,000 unrecaptured Section 1250 gain; $34,000 long-term capital gain The entire gain of $69,000 is treated as Section 1231 gain, because there is no excess depreciation on the use of the straight-line method. So, $35,000 of the gain is subject to a maximum rate of 25%, as unrecaptured Section 1250 income, and the remaining $34,000 of gain is subject to the maximum regular long-term rate of 20%. The 20% long-term capital gain rate applies, as his taxable income is over the $445,850 breakpoint for the 20% rate. Note that Section 1250 recapture (ordinary income treatment) applies only to excess depreciation—in other words, the excess of an accelerated method over what would have been deducted if straight-line had been used. All realty placed in service after 1986 is depreciated using straight-line, and there is NO recapture (ordinary income) where straight-line depreciation was used. LO 6.1.4

On April 1 of the current tax year, Susan sold her principal residence for a total price of $501,000: $301,000 was in cash, with the buyer assuming a $200,000 mortgage on the house. Susan purchased the house 15 years ago for $290,000, but she has an adjusted basis of $80,000. She has not made any improvements to the house. To assist in the sale of the residence, she incurred costs of $1,500 for repairs three weeks before the sale occurred. Realtor commissions of $31,000 resulted from the sale. On May 1 of the current tax year, Susan bought a new residence for $260,000. What amount, if any, must be realized on the sale of Susan Sharp's residence? A)$421,000 B)$180,000 C)$169,000 D)$390,000

$390,000 The answer is computed as follows: Gain realized: Amount realized:Sale price$501,000Selling expenses(31,000)Total amount realized$470,000Less adjusted basis(80,000)Gain realized$390,000 Note that the repairs do not impact the basis in the residence. Also, the mortgage assumption by the buyer is part of the sales price. LO 6.1.2

This year, Randy sold a classic painting to his neighbor, Norm, under the following terms: The price was $22,000, equal to the fair market value. Randy's basis in the painting was $5,000. Starting this year, Norm will pay in five annual installments of $4,000 plus accrued interest. Norm will make a $2,000 down payment. Ignoring interest income, what amount of gain will Randy recognize for the current year? A)$2,000 B)$4,500 C)$4,636 D)$3,000

$4,636 The profit on the sale of $17,000 divided by the $22,000 contract price equals a 77.27% gross profit percentage. This is multiplied by the $6,000 of payments received during the year to calculate the amount of gain recognized ($4,636): $17,000 profit$22,000 contract price=77.27%×$6,000 payments=$4,636 recognized$17,000 profit$22,000 contract price=77.27%×$6,000 payments=$4,636 recognized

Kris anticipates adjusted gross income of $100,000 during the current tax year. She is considering making a gift of appreciated real estate to the university she attended, Sinton Tech, a qualified charitable institution (a public charity). Kris's adjusted basis in this real estate is $40,000. The real estate has a current fair market value of $50,000. Kris has owned the real estate for six months. If Kris does gift the real estate to Sinton this year, what is the maximum allowable charitable deduction Kris can receive for the current tax year? A)$30,000 B)$50,000 C)$100,000 D)$40,000

$40,000 For a gift of ordinary income (short-term capital gain) property to a 50% organization, the deduction is based on the basis of the property with a 50% of adjusted gross income limitation. 50% of her AGI is $50,000. However her adjusted basis in the property is $40,000. Thus her deduction is limited to $40,000.

Joe has $60,000 in wages, $13,000 in deductions, and $5,000 in passive losses this year. The passive loss did not arise from rental property or real estate. What is Joe's taxable income this year? A)$60,000 B)$44,000 C)$47,000 D)$48,000 PREV

$47,000 Joe's taxable income is $47,000 ($60,000 − $13,000). Passive losses may only offset passive income so his $5,000 loss is not allowed, except for rental and real estate property. Rental and real estate passive losses are allowed up to $25,000 and are subject to the AGI phase out limitations.

Last year, Geoff invested $70,000 for a 25% interest in a partnership in which he was not a material participant. The partnership made a profit last year, of which $19,000 was allocated to Geoff. This year, the partnership suffered a loss and Geoff's share was $28,000. What is Geoff's capital at risk in the partnership at the end of the current year? A)$61,000 B)$48,000 C)$51,000 D)$38,000

$61,000 At the beginning of this year, Geoff had capital at risk of $89,000 ($70,000 initial investment + $19,000 allocated profit). At the end of the current year, Geoff's capital at risk is $61,000 ($89,000 beginning capital at risk - $28,000 allocated loss).

Dave owns equipment that has an adjusted basis of $10,000 and a fair market value of $75,000. Through an exchange, he acquires new equipment from Rachel that has a fair market value of $60,000 and an adjusted basis of $35,000. In the exchange, Dave receives $15,000 from Rachel. What is Dave's substitute basis in the acquired property? A)$15,000 B)$75,000 C)$10,000 D)$35,000

$75,000 Only realty is eligible for like-kind exchange tax treatment. Thus, Dave is deemed to have sold his equipment for $75,000 and purchased the new equipment for $75,000. LO 6.2.2

n 2021, Floyd, age 15, is a dependent on his parents' income tax return. When Floyd was born, his parents established an UGMA with corporate bonds and have contributed a little to it every year since. This year, the account generated $5,000 of interest income. There were no distributions from the account this year. Floyd's parents file jointly and have taxable income of $175,000 and are in the 24% MFJ tax bracket. What is Floyd's income tax liability for the current year? A) $1,029 B) $110 C) $782 D) $260

$782 Floyd's liability is $782. This is computed as follows: $5,000($1,100)limited standard deduction($1,100)taxed at child's rate of 10%$1,100×10%=$110.00$2,800taxed at parents' rate of 24%=$672.00$782.00

Sharon and Oliver South are a married couple filing jointly, with one dependent child. For the 2021 tax year, they have the following items relevant to their income tax situation: Wages$100,000Sole proprietorship net income$10,000Alimony paid to Oliver's former spouse$18,000Child support paid$12,000IRA contribution$6,000Self-employment tax liability$1,413Net capital loss$4,200Child tax credit$2,000Unreimbursed medical expenses$17,000 Oliver's divorce decree was finalized in 2015. Neither spouse is an active participant in a company-maintained retirement plan. What is the amount of the Souths' AGI?

$82,293 Explanation Wages$100,000Schedule C income10,000Net capital loss(3,000)Total income$107,000Alimony paid(18,000)IRA contribution($6,000)½ self-employment tax(707)AGI$82,293 Remember that net capital losses are only deductible up to $3,000 in a given year. Child support payments are neither deductible by the payor nor taxable to the recipient. The child tax credit is deducted after computing the income tax (a tax credit is a dollar-for-dollar reduction of the tax liability). Medical expenses are an itemized deduction, which are deducted after computing AGI (a below-the line deduction, or deduction from AGI)

Kurt and Allison Long are married and file a joint income tax return. Their adjusted gross income (AGI) is $180,000 per year. On last year's tax return, the Longs claimed a $1,200 credit for child care expenses. The Longs are in the 22% marginal income tax bracket. What amount of deductions for AGI would be required to equal the tax benefit of the $1,200 child care credit?

5,455 1,200 divided by the 225 marginal income tax bracket gives us 5455

Several years ago, Jack purchased a duplex to use as a rental property. The cost of the duplex was $79,500. He paid an attorney $800 to draft the legal documents related to the purchase. He paid $7,500 for improvements in the first year and this year paid $1,900 for various repairs to the property. He claimed a first-year cost recovery deduction of $3,000. What is Jack's adjusted basis in the duplex? A)$82,100 B)$87,800 C)$82,900 D)$84,800

$84,800 The cost of the duplex of $79,500 is increased by the capitalized costs (i.e., $800 for the attorney's fees and $7,500 for improvements). This amount would be reduced by the cost recovery deduction of $3,000, leaving an adjusted basis of $84,800. The repairs of $1,900 represent a current deduction and therefore do not impact the basis of the property.

Paul and Mary form an equal partnership to produce hammers for the military. They each have a 50% profit and loss sharing agreement. Paul contributes cash of $50,000 and property with a fair market value of $50,000 and an adjusted basis of $20,000. Mary contributes cash of $25,000 and property with a fair market value of $75,000 and an adjusted basis of $40,000. A bank gives the partnership a recourse loan of $30,000 and a nonrecourse loan of $25,000. What is the amount at-risk for Paul and Mary, respectively? A)$97,500 for Paul, $92,500 for Mary B)$70,000 for Paul, $65,000 for Mary C)$127,500 for Paul, $127,500 for Mary D)$85,000 for Paul, $80,000 for Mary PREV

$85,000 for Paul, $80,000 for Mary The amount at-risk is measured by the amount of cash contributed, increased by the adjusted basis of property contributed, and further increased by the partner's share of recourse financing only. Nonrecourse financing is included only if it is qualified nonrecourse financing in a real estate activity only.

Three years ago, Mort purchased equipment for use in his business at a cost of $26,000. He claimed a Section 179 deduction in the year of acquisition of $10,000, and has since claimed cost recovery deductions totaling $7,604. The equipment was sold for $18,000. What is the amount of cost recovery deductions that must be recaptured? A)$7,604 B)$0 C)$9,604 D)$8,396

$9,604 The cost basis of the property, $26,000, would be reduced by the Section 179 and cost recovery deductions taken, $17,604. This leaves an adjusted basis of $8,396. When the property is subsequently sold for $18,000, the difference between the sales price ($18,000) and the adjusted basis of $8,396 is the gain realized of $9,604. The recapture is the lesser of the gain realized of $9,604 or the cost recovery deductions taken of $17,604. LO 6.1.4

Philip, a professor, earned a salary of $140,000 from a university in the current year. He received $35,000 in dividends and interest during the year. In addition, he incurred a loss of $25,000 from an investment in a passive activity. His at-risk amount in the activity at the beginning of the current year was $15,000. What is Philip's adjusted gross income (AGI) for the current year? A)$160,000 B)$150,000 C)$115,000 D)$175,000 PREV

) $175,000 Philip's AGI, after considering the passive investment, is $175,000 ($140,000 active income + $35,000 portfolio income). He cannot offset the passive loss against active or portfolio income. The loss may be deducted only against passive income, which he does not have in the current year.

Which of the following rules regarding the sale of Section 1231 property is CORRECT? When Section 1231 property is sold for more than the purchase price, the gain is afforded capital gain treatment and taxed using capital gain tax rates. When Section 1231 property is sold at a loss, the loss is treated as a capital loss. A) Both I and II B) II only C) Neither I nor II D) I only

) I only Statement II is incorrect. When Section 1231 property is sold at a loss, the loss is treated as an ordinary loss, not a capital loss.

Which of the following applies to the at-risk rules, as related to passive loss restrictions for partners? It is the maximum deductible loss for an investment limited to the amount of risk that the taxpayer has at the end of the current year. Determining the amount at risk includes the adjusted basis of other property contributed to the partnership. The inclusion of nonrecourse financing is essentially the only difference between the basis in a partnership and the amount at risk. A) I only B) I and II C) I, II and III D) II only

) I, II and III In the Tax Code, the at-risk rules are defined as the maximum deductible loss for an investment limited to the amount that the taxpayer-investor has at risk at the end of the current year (i.e., the amount of potential economic loss). A partner may deduct losses only to the extent of the amount that they have "at risk." The amount at risk equals the sum of the following: The money invested (except to the extent the money invested was borrowed and was secured only by the investment) The adjusted basis of other property contributed to the partnership Amounts borrowed for use in the activity, but only to the extent that the partners are personally liable for repayment of the debt (recourse indebtedness) The partner's share of income, less the partner's share of losses or withdrawals from the partnership The proportionate share of qualified nonrecourse financing in a real estate activity ONLY Essentially, the only difference between the amount at risk and the basis in a partnership interest is the treatment of nonrecourse financing. LO 7.2.3

During the current tax year, Jamie sold several securities that resulted in the following types of gains and losses: a long-term capital loss of $6,700; a short-term capital loss of $7,000; a long-term capital gain of $1,900; and a short-term capital gain of $9,200. What is the net capital gain or loss on Jamie's security sales? A) Net short-term gain of $7,300; net long-term loss of $300 B) Net long-term loss of $2,600 C) Net long-term loss of $4,800; net short-term gain of $2,200 D) Net short term loss of $3,800

) Net long-term loss of $2,600 The long-term items are netted, leaving a long-term capital loss of $4,800. The short-term items are netted, leaving a short-term capital gain of $2,200. These are netted, leaving a net long-term capital loss of $2,600. LO 6.2.1

Jasper invested $20,000 for a 30% interest in a nonpublicly traded limited partnership. Jasper is not a material participant. The partnership has a loss this year and Jasper's share is $15,000. This is the only nonpublicly traded limited partnership Jasper owns an interest in, but Jasper did have portfolio income of $9,000 in the same year in addition to his salary of $150,000. How much of the loss can Jasper deduct this year? A)$9,000 B)$15,000 C)$6,000 D)$0

0 Because Jasper has no passive income to offset the passive loss, the loss is suspended under the passive activity loss rules.

Phillip's personal automobile was almost destroyed in an accident. The insurance company paid $6,000 on the claim. The auto's fair market value before the accident was $16,000, and the value after the accident was $1,000. His basis in the automobile was $12,000. Phillip's AGI is $42,500. What is the amount of Phillip's deductible casualty loss? A) $6,000 B) $1,750 C) $0 D) $1,650

0 Casualty losses are only deductible for damages sustained within a federally declared disaster area. Thus, there is no deduction for this loss. If the loss had been incurred in a federally declared disaster area (as a result of the disaster), the deductible casualty loss computation would begin with the lesser of the decrease in fair market value ($15,000 decrease in FMV) or the adjusted basis in the property. In this situation, the adjusted basis of $12,000 must be reduced by a $100 floor, the insurance of $6,000, and further reduced by 10% of the adjusted gross income. Thus, $12,000 reduced by $100, $6,000 insurance, and further reduced by $4,250, equals $1,650. LO 6.2.3

Ron, age 43, and Sandy, age 41, are married with two children: Michael, age 12, and Victoria, age 8, who has been blind since her birth. Ron is an architect and general partner with XYZ partnership. Sandy is self-employed as an attorney and works out of a home office. Her home office is exclusively and regularly used for business, and the home office is her principal place of business. Their information for the tax year 2021 is as follows: Adjusted gross income: $217,300 Itemized deductions (including qualified residential mortgage interest, taxes paid, and charitable contributions): $33,000 Early in the current year, Sandy's father died. Sandy is the sole beneficiary of her father's entire estate. The estate is presently in the probate process. Sandy's mother, Lisa, age 68, has moved in with them but provides her own support. She was married to Sandy's father when he died earlier this year. This is Ron's second marriage. He makes monthly support payments to his former spouse and his daughter. Because both Ron and Sandy are considered to be self-employed, they make quarterly estimated tax payments each year to cover both their income tax and self-employment tax obligations. Ron's divorce decree specifies that the payment made to his former spouse is $300 per week until his former spouse dies, at which point payments will continue to be made to her estate until her daughter is age 18. Based on the information provided in the case scenario for Ron and Sandy, what amount per week is considered to be alimony? A) $300 B) $0 C) $150 D) $50

0 If there is an obligation to continue to make payments after the death of the ex-spouse, the payments are not treated as alimony.

Lindsey is age 2 and her total income was $6,000 in qualified dividends in 2021. What is the tax on the dividends at Lindsey's rate? A) $30 B) $0 C) $143 D) $95

0 Lindsey is in the 10% marginal income tax bracket. She can use the long-term capital gains tax rate on qualified dividends received. At her income and filing status, that capital gain tax rate is 0%.

Lindsey is age 2 and she received $6,000 in municipal bond interest income and $900 in other interest income in 2021. What is the total federal income tax due on her income in 2021?

0 Lindsey owes no federal income taxes in 2021. Municipal bond interest income is not taxable. The $900 in other interest income is less than Lindsey's $1,100 standard deduction amount

Paula purchased an interest in a publicly traded partnership and has experienced a current loss of $7,000. If she purchased a nonpublicly traded partnership with $10,000 of passive income, how much of the passive loss may be used to offset Paula's income in the current year? A)$10,000 B)$3,000 C)$0 D)$7,000

0 The answer is zero. Losses from publicly traded partnerships cannot be offset against income from nonpublicly traded partnerships.

Nathan has a salary of $100,000, dividends of $4,000, and limited partnership income of $10,000. The limited partnership is publicly traded. During January of the current year, Nathan purchased an interest in a nonpublicly traded limited partnership that will generate a $12,000 passive loss during the current tax year. How much of this passive loss, if any, is deductible by Nathan during the current tax year? A) $4,000 B) $0 C) $12,000 D) $10,000

0 The general rule is that passive losses are deductible only against passive income. However, passive income from a publicly traded partnership cannot be offset by passive losses arising from any other source. Thus, the passive losses from the new partnership will not be deductible. LO 7.2.2

Which of the following statements regarding constructively received income is CORRECT? Income is constructively received in the taxable year in which there is no substantial limitation or restriction on a taxpayer's right to bring the funds under personal control. Constructive receipt occurs only when income is actually received during the taxable year. A)II only B)Neither I nor II C)I only D)Both I and II

1 only Statement II is incorrect; income may be constructively received in a taxable year even if it is not actually received. LO 8.2.3

Sandy operates a hairstyling sole proprietorship, and performs her business dealings from her home on a part time basis. She also has a full time job as an esthetician and is paid as a W-2 employee earning $46,000.. Her gross income for the current tax year is $71,000. The hairstyling income is $25,000. Due to supply and marketing costs, the business expenses not associated with the home office total $27,275. Expenses associated with the home office total $4,200. How much of the home office expense, if any, may Sandy deduct for the current year? A)$2,000 B)$0 C)$4,200 D)$275

0 The home office expense deduction is limited to the earned income from the business. In other words, the home office expense deduction can generally neither create nor add to a loss. In this situation, the $71,000 of gross income is reduced by the $27,275 of business expenses not associated with the home office. Sandy may claim a loss against ordinary income with her Schedule C. However, none of the home office expenses would be deductible in the current year. Note that the entire $4,200 of home office expenses would be subject to a carryforward.

Lisa and William are married taxpayers who file as married filing separately (MFS) for income tax purposes as it is advantageous for their tax positions; although the couple does live together. Lisa has rental property she inherited from her uncle that will generate a loss this year of $14,000. Lisa meets the active participation standard. Lisa's AGI is $65,000 and William's AGI is $80,000. How much is Lisa's allowed passive activity loss this year? A) $14,000 B) $5,000 C) $0 D) $12,500

0 The rental real estate loss allowance is not available to taxpayers who file as MFS and have lived together at any time during the tax year.

Angela, age 16, is claimed as a dependent on her parents' income tax return. During 2021, she earned $2,200 from a summer job. She also earned $2,600 in interest and dividends from investments that were given to her by her great-aunt five years ago. How much of Angela's unearned income, if any, will be taxed to her in 2021 using her great-aunt's marginal tax rate of 35%? A)$400 B)$0 C)$2,600 D)$2,200

0 When applying the kiddie tax, the parents' marginal tax rate is always used (regardless of the source of the property generating the unearned income). Therefore, none of the income is taxed to Angela using the great-aunt tax rate. The $400 of unearned income ($2,600 − $2,200) is taxed to Amy at her parents' marginal tax rate.

Paul, age 16, is listed as a dependent on his parents' income tax return. During 2021, he earned $2,600 from a summer job. He also earned $2,600 in interest and dividends from investments that were given to him by his uncle five years ago. How much of Paul's income, if any, will be taxed to him in 2021 using his uncle's marginal tax rate of 32%? A) $2,000 B) $400 C) $2,600 D) $0

0 When applying the kiddie tax, the parents' marginal tax rate is always used (regardless of the source of the property generating the unearned income). Therefore, none of the income is taxed to Paul using the uncle's tax rate. The $400 of income ($2,600 − $2,200) is taxed to Paul at his parents' marginal tax.

Alicia is age 16 and she received $6,000 in municipal bond interest income and $900 in other interest income in 2021. Her parents' marginal tax rate is 28%. What is the total federal income tax due on her income in 2021? A) $90 B) $735 C) $1,472 D) $0

0$ Alicia owes no federal income taxes in 2021. Municipal bond interest income is not taxable. The $900 in other interest income is less than Alicia's $1,100 standard deduction amount (for 2021

Which of the following are includible in an individual's gross income for income tax purposes? Gambling winnings Inheritances Interest collected by the taxpayer on federal obligations Scholarships and fellowships in degree programs

1 and 3 Gambling winnings and interest on federal obligations are includible in an individual's gross income for income tax purposes. The other items are not subject to income taxation

Ken is a successful attorney with a full-time practice from which he earns a salary of nearly $500,000 a year. Which of the following investments would be appropriate in reducing Ken's income tax liability? Investment in an oil and gas working interest A real estate limited partnership A rental real estate general partnership in which Ken will not participate Active participation rental real estate A)I only B)III only C)I, III, and IV D)II and III

1 only Only a direct interest in oil and gas (the working interest) would generate any tax savings. The other activities would be passive activities and would not meet any deduction exceptions. An investment in an oil and gas working interest is an exception to the passive activity rules. The real estate partnerships are both passive and could only be used if Ken had passive income against which he could offset the losses. The active participation rental real estate is also passive, and Ken does not meet the AGI requirements for the $25,000 exception because his AGI is well in excess of $150,000 (the upper limit of the active participation requirement). LO 7.1.2

Which of the following are qualified interest expense deductible in arriving at an individual's AGI? Student loan interest Mortgage interest on a loan acquired for a personal residence in 2021 for $600,000 Interest on home equity loan indebtedness to buy an automobile Investment interest expense A)II and III B)IV only C)I only D)I, II, III, and IV

1 only Only item I, student loan interest, is deductible in arriving at an individual's AGI. Statements II and IV are incorrect and are deductions from AGI. Statement III is not a deductible expense. Home equity loan interest is not deductible for AGI and is only deductible from AGI to the extent the proceeds are used for home acquisition or improvement and not personal expenses.

Which of the following statements regarding passive activity losses is CORRECT? When determining the amount of suspended loss that may be used against income, the at-risk rules are applied before the passive activity loss rules. If a loss is not allowed because of the at-risk limitations, the loss is a suspended loss eligible for deduction as a disposition of a passive activity. A) Both I and II B) II only C) I only D) Neither I nor II

1 only Statement I is correct. Statement II is incorrect. If a loss is not allowed because of the at-risk limitations, the loss is a suspended loss and is not eligible for deduction as a disposition of a passive activity.

Which of the following statements regarding the disposition of passive activities with suspended losses is NOT correct? After using losses to offset gain on the sale of the activity, any remaining losses are still passive losses. Suspended losses from the activity are first offset against any gain on the sale of the activity. A)I only B)II only C)Neither I nor II D)Both I and II

1 only Statement II is a correct statement. Statement I is incorrect. After using losses to offset gain on the sale of the activity, any remaining losses are classified as nonpassive losses. LO 7.1.1

Which of the following statements regarding charitable deductions by corporations is CORRECT? The corporate statutes of most states permit corporations to make charitable contributions, and the Tax Code permits a charitable deduction for contributions by a corporation. The charitable deduction is limited to a maximum of 20% of the corporation's adjusted taxable income. In the event the contribution is in excess of 20% of the corporation's adjusted taxable income, the balance can be carried forward for up to five years. A) Both I and II B) II only C) Neither I nor II D) I only

1 only Statement II is incorrect because the charitable deduction is limited to a maximum of 10% of the corporation's adjusted taxable income.

In February, Bryan purchased a new high-speed copier for use in his printing business. The cost of the copier was $8,250, sales taxes were $550, and installation charges totaled $1,200. Assume that Bryan opts out of the bonus depreciation provision. What is the first-year cost recovery deduction using the straight-line method? A) $880 B) $945 C) $2,000 D) $1,000

1,000 The installation charges of $1,200 and the sales taxes of $550 must be capitalized—that is, added to the cost of the copier to give a total basis of $10,000. A copier is five-year property. (Copiers, cars, computers, and computer peripherals are five-year properties; furniture and other equipment are seven-year properties.) The straight-line rate for five-year property is 20% (100% divided by five), but the half-year convention limits the deduction to half of a full year's depreciation in the year of acquisition. Thus, $10,000 times 10% equals $1,000. If Bryan had not opted out of bonus depreciation, the entire $10,000 would be depreciated in the first year.

John and Karen Postman will spend a total of $5,000 on day care for their two children (ages 9 and 10) in the current tax year. These expenses were incurred to allow both John and Karen to work outside the home. Their adjusted gross income is estimated at $138,000. What is the amount of child and dependent care credit, if any, to which they are entitled?

1,000 The maximum amount of qualifying expenditures on which the credit may be based is $3,000 per child, or $6,000 for two or more children. In this situation, they spent $5,000. This is multiplied by 20% for taxpayers with an AGI greater than $43,000. Thus, $5,000 × 20% = $1,000.

If an employer fails to withhold Social Security and federal income taxes from employee paychecks, what is the percentage penalty that is imposed by the IRS? A)50% B)75% C)100% D)25%

100% Employers are required to withhold amounts from an employee's paycheck for Social Security taxes and federal income taxes. If the employer fails to do so or fails to pay such amounts to the IRS, they or any other responsible person will be subject to the 100% penalty, which is simply having to pay 100% of the amount they should have collected, accounted for, and paid; they are not subject to any additional penalty.

A client sold an apartment building last year for $100,000, paying a sales commission of $5,000 plus $2,500 in closing costs. The building originally cost $80,000 20 years ago. Total straight-line depreciation of $40,000 had been taken. The building had a mortgage of $60,000 that was assumed by the buyer. The client is in the 24% marginal income tax bracket. What is the purchaser's cost basis? A)$70,000 B)$92,500 C)$100,000 D)$107,500

100,000 The purchaser's cost basis is simply what the purchaser paid for the property—$100,000.

Brent and Sheila are married and will file a joint return. They have provided you with the following information. Brent's salary$80,000Sheila's salary$65,350Alimony payments to Brent's ex-spouse$20,000Brent's child support paid$12,000Itemized deductions$13,000 Brent's divorce from his prior marriage was finalized in 2016. Based on the information given, what is their taxable income for the 2021 tax year? A) $111,250 B) $100,250 C) $112,050 D) $88,250

100250 The $145,350 in salaries is reduced by the alimony payment of $20,000 to give an AGI of $125,350. The AGI is reduced by the greater of the itemized deductions ($9,000) or the standard deduction ($25,100 in 2021) to equal $100,250. The child support paid is not a deductible item.

In the current year, William has passive losses of $21,000 from an equipment-leasing limited partnership purchased many years ago. He also has passive income of $11,000 from an older real estate limited partnership. Both limited partnerships are not publicly traded. What is the total amount of passive losses that may be used to offset active, passive, and portfolio income in the current year? A)$13,000 B)$21,000 C)$11,000 D)$12,000

11,000 The passive losses of $21,000 are deductible, but only up to the current year's income ($11,000).

In 2021, Jim had an AGI of $160,000. What percentage of prior-year income tax must be paid by Jim in 2021 to avoid the imposition of a penalty for underpayment of estimated tax? A) 80% B) 110% C) 90% D) 100%

110 The common exception for avoiding estimated tax penalties generally is 90% of the current year's tax or 100% of the prior year's tax. However, if the prior year's AGI exceeds $150,000, then the requirement is 90% of the current year tax or 110% of the prior year's tax liability. LO 8.2.3

John owns a classic automobile that had a cost basis of $32,000. John paid $38,000 to have the automobile fully restored. John sells the automobile through an installment sale for $100,000. John is to receive a $25,000 down payment in the current year, and $15,000 per year for five years, beginning this year. What amount of gain must John recognize during the current year? A) $12,800 B) $7,500 C) $4,500 D) $12,000

12,000 The profit on the sale was $30,000 divided by the $100,000 contract price, which equals a 30% gross profit percentage. This is multiplied by the $40,000 of payments received during the year to calculate the amount of gain recognized, $12,000. The $38,000 of restoration costs are capitalized, added to basis, to give us the $70,000 basis.

During 2021, your client, Bob, purchased several items of equipment with a total cost of $265,000 for use in his sole proprietorship. Bob has taxable (earned) income from his Schedule C business of $112,000 (without regard to the Section 179 expense). He also has wages from a part-time job of $10,000. What is the maximum amount of Section 179 expense that Bob may deduct in the current year? A) $122,000 B) $1,000,000 C) $265,000 D) $112,000

122000 The Section 179 expense election is limited to the taxable (earned) income of the taxpayer. For purposes of Section 179, salary or wages received as an employee, even from a completely unrelated source, are also considered to be from the active conduct of the trade or business. Thus, the total taxable (earned) income in this situation is $122,000. The maximum Section 179 expense election is $1.05 million (for 2021), but for Bob, it is limited to his earned or taxable income of $122,000.

Caroline, age 16, has earned income of $18,605 and interest income of $750 in 2021. She is listed as a dependent on her parents' income tax return. What is Caroline's standard deduction for earned income in 2021? A)$1,100 B)$18,000 C)$0 D)$12,550

12550 Caroline's standard deduction for earned income is $12,550 (for 2021), whereas her standard deduction for unearned income is only a maximum of $1,100 (limited to the $750 of actual unearned income in this instance). Caroline would, therefore, elect to offset her total income of $19,355 by the greater of the standard deductions, which is $12,550 in 2021.

Philip's grandmother gave securities to him when he was born nine years ago. In 2021, he has dividends of $15,000 from the securities. What is his net unearned income taxed at his parents' marginal tax rate? A)$14,100 B)$12,550 C)$9,300 D)$12,800 PREV

12800 Some of Philip's unearned income is taxed at his parents' marginal tax rate and is calculated as follows: $15,000 UI - $1,100 (standard deduction) - $1,100 (greater of $1,100 for 2021 or amount of allowable itemized deductions directly connected with the production of the unearned income) = $12,800

Jeff Munroe has an annual salary of $140,000 and is not an active participant in a company-maintained retirement plan. He had the following financial transactions during the current tax year: Received a $100,000 cash inheritance due to the death of his brother Received unemployment compensation of $2,000 Had a Schedule C loss of $10,000 (assume material participation) Made an IRA contribution of $6,000 Paid qualified student loan interest of $2,000 What is Jeff's total income for the current tax year?

132000 The $140,000 salary is reduced by the $10,000 self-employment loss and increased by the unemployment compensation of $2,000. The inheritance is excluded. The IRA contribution is a potential adjustment to income, as is the student loan interest. Thus, those items do not affect the total income. Remember that total income is the figure approximately two-thirds of the way down the front of the 1040. It is the figure from which allowable adjustments to income are subtracted.

Several years ago, Allison Colbert purchased a deferred fixed annuity. The cost of the annuity was a single payment of $40,000. The annuity will provide monthly payments of $275. At the time the annuitized distributions are to begin, Allison's life expectancy will be 25 years. How much of each payment will be excluded from taxation? A) $133 B) $142 C) $57 D) $206

133 Allison is expected to receive $82,500 ($275 × 12 × 25). Her investment in the contract ($40,000) is then divided by the total expected return ($82,500) to determine the excludable portion of each payment. The exclusion ratio is the $40,000 divided by $82,500, which equals 48.48%. 48.48% of $275 = $133 excludable from each payment.

Clare is a single taxpayer. In 2021, her AGI is $235,000, including a net long-term capital gain of $50,000. What is the amount, if any, of Medicare contribution tax that she must pay? A) $570 B) $1,330 C) $0 D) $1,900

1330 She will pay the 3.8% Medicare contribution tax on $35,000. This is the lesser of the net investment income ($50,000) or the AGI in excess of the threshold amount ($235,000 - $200,000, or $35,000). In this situation, only $35,000 of the net investment income is subject to the Medicare contribution tax. Clare will pay a $1,330 Medicare contribution tax (3.8% on $35,000).

Larry and Paula are a married couple who file their federal income tax returns separately. They are both over 65 and still provide full support for a son who has been blind since birth. They live together and do not itemize. They alternate listing their son as a dependent, and it is Paula's turn this year. Paula will be required to file a federal income tax return if her gross income is at least which of the following amounts in 2021?

13900 The normal filing threshold for the MFS filing status is $12,550 in 2021. For married taxpayers over age 65, the threshold is raised by $1,350 per spouse. The additional blind deduction applies only to the taxpayers themselves, not their dependents. Tangentially, if the other MFS spouse itemizes, the filing threshold is reduced to $5. (IRS pub 501, 2021) Because Larry and Paula still live together, neither can file as head-of-household with a dependent

James and Julie are a married couple filing jointly. For the 2021 tax year, they have a taxable income of $400,000. Included in the taxable income is $35,000 of net long-term capital gains from the sale of securities and $15,000 of qualified dividends. At what tax rate will their net capital gains and qualified dividends be taxed? A)25% B)20% C)15% D)0%

15% The long-term capital gains and qualified dividends are taxed at 15%. The long-term capital gains and qualified dividends fall into the taxable income range of $80,800 to $501,600 (2021) for a married couple filing jointly.

James has suspended losses from a nonpublicly traded partnership of $17,000. In 2021, he has income from a nonpublicly traded partnership of $15,000. What amount of suspended losses, if any, may James deduct in 2021? A)$17,000 B)$0 C)$2,000 D)$15,000

15,000 He may deduct (under the at-risk limitations) his share of the partnership loss to the extent of his investment in the partnership. With a $17,000 loss and only $15,000 in income, James can only deduct $15,000 of the loss.

If Jason files single with gross income of $110,000 and taxable income of $91,000, what is his effective tax rate based on the tax rate schedule provided in your course references?

17.43% 24%($91,000 ‒ $86,375) + $14,751 = $1,110 + $14,751 = $15,861 ÷ $91,000 = 17.43%.

Which of the following statements describes the constructive receipt doctrine? Darrell was issued notification in December of this year that his bonus for the current year would be $10,000 and the check would be issued in January next year. Martha was notified that a check would be issued to her on December 31 for royalties on her song used in a local commercial; the check was available for pick up that day. A) II only B) Both I and II C) Neither I nor II D) I only

2 only Darrell was only notified in December of the amount of the bonus that would be paid. No check was issued and the funds were not available until January of the following year. The check to Martha constitutes constructive receipt, whether or not she picks up the check in December.

Your client Sally, age 30, is designing an educational investment program for her 8-year-old son. She expects to need the funds in about 10 years when her AGI will be approximately $70,000. She wants to invest at least part of the funds in tax-exempt securities. Which of the following investment(s) may yield tax-exempt interest on her federal return if the proceeds were used to finance her son's education? Treasury bills EE bonds GNMA funds Zero coupon Treasury bonds A) III and IV B) II only C) I, III, and IV D) II and III

2 only Proceeds from EE savings bonds may be exempt if the proceeds are used for qualified higher-education expenses of the taxpayer, spouse, or dependent. There is an AGI phaseout, which is $83,200‒$98,200 (2021) for a single taxpayer. (The actual phaseouts are provided on the exam.) All the other options generate currently taxable income. The Treasury bills and GNMA funds both produce taxable income on the federal return (Treasury bill interest would typically be tax exempt on her state return). The zero Treasury also produces taxable income each year as the amortized discount is added to taxable income, even though no cash income is received.

Which one of the following is an application of the administrative powers of the Internal Revenue Service and not of the powers of Congress? Regulations Revenue Rulings A) II only B) Neither I nor II C) I only D) Both I and II

2 only Regulations are a direct extension of the lawmaking powers of Congress, whereas revenue rulings are an application of the administrative powers of the Internal Revenue Service. LO 8.1.1

Steve's tax return for a prior tax year has been audited, and the IRS has assessed a deficiency of $10,000 against him. In addition, he owes interest of $3,000 on the deficiency. The deficiency was due to negligence on Steve's part. What is the tax penalty that may be imposed on Steve? A)$7,500 B)$2,000 C)$5,000 D)$700

2,000 The negligence penalty is imposed if any part of the underpayment of tax is due to taxpayer neglect or a disregard for the tax rules and regulations, without the intent to defraud. The penalty is 20% of the portion of the underpayment attributable to negligence, which is $2,000 in this case. LO 8.2.2

Which of the following rates apply to qualified dividends that fall above the $501,600 (2021) taxable income breakpoint for a married couple filing jointly (MFJ)? A)25% B)28% C)20% D)15%

20% Qualified dividends (and net long-term capital gains) that fall into the taxable income above $-501,600 (for MFJ) are taxed at a 20% rate

Frank, a taxi driver, inadvertently fails to report approximately $1,200 of his tips received during the tax year. If the IRS imposes a penalty due to the underreported income, it would most likely impose a penalty equal to A) 20% of the deficiency plus 50% of the interest. B) 50% of the deficiency. C) 20% of the deficiency. D) 75% of the deficiency.

20% of the deficiency The negligence penalty is likely to be imposed, due to the failure to make a reasonable effort to comply with the requirements of the Code. This carries a penalty of 20% of the deficiency.

For the current tax year, Bob Phillips, an individual taxpayer filing a joint return, has $50,000 of investment interest expense and $20,000 of net investment income (interest and dividends). Bob's AGI is $200,000. How much investment interest expense, if any, may Bob deduct in the current tax year? A) $0 B) $50,000 C) $20,000 D) $21,000

20000 Investment interest expense is deductible up to the amount of net investment income. The problem tells us that the net investment income is $20,000; thus that is the maximum deduction. The fact that the dividends are included in the net investment income indicates that the taxpayer elected to include them in investment income and is forgoing the preferential rates associated with qualified dividends. The AGI has no bearing on the answer.

If Leslie and Armando file as married filing jointly, with gross income of $176,000 and taxable income of $158,000, what is their marginal tax rate based on the following tax information? Refer to the 2021 tax table provided in your course references.

22% His marginal tax bracket is 22%.

If Phoebe files single with gross income of $86,000 and taxable income of $75,000, what is her marginal tax rate? Refer to the 2021 tax table provided in your course references.

22% His marginal tax bracket is 22%.

Terry and Jan are married taxpayers filing a joint tax return. In 2021, their AGI is $310,000, and their net investment income (included in the AGI) is $90,000. What is the amount of their Medicare contribution tax for 2021? A) $2,280 B) $0 C) $3,420 D) $4,180

2280 Terry and Jan will pay the 3.8% Medicare contribution tax on $60,000. This is the lesser of the net investment income ($90,000) or the AGI in excess of the threshold amount ($310,000 - $250,000, or $60,000). In this situation, only $60,000 of the net investment income is subject to the Medicare contribution tax and calculates to $2,280 ($60,000 × 0.038).

Michelle Will has interest income of $23,000 in the current tax year. She paid brokers' commissions of $2,000 on stock purchases and had $40,000 of investment interest expense. What amount, if any, of investment interest expense may be deducted as an itemized deduction? A) $21,000 B) $33,000 C) $0 D) $23,000

23000

Jack and Melissa, who are both age 49, are married and file jointly for federal income tax purposes. What amount must their total gross income meet or exceed to require the couple to file a federal income tax return in 2021? A)$18,800 B)$12,550 C)$25,100 D)$26,800

25100 Married individuals who file jointly are not required to file unless their combined gross income equals or exceeds the basic standard deduction amounts, $25,100 in 2021.

Carla anticipates adjusted gross income of $90,000 during the current tax year. She is considering making a gift of real estate to the United Way. Carla's adjusted basis in this real estate is $20,000. The real estate has a current fair market value of $55,000. Carla has owned the real estate for three years. If Carla gifts the real estate to the United Way this year, what is the maximum allowable charitable deduction she can receive for the current tax year? A)$45,000 B)$20,000 C)$55,000 D)$27,000

27000 The gift of long-term capital gain property to a 50% organization is limited to 30% of AGI, which is $27,000. The deduction is based on the FMV of the property contributed. There would be a five-year carryforward of $28,000. The use-related or use-unrelated provisions do not apply here, as the distinction is relevant only with respect to tangible personalty, not realty. LO 3.2.2

Samantha received the following dividends in 2021 from her portfolio: Ordinary dividends from HOT stock, a publicly traded company Dividends from Sky High Realty and Trust, a publicly traded REIT Life insurance dividends from her whole life policy Qualified dividends from BET stock, a publicly traded company Which of the above is NOT considered taxable? A) IV only B) II and IV C) III only D) I and II

3 only Life insurance dividends are considered a return of premium paid (provided the cumulative dividends received over the life of the policy do not exceed the basis in the policy) and thus are not taxable. The other choices listed are taxable. Qualified dividends are eligible for long term capital gains rates. REIT dividends may qualify for a QBI deduction but nonetheless will still be taxable

Assume that married taxpayers filing jointly have a taxable income of $175,000. Using the tax rate schedule provided in your course references, what is the amount of federal income tax? Round your answer to the nearest dollar.

30,042 Taxable income$175,000 Less (from tax rate schedule)(172,750) Amount over $172,750 $2,250 Times (marginal rate, from tax rate schedule)24% Tax on amount over $172,750 $540 Plus (from tax rate schedule)$29,502 Total Tax$30,042 If you arrived at another answer, you likely did the calculation correctly but may have used the single filing status rather than the married filing joint return status or used a flat rate of 24%. While 24% is the marginal rate (the rate paid on the last dollar of income), there are dollars that are taxed at the 10%, 12%, and 22% rates as well.

Alicia is age 16 and her total income was $3,000 in bank interest in 2021. Her parents' marginal tax rate is 24%. What is her total tax on the interest? (Round the answer to the nearest dollar.)

302 Alicia is in the 10% marginal tax bracket. $1,100 is eliminated by her standard deduction for unearned income. The next $1,100 is taxed 10%, generating tax of $110. The remaining $800 is taxed at her parent's marginal rate of 24%, or $192. Therefore Alicia's total tax due is $302.

This year, Harold sold a rare painting to his neighbor, Ken, on the following terms: The price was $6,000, equal to the fair market value. Harold's basis in the painting was $4,000. Ken will pay in six annual installments of $1,000 plus accrued interest. Ken will make the first installment payment this year. Ignoring interest income, what amount of gain will Harold recognize for the current year? A)$6,000 B)$333 C)$1,000 D)$667

333 The profit on the sale of $2,000 divided by the $6,000 contract price equals a 33.33% gross profit percentage. This is multiplied by the $1,000 of payments received during the year to calculate the amount of gain recognized ($333): $2,000 profit/$6,000 contract price=33.33%×$1,000 payments=$333 recognized

Janet and Bruce Robinson, both age 43, are married taxpayers filing jointly. They have itemized deductions consisting of the following: Home mortgage interest$19,500State income taxes$8,700Property taxes$5,200Charitable contributions$6,200Tax return preparation fee$895Unreimbursed employee business expenses$2,100Unreimbursed medical expenses$18,460 Their AGI for 2021 is $466,000. What is the amount of their allowable itemized deductions?

35,700 The total itemized deduction amount is $35,700. Note that the tax preparation fee and the unreimbursed employee business expenses are not deductible. The medical expenses are deductible only to the extent that they exceed 7.5% of AGI for 2021, which they do not. The deduction for the state income taxes and the property taxes is capped at $10,000. Taxes of $10,000, mortgage interest of $19,500, and charitable contributions of $6,200 total $35,700.

Bruce and Melissa Parish, married taxpayers filing jointly, have the following items related to their investments during the current tax year: Investment interest expense $5,000 Interest income $2,500 Short-term capital gains $1,000 Investment adviser's fees $1,250 Commissions paid on stock purchase $200 Adjusted gross income $60,000 What is the Parishes' allowable investment interest expense deduction for the current year? A) $3,450 B) $3,500 C) $5,000 D) $3,250

3500

Kris Swenson anticipates adjusted gross income of $100,000 for the current tax year. She is considering making a gift of a painting to the American Red Cross in the current tax year. Kris's basis in the painting is $35,000. The painting has a current fair market value of $50,000. Kris has owned the painting for 15 years. If Kris does gift the painting to the American Red Cross this year, what is the maximum allowable charitable deduction she can receive in the current tax year? A) $20,000 B) $30,000 C) $50,000 D) $35,000

35000 The painting would be considered use-unrelated tangible personalty. The deduction for use-unrelated tangible personalty is limited to basis, with a 50% of AGI limitation. Thus, the current-year deduction is $35,000. If the painting had been donated to an art museum, for example, the contribution would be of use-related tangible personalty. Since the painting had been held for the long-term holding period, the deduction would have been $30,000 (long-term capital gain property to a 50% organization uses FMV with a 30% of AGI limitation) with a $20,000 carryforward.

Claudia makes $5,000 a month, and has a disability policy that pays 60% of her salary. Her employer pays 60% of the premium and she pays the remaining 40%. She needed surgery last year and received disability benefits for 60 days. What amount of taxable disability benefits did she receive

3600 Claudia received 60 days, or two months, of disability payments in the amount of 60% of her $5,000 monthly salary, or a total of $6,000. Claudia pays 40% of the premium and her employer pays 60%. The portion of the benefits received that was paid by Claudia's employer was 60% of $6,000, or $3,600, and is taxable. The remaining $2,400 is not taxable to Claudia

Jim is planning to make a charitable contribution to a local university, a qualifying charitable organization. He is going to contribute a piece of real estate that he has owned for six years. The fair market value of the property is $80,000, and his basis in it is $55,000. He has an AGI of $120,000. Jim wants to maximize the amount of charitable contribution deductions from the donation of the real estate. What is the amount of charitable contribution deduction that Jim may claim in the current year? A) $40,000 B) $55,000 C) $36,000 D) $60,000

36000 The gift of long-term capital gain (LTCG) property is generally based on the fair market value of the property. The university is a 50% organization, a public charity. LTCG property contributed to a 50% organization involves a 30% of AGI limitation, and 30% of $120,000 is $36,000. There is also a $44,000 carryforward for up to five years. Jim could have made a 50% election to maximize the current-year deduction, but that would have reduced his overall deductions. If Jim had made a 50% election, he could have deducted $55,000 in the current year. By forgoing the 50% election, he is allowed to deduct the full $80,000 fair market value—$36,000 this year and $44,000 over the next several years.

Kevin Riley anticipates adjusted gross income of $120,000 for the current tax year. He has made no charitable gifts during the year, but now he wants to give his church a stamp collection with a fair market value of $70,000. Kevin paid $38,000 for the collection five years ago. The collection is appreciated tangible personal property that is unrelated to the church's exempt function. What is the maximum allowable charitable deduction Kevin can receive during the current year if he makes an immediate gift of the stamp collection? A) $24,000 B) $60,000 C) $38,000 D) $36,000

38000

Neil McElroy is an engineer for Causley Computer Inc. In addition, Neil operates a janitorial service that cleans several local office buildings. Neil was divorced in 2019, and his wife received custody of their two children. He has assembled the following information for preparation of his tax return for the current tax year. Neil's salary$71,500Interest income$9,500Monthly alimony paid to ex-spouse$1,500Monthly child support$500Purchase of equipment for use in janitorial service$10,000IRA contribution$6,000 Based on the information given, which of the following are fundamental methods of managing Neil's tax liability? Tax credit: Neil could take an investment tax credit for purchases of qualifying business equipment. Deductions for AGI: Neil may deduct alimony payments of $18,000 made to ex-spouse. Deductions for AGI: Neil may deduct child support payments of $6,000. Exclusions: Neil could have invested in municipal bonds to receive tax-free income.

4 only Neil may not deduct alimony paid to his former spouse because the deduction is disallowed for alimony under divorce decrees in 2019 and thereafter. He may invest in municipal bonds to receive tax-free income. There is no investment tax credit for equipment purposes. Some students confuse this with the Section 179 expense election, but that provision provides a deduction, not a credit. Child support payments are specifically nondeductible.

A client sold an apartment building last year for $100,000, paying a sales commission of $5,000 plus $2,500 in closing costs. The building originally cost $80,000 20 years ago. Total straight-line depreciation of $40,000 had been taken. The building had a mortgage of $60,000 that was assumed by the buyer. The client is in the 24% marginal income tax bracket. What is the seller's adjusted cost basis? A) $37,500 B) $32,500 C) $52,500 D) $40,000

40,000 The seller's adjusted basis is the $80,000 purchase price, decreased by the $40,000 of straight-line depreciation. The mortgage has no bearing on the basis of the property.

Mary filed her 2020 income tax return on April 3, 2021. The IRS has recently determined that she worked a part-time job, but the employer failed to provide Mary with a W-2. Mary honestly did not realize that she was required to report the income because she did not receive a W-2. The IRS has determined that her negligent failure to report the income resulted in an additional income tax liability of $2,000. What is Mary's tax penalty? A)$200 B)$1,000 C)$400 D)$100

400 The negligence penalty is 20% of the deficiency due to the taxpayer's negligence. For the $2,000 tax deficiency, 20% results in a negligence penalty of $400. It may be argued that the failure to report the income was fraud, but the fact pattern states that the act was merely negligent. LO 8.2.2

Frank Swanson anticipates adjusted gross income of $80,000 during the current tax year. He is considering making a gift of real estate to the public university he attended. Frank's adjusted basis in this real estate is $50,000. The real estate has a current fair market value of $70,000. Frank has owned the real estate for 19 months. If Frank donates the real estate, what is the maximum allowable charitable deduction Frank can receive for the current tax year? A) $70,000 B) $50,000 C) $40,000 D) $24,000

40000 If Frank makes a 50% election, he must utilize the basis of the property but may deduct up to 50% of AGI. This yields a $40,000 current-year deduction with a $10,000 carryforward. If no 50% election were made, the deduction would be based on the fair market value of the property but would be limited to 30% of AGI, which is $24,000, with a $46,000 carryforward.

Helen purchased an antique cabinet as an investment for $30,000 a few years ago. On January 15 of this year, she sold the cabinet to an art museum for $120,000 in an installment sale. She received a down payment of $12,000 and a note requiring monthly principal payments (to begin in March of this year) of $5,000. What amount of gain must Helen recognize for the current year? A) $50,000 B) $46,500 C) $42,500 D) $62,000

46500 Step 1: Calculate gross profit percentage: profit divided by sale price. $90,000$120,000=75%$90,000$120,000=75% Step 2: Calculate payments received in current year. $12,000 down payment + (10 × $5,000) = $12,000 + $50,000 = $62,000 (payments received) Step 3: Calculate gain recognized for current year. gross profit percentage×payments received=gain recognized75%×$62,000=$46,500 LO 6.2.4

In the current year, Jeff makes the following charitable donations: BasisFMVInventory used in Jeff's business (sole proprietor):$8,000$6,000Stock in ABC Co. (acquired 2 years ago):$10,000$40,000Personal coin collection (acquired 10 years ago):$1,000$7,000 The ABC stock was given to Jeff's church, and the coin collection was given to the Boy Scouts of America. Both donees promptly sold the property for the stated FMV. Ignoring percentage limitations on AGI, Jeff's maximum charitable contribution valuation for deduction purposes available for the current year is A) $47,000. B) $53,000. C) $55,000. D) $19,000.

47000 Jeff's maximum valuation for deduction purposes available for the current year is $47,000 as follows: Inventory: $6,000 (as ordinary income property, limited to lesser of basis or FMV) Stock in ABC Co.: $40,000 (the maximum possible deduction is FMV for this long-term capital gain property) Personal coin collection: $1,000 (tangible personal property depends on dedicated use of property from standpoint of the charity; this is use-unrelated property because it was given to the Boy Scouts also limited to the lesser of basis or FMV)

Your client, Hal Meyer, will receive a deductible loss of $15,100 from a working oil and gas interest. Hal is in a 35% marginal income tax bracket and has asked you the approximate amount of tax savings that this will generate. What is the approximate amount, if any, of tax savings generated by this loss?

5200 In a 35% tax bracket, a $15,100 loss deduction will save $5,285. Thus, $5,200 is the closest answer.

Jane, age 35, whose filing status is single, earned a salary of $55,000 in 2021. She also made a $2,000 contribution to her Roth IRA for 2021. Jane had a capital loss of $3,000 during the year. Her uncle, Charles, gave her $100,000 in municipal bonds for which she earned interest of $3,500. In her employment as a sales representative for her company, Jane incurred $650 of unreimbursed business expenses. What is Jane's adjusted gross income (AGI)?

52000 Jane's AGI is $52,000 ($55,000 ‒ $3,000). Jane's $3,000 capital loss is a deduction for calculating AGI. Roth IRA contributions are never deductible from gross income. Municipal bond interest is not included in income. The unreimbursed business expenses are not deductible.

Pierre, a U.S. citizen, is meeting with his financial advisor regarding some transactions he made during this tax year. He has ample income and has made several charitable donations. They were as follows: $30,000 to his church for a building addition fund $100,000 to an orphanage in France in the city his parents lived as children $10,000 to the United Way $15,000 to a nonprofit organization in New Orleans that teaches French to children $50,000 to a gubernatorial campaign in his state Without regard to AGI limitations, how much of his donations may be deductible on his income tax return? A)$55,000 B)$40,000 C)$205,000 D)$105,000

55000 Of the listed donations, the $100,000 to the orphanage is nondeductible because it is a charity located in France. The $50,000 given to the gubernatorial campaign is a nondeductible political donation. The $30,000 given to the church, the $10,000 given to the United Way, and the $15,000 given to the nonprofit organization are deductible donations.

Terry and Jan are married taxpayers filing a joint tax return. Their AGI is $265,000, and their net investment income (included in the AGI) is $70,000. What is the amount, if any, of the Medicare contribution tax? A)$0 B)$15,000 C)$570 D)$2,660

570 The answer is $570. ($265,000 - $250,000 = $15,000. $15,000 × 3.8% = $570.)

Victoria Glass has $6,500 of investment interest expense and net investment income of $6,000 in the current tax year. She paid broker's commissions of $1,000 during the tax year. How much investment interest expense, if any, may Victoria deduct in the current tax year?

6000 Investment interest expense is deductible only to the extent of net investment income.

Chris Burdick anticipates adjusted gross income of $200,000 for the current tax year. He contributed appreciated stock to the United Way. Chris's adjusted basis in this stock is $50,000. The stock has a current fair market value of $140,000. Chris has owned the stock for 12 years. If Chris does gift the stock to the United Way, what is the maximum allowable charitable deduction he can receive in the current tax year? A) $140,000 B) $50,000 C) $60,000 D) $100,000

60000 A gift of long-term capital gain property to a 50% organization is based on the FMV of the property, with the deduction for the current year limited to 30% of AGI.

Walter anticipates his adjusted gross income to be $200,000 for the current tax year. He has not made any charitable contributions during the year, but now wants to give his church a block of stock with a fair market value of $110,000. Walter paid $50,000 for the stock six years ago. What is the maximum allowable charitable deduction Walter may receive during the current tax year if he makes a contribution of the stock? A)$50,000 B)$33,000 C)$60,000 D)$110,000

60000 The answer is $60,000. The allowable deduction for a contribution of long-term capital gain property, such as stock to a public charity, is limited to 30% of AGI. The remainder ($110,000 - $60,000 = $50,000) may be carried forward up to 5 years.

Dave owns equipment that has an adjusted basis of $10,000 and a fair market value of $75,000. Through an exchange, he acquires new equipment from Rachel that has a fair market value of $60,000 and an adjusted basis of $35,000. In the exchange, Dave receives $15,000 from Rachel. What is the amount of gain or loss, if any, recognized by Dave in the exchange? A)$65,000 B)$10,000 C)$50,000 D)$15,000

65,000 In the exchange, Dave received new equipment with a fair market value of $60,000 and cash of $15,000. He gave up an adjusted basis in his property of $10,000. The difference between $75,000 and $10,000 is the gain realized, $65,000. Because this is not realty, it is not eligible for 1031 exchange treatment. Thus, the transaction is treated as a sale and subsequent purchase. The entire gain of $65,000 is recognized and taxable.

During the current tax year, Carolyn has Section 1231 gains of $14,000 and Section 1231 losses of $3,000. Four years ago, Carolyn reported a net Section 1231 gain of $6,000. Three years ago, Carolyn had a Section 1231 loss of $4,000. Which of the following correctly describes the amount and treatment of the gain? A)$11,000 treated as capital gain income B)$7,000 treated as ordinary gain; $4,000 treated as capital gain C)$11,000 treated as ordinary income D)$7,000 treated as capital gain; $4,000 treated as ordinary income

7,000 treated as capital gain; $4,000 treated as ordinary income The current Section 1231 gain ($11,000) is treated as ordinary income to the extent of unrecaptured Section 1231 losses during the five-year lookback period. There are $4,000 of unrecaptured Section 1231 losses during the lookback period. Thus, $4,000 is ordinary income, and the remaining $7,000 is treated as long-term capital gain. LO 6.1.4

Tom Bell has investment income (interest) of $8,000 in the current year. He paid $1,200 in investment adviser fees and had $7,000 of investment interest expense. His AGI is $35,000. What amount of investment interest expense may be deducted in the current year as an itemized deduction? A) $7,000 B) $6,800 C) $6,500 D) $8,000

7000 Investment interest expense is deductible up to the amount of investment income. The investment income is the interest income of $8,000. However, the deduction cannot exceed the actual investment interest expense of $7,000. Historically, the adviser fees would impact the calculation, but the Tax Cuts and Jobs Act of 2017 (TCJA) eliminated the Tier II miscellaneous itemized deductions.

Bob passed away during the current year. He had suspended losses from a passive activity of $15,000. Bob's basis in the partnership was $1,000, and the fair market value at the time of his death was $9,000. Bob originally purchased the partnership interest for $25,000. What amount of passive losses, if any, is deductible on Bob's final return? A)$15,000 B)$0 C)$1,000 D)$7,000

7000 The suspended losses are "freed up" only to the extent that the losses exceed the step-up in basis. In this situation, the basis was stepped up by $8,000; subtracted from the $15,000 of losses leaves $7,000 of losses that are freed up

Jim made charitable contributions of cash to a local university, a qualifying charitable organization. Jim's cash contributions for the current year totaled $80,000. Jim has an AGI of $120,000. What is the amount of charitable contribution deduction that Jim may claim in the current year? A)$36,000 B)$60,000 C)$24,000 D)$72,000

72000 Gifts of cash to a 50% organization are limited to 60% of AGI under TCJA. Sixty percent of the $120,000 AGI equals $72,000. Jim would also have an $8,000 carryforward.

During the current tax year, Paul has the following items from his four investments: Passive income from a publicly traded limited partnership$15,000Passive loss from a publicly traded limited partnership$10,000Passive income from a nonpublicly traded limited partnership$8,000Passive loss from a nonpublicly traded limited partnership$16,000 What is the total amount, if any, of passive losses that may be deducted during the current year? A)$26,000 B)$10,000 C)$8,000 D)$16,000 PREVSKIP

8000 Only passive losses from nonpublicly traded limited partnerships may be offset against income from other nonpublicly traded limited partnerships. Thus, $8,000 of the $16,000 loss may be deducted. The passive loss from a PTP must be held in suspense until that same activity generates income (or until a taxable disposition of the PTP). LO 7.2.2

John is a single taxpayer. He has an annual salary of $88,000 per year. In addition, he had consulting income of $4,000, incurred losses of $5,000 from active participation real estate, and had short-term capital losses of $4,900. What is John's total (gross) income for the current tax year? A)$84,000 B)$87,000 C)$86,000 D)$82,100

84,000 The consulting income is included, meaning $84,000 is correct. The rental loss is allowable and used to calculate total income (Schedule E). The short term capital loss is capped at $3,000.

Mike has interest and short-term capital gain income of $9,000 in the current tax year. He paid broker commissions on security purchases of $1,000, paid $1,800 for investment adviser fees, and had $8,500 of investment interest expense. His AGI is $225,000. What amount of investment interest expense may be deducted as an itemized deduction?

8500 Investment interest expense is deductible up to the amount of net investment income, which is $9,000. The net investment income is simply the investment income (interest and short-term capital gains) of $9,000. Remember that the investment adviser fees were a Tier II miscellaneous itemized deduction, which are no longer deductible under the TCJA. The commissions are not a deductible item. The commissions increase the basis of the securities upon purchase and reduce the gain realized upon sale.

John has the following items from four separate investments during the current tax year: Passive income from a publicly traded limited partnership: $8,000 Passive loss from a publicly traded limited partnership: $10,000 Passive income from a nonpublicly traded limited partnership: $17,000 Passive loss from a nonpublicly traded limited partnership: $9,000 What is the total amount, if any, of passive losses that may be deducted during the current year? A) $19,000 B) $9,000 C) $0 D) $17,000

9000 Losses from a non-PTP may be deducted up to the amount of income from a non-PTP. In this situation, the passive loss of $9,000 may be deducted in full against the $17,000 of passive income. The income from a PTP may not be offset by passive losses arising from any other source, and the losses from a PTP must be held in suspense until that same partnership generates income.

Larry and Pam are married and will file a joint return for the 2021 tax year. Pam is an active participant in a company-maintained retirement plan, but Larry is not. They have provided you with the following information. Pam's divorce was finalized in 2018. Larry's salary$70,000Larry's IRA contribution$6,000Pam's salary$65,000Pam's IRA contribution$6,000Alimony payments to Pam's ex-spouse$9,600Itemized deductions$15,000 Based on the information given, what is their taxable income for the 2021 tax year? A) $89,900 B) $107,400 C) $94,300 D) $100,900

94300 The $135,000 in salaries is reduced by the alimony payment of $9,600 and Larry's IRA contribution of $6,000 to give an AGI of $119,400. The AGI is reduced by the greater of the itemized deductions ($15,000) or the standard deduction ($25,100 in 2021) to leave $94,300. Pam's IRA contribution is nondeductible because the MAGI exceeds the phaseout range of $105,000 to $125,000. Because the MAGI is less than $198,000, Larry may still deduct his full IRA contribution. Note that the MAGI ($125,400 in this situation) is the AGI computed without regard to the IRA deduction itself. The AGI phaseout limits for active participants will be provided on the exam.

Which of the following is NOT a step in the tax calculation process?

A) Calculate federal tax on total income. The following are involved in the income tax computation: subtracting adjustments to income from total income to get adjusted gross income, subtracting tax withholdings from total tax liability, and deducting the greater of itemized deductions or the standard deduction from AGI to arrive at taxable income. Credits are applied to tax liability. The calculation of federal tax is on federal taxable income. LO 1.1.2

Which of the following may be allowed as a like-kind exchange? A)A heifer exchanged for a bull B)Farmland exchanged for farming equipment C)A parking lot exchanged for a shopping center D)An apartment building located in San Diego exchanged for an apartment building located in Acapulco, Mexico PREV

A parking lot exchanged for a shopping center An exchange of U.S. realty for foreign realty is not considered like-kind. The like-kind requirements were changed under TCJA and now limits exchanges to realty for realty. The like-kind requirement does not mean that the property transferred must be identical to the property received; it merely requires that realty be exchanged for realty. LO 6.2.2

Which one of the following statements is true regarding self-employment taxes?

A taxpayer is allowed to deduct one-half of his self-employment tax liability as an adjustment to income. A taxpayer may deduct one-half of his self-employment tax liability as an "above the line" adjustment to income. The wage base is adjusted annually for cost of living increases. Net earnings from self-employment are determined under the same accounting method as that used for income tax purposes. Self-employed taxpayers are not subject to employer withholding.

On June 1, 2021, Hugh and Judy, both age 45, moved into a new house with a purchase price of $380,000. Hugh and Judy sold their previous home in January 2021. Their previous home sold for $650,000. Their basis in their old home was $305,000. They had lived in the old home for eight years. How much gain, if any, must Hugh and Judy recognize for tax purposes as a result of this sale? A)$0 B)$20,000 C)$345,000 D)$270,000

A) $0 The realized gain is $345,000. The entire gain may be excluded under Section 121. They had owned and lived in the residence for eight years, easily meeting the two years of ownership and use requirement. Thus, the recognized gain (the amount subject to taxation) is $0. The exclusion is not affected by the age of the taxpayers or the purchase of the new residence. LO 6.2.5

Jim owns an apartment building with a fair market value of $225,000 and an adjusted basis of $85,000. He wants to acquire Frank's duplex, which has a fair market value of $240,000 and an adjusted basis of $130,000. In the exchange, Jim will pay Frank $15,000 in cash. What is Jim's substitute basis in the acquired duplex? A) $100,000 B) $140,000 C) $225,000 D) $240,000

A) $100,000 Jim is receiving an FMV of $240,000 for the duplex. He is giving up an adjusted basis of $85,000 plus $15,000 cash. The difference between the $240,000 received and the $100,000 given up is the realized gain of $140,000. The gain recognized (the taxable amount reported on the income tax return) in a like-kind exchange is the lesser of gain realized ($140,000) or boot received ($0). The substitute basis in an asset acquired in a like-kind exchange is the FMV of the qualifying property received ($240,000) reduced by the gain realized, but not recognized ($140,000 - $0 = $140,000). Thus, $240,000 - $140,000 = $100,000.

On June 1, 2019, Hugh and Judy, both age 45, moved into a new house that they had custom-built at a cost of $524,000. Because of a slow real estate market in their area, Hugh and Judy were unable to sell their previous home until January 1, 2021. Their previous home, which they had lived in for 15 years, sold for a price of $916,000. Their basis in their old home was $312,000. How much gain, if any, must Hugh and Judy recognize for tax purposes as a result of this sale? A)$104,000 B)$604,000 C)$0 D)$92,000

A) $104,000 The gain realized is $604,000. This is the difference between the sales price of the residence—$916,000—and the basis of $312,000. They lived in the new home for only 19 months before the old home sold. Thus, for the previous home, they meet the two-out-of-five-year rule allowing for up to a $500,000 exclusion of gain on the sale of a principal residence under Section 121. The realized gain of $604,000, reduced by the $500,000 exclusion, leaves a gain recognized (subject to tax) of $104,000

Reese and Belinda are married taxpayers who will file separately in 2021. What is the gross income threshold that will trigger filing an income tax return for Reese or Belinda? A)$12,550 B)$14,250 C)$25,100 D)$18,800

A) $12,550 Married individuals who file separately must file if income equals or exceeds the standard deduction amount for a single taxpayer ($12,550 in 2021).

An office building with an adjusted tax basis of $120,000 was destroyed by fire on January 2 of last year. On January 15 of the current year, the insurance company paid the owner $200,000. The owner reinvested $190,000 in a new office building. What is the basis of the new building under Section 1033 (the involuntary conversion rules)? A)$120,000 B)$190,000 C)$180,000 D)$110,000

A) $120,000 Gain realized equals $80,000, and gain recognized equals $10,000 ($200,000 amount realized − $190,000 amount reinvested). This is a net postponed gain of $70,000 ($80,000 − $10,000). The basis of the new building equals $120,000 ($190,000 − $70,000).

Claudia, who has an AGI of $40,000, wants to donate a portrait of an ancestor who served in the American Revolution to the museum in her town that houses a collection of Revolution Era items. Her basis in the portrait is $1,750, and it has a fair market value of $2,000. How much can she potentially deduct as a charitable contribution this year, assuming it is her only donation? A) $2,000 (It is related-use capital gain property.) B) $1,750 (It is related-use capital gain property, so she must use basis.) C) $600 (The museum is a 30% organization, so she must use FMV.) D) None (It is the portrait of a relative.)

A) $2,000 (It is related-use capital gain property.) The portrait is related-use, capital gain property. Claudia may deduct an amount up to 50% of her AGI if she uses the basis of the painting and 30% of AGI if she uses FMV. As long as Claudia's AGI is greater than $6,667, she can deduct the FMV of the portrait.

Dwight has an active participation rental real estate activity. In 2020, he had losses of $20,000 from the active participation real estate and his AGI was $140,000. He was allowed to deduct $5,000 of the losses against other income. The remaining $15,000 loss was carried forward into 2021. In 2021, Dwight has an AGI of $90,000 and only $6,000 of current losses from his real estate rental activity. What amount of loss, if any, may Dwight deduct in 2021? A) $21,000 B) $0 C) $6,000 D) $15,000

A) $21,000 Dwight is allowed to deduct $21,000 of losses in 2021. The $15,000 carryforward losses are treated as if they occurred in 2021. The $6,000 current losses plus the $15,000 carryover losses total $21,000. This loss would be fully deductible as the AGI is under $100,000.

Eric purchased a painting for $200,000. He sold the artwork 5 years later for $500,000 in an installment sale, receiving $50,000 as the first-year payment. How much gain must Eric recognize in the year of sale? A)$30,000 B)$20,000 C)$500,000 D)$50,000

A) $30,000 Eric would recognize a capital gain of $30,000 in the year of sale, calculated as follows: Gross profit percentage (60%) = profit ($300,000) ÷ total contract price ($500,000). Gain recognized ($30,000) = gross profit percentage (60%) × installment payment ($50,000). LO 6.2.4

Three years ago, Sam received a gift of 100 shares of common stock from his uncle. The fair market value of the stock on the date of the gift was $12 per share. His uncle had purchased the stock four years earlier at $5 per share. Sam sold this stock for $17 per share last week. What was Sam's per-share basis in the stock when it was sold? A) $5 B) $12 C) $17 D) $22

A) $5 If the fair market value on the date of the gift is greater than the donor's adjusted basis, the donor's adjusted basis is used as the recipient's basis. Note that the donor's holding period would be tacked to the donee's holding period.

Margo files her tax return 39 days after the due date. Along with the return, she remits a check for $6,000 (the balance of the tax owed). Disregarding any interest element, her combined failure-to-file and failure-to-pay penalties are A)$600. B)$400. C)$660. D)$440.

A) $600. The failure-to-file penalty is netted against the failure-to-pay penalty: $60 + ($600 − $60) = $600.

During the current tax year, Carolyn has Section 1231 gains of $14,000 and Section 1231 losses of $3,000. Four years ago, Carolyn reported a net Section 1231 gain of $6,000. Three years ago, Carolyn had a Section 1231 loss of $4,000. Which of the following correctly describes the amount and treatment of the gain? A)$7,000 treated as capital gain; $4,000 treated as ordinary income B)$11,000 treated as ordinary income C)$11,000 treated as capital gain income D)$7,000 treated as ordinary gain; $4,000 treated as capital gain

A) $7,000 treated as capital gain; $4,000 treated as ordinary income The current Section 1231 gain ($11,000) is treated as ordinary income to the extent of unrecaptured Section 1231 losses during the five-year lookback period. There are $4,000 of unrecaptured Section 1231 losses during the lookback period. Thus, $4,000 is ordinary income, and the remaining $7,000 is treated as long-term capital gain. LO 6.1.4

Dr. John Welch, a dentist, is a single taxpayer. For the 2021 tax year, he has a taxable income of $300,000. Included in the taxable income is $50,000 of net long-term capital gains from the sale of an antique clock collection. At what rate will his net capital gain be taxed? A)28% B)20% C)35% D)25%

A) 28% The long-term capital gain rate for collectibles is 28% if the taxpayer is in a marginal income tax bracket greater than 28%. John's taxable income places him in the 35% marginal income tax bracket, so his collectibles gain is taxed at 28%.

If the Taylors rented their condo for 300 days last year, how many days could the Taylors have used it personally and still maintained the classification of the condo as primarily a rental use property? A)30 days B)3 days C)14 days D)0 days

A) 30 days The Taylors could have used the property personally for a total of 30 days (300 rental days × 10%). If rental property is rented at least 15 days a year and is not used for personal use more than the greater of 14 days per year or 10% of the rental days, it is classified as primarily rental use. This treatment permits the Taylors to deduct expenses associated with the rental (not personal) use on Schedule E of IRS Form 1040.

If Stewart and Hope file married filing separately and Hope has gross income of $300,000 and taxable income of $267,000, what is her marginal tax rate? Refer to the 2021 tax table provided in your course references

A) 35.00%

Which one of the following is NOT a type of tax audit performed by the IRS? A) A home audit B) A field audit C) A correspondence audit D) An office audit

A) A home audit The three types of tax audit performed by the IRS are as follows: a correspondence audit, which is usually performed through the mail because the disputed tax issue is minor; an office audit, which is usually restricted in scope to a specific item or items and is performed at the IRS office by an office auditor; and a field audit, which is an examination of numerous items and is usually performed on the premises of the taxpayer (such as a business office) by a revenue agent. LO 8.2.2

Which of the following businesses may be considered a specified service business (SSB) for purposes of the qualified business income (QBI) deduction? A) A medical practice B) An engineering firm C) An automobile repair shop D) A furniture manufacturer/wholesaler

A) A medical practice An SSB is any trade or business involving the performance of services in health, law, accounting, actuarial science, performing arts, consulting, athletics, financial services, brokerage services, or any trade or business where the principal asset of such trade or business is the reputation or skill of one or more of its employees; or, any trade or business that involves the performance of services that consist of investing and investment management, trading, or dealing in securities, partnership interests, or commodities. Thus, the medical practice would be an SSB, but none of the other businesses would. Engineering and architecture are specifically excluded from the definition of SSB.

Which of the following dispositions of Section 1245 recapture property would result in the immediate recapture of some or all of previous depreciation deductions? A) A sale for cash and an interest-bearing note B) A distribution by a partnership to its partners C) A gift of the property D) A transfer at death

A) A sale for cash and an interest-bearing note A sale of Section 1245 property at a gain will result in Section 1245 recapture. None of the other choices are considered taxable dispositions that would trigger recapture of depreciation (cost recovery) deductions.

Seven years ago, Karen Price purchased U.S. EE savings bonds for $5,000. During the current year, when Karen was 27 years old, she redeemed the bonds to help pay for her graduate school tuition. The accrued value at the time of redemption was $7,000. Assume Karen incurs $11,000 of tuition expenses in the year. What are the tax consequences upon the redemption of the bonds? A) All accrued interest is taxable in the current year. B) The income on the bonds is generally subject to state income taxes. C) A portion of the interest may be excluded. D) All the interest may be excluded.

A) All accrued interest is taxable in the current year. The exclusion for EE bond interest redeemed to pay for qualifying higher-education expenses applies only to bonds purchased by an individual age 24 or older, and held in that person's name, or jointly with a spouse. Karen is 27 years old; the bonds were purchased 7 years ago, when Karen was approximately 20. Because Karen does not qualify for the exclusion of the interest income because she was not age 24 or older at the time of purchase. All the interest is taxable in the year the bonds are redeemed. Remember that the interest of EE bonds is deferred until maturity, unless an election has been made to have the interest taxed each year as it accrues. Also, the interest income from EE bonds (and other federal government obligations) is generally not subject to state income tax

Gil owns a portfolio of income-producing real estate. Gil retains ownership of the real estate but directs that the rental income be paid to his son, Kevin. The income is paid directly to Kevin, who reports it as part of his taxable income. Gil does not report the income on his tax return. With which one of the following potential tax traps should Gil be most concerned? A) Assignment of income B) Substance over form C) Constructive receipt D) Ownership attribution rules

A) Assignment of income The fact that Gil retains ownership of the property and merely assigns the income to someone else is a potential tax trap for him. The assignment of income doctrine serves to tax the person who actually owns the property producing the income. The income cannot merely be assigned to another in order to generate tax advantages. LO 8.2.3

Lois and Clark recently divorced. Clark was ordered to pay child support and alimony. Clark works as a police officer at the local precinct. Lois expressed concern that he may be killed in the line of duty. Therefore, the judge has ordered him to obtain a life insurance policy. Which of the following is true? A)Clark can treat the premiums as alimony. B)Clark should claim he is uninsurable because of his profession. C)Clark must be the owner of the policy. D)Clark must pay the life insurance premiums in addition to alimony.

A) Clark can treat the premiums as alimony. Some states require alimony payors to obtain life insurance. Clark can include the money he spends on life insurance as part of his overall alimony payment. Lois has the insurable interest and therefore should be the owner of the policy. Although Clark's premiums may be higher due to his profession, his profession will not prevent him from becoming insured.

Which one of the following is CORRECT regarding the Coverdell Education Savings Account? A) Distributions may be tax free even if made for K-12 expenses. B) Room and board may be covered with a tax-free distribution only if the student is full-time. C) Distributions may be tax free only if made for a full-time student. D) Deductible contributions of up to $2,000 may be made per beneficiary.

A) Distributions may be tax free even if made for K-12 expenses. The predominant benefit of the Coverdell ESA is distributions may also be used to pay for K-12 expenses. This is unlike the 529 plan which is designed primarily to pay for college expenses (Note: a limited amount of $10,000 may now be withdrawn from a 529 for K-12 expenses per the TCJA).

Which one of the following statements is incorrect regarding investment interest expense? A) Excess investment interest expense cannot be carried forward into succeeding tax years. B) Interest paid or accrued to purchase or carry tax-exempt investments is not deductible. C) Net investment income is the taxpayer's investment income—typically interest, nonqualified dividends, and short-term capital gains. D) Investment interest expense is deductible up to the amount of the net investment income.

A) Excess investment interest expense cannot be carried forward into succeeding tax years. Excess investment interest expense can be carried forward into succeeding tax years. Investment interest expense is deductible up to the amount of net investment income. The interest on funds borrowed to purchase tax-exempt investments is not deductible. The net investment income is typically interest, nonqualified dividends, and short-term capital gains. Long-term capital gains and qualified dividends may be included at the taxpayer's election, but the taxpayer must forgo the preferential tax rates on these items.

Now that their parents have died, Joseph is assuming the responsibility for the care of his 25-year-old disabled sister. Joseph has told his financial planner there is a trust for his sister to provide funds sufficient for her to live in an assisted-living facility or to reimburse him for the cost of an at home caregiver and his sister's other expenses if she stays in his home. He would prefer for his sister to live with him but does not want to cause her any problems with tax issues on any income she gets if she does. His sister is adamant that she does not want to be anyone's dependent. What advice can the planner give to Joseph? If his sister's income from the trust is paying for all of her needs, then she is not a dependent on anyone's income tax return. If Joseph's sister lives in an assisted-living facility, she cannot be a dependent of Joseph. A) I only B) Neither I nor II C) Both I and II D) II only

A) I only Statement I is correct. If all of his sister's support is paid for by funds she receives from the trust, she cannot be a dependent of Joseph. Statement II is incorrect. Simply not living with Joseph does not preclude him from listing his sister as a dependent. If Joseph provided more than half of the support for his sister, he would be allowed to list her as a dependent and take any tax credits that may be available.

In 2021, which of the following is a method in calculating total self-employment tax, where net income from self-employment is above the taxable wage base? Calculate self-employment income; multiply by 0.9235 (1 − 0.0765); subtract the taxable wage base, and from that, multiply the excess over the taxable wage base by 2.9% (Medicare portion of the tax). Multiply the taxable wage base by 15.3%, and add the results of the previous two steps together to arrive at the total self-employment tax. Calculate self-employment income; multiply by 0.9235 (1 − 0.0765); and multiply the result by 15.3%. Calculate self-employment income; multiply by 14.13%. Calculate self-employment income; multiply by 15.3%; multiply the excess over the taxable wage base by 2.9% (Medicare portion of the tax); and add the results of the previous two steps together to arrive at the total self-employment tax. A) I only B) I and II C) II and III D) I and IV

A) I only The steps to calculate self-employment tax for 2021 where net income from self-employment is above the taxable wage base are as follows: Step 1: Calculate self-employment income. Step 2: Subtract 7.65% or multiply by 0.9235 (1 − 0.0765). Step 3: From Step 2, subtract the taxable wage base and multiply the excess over the taxable wage base by 2.9% (Medicare portion of the tax). Step 4: Multiply the taxable wage base by 15.3%. Step 5: Add the results of Steps 3 and 4 together to arrive at the total self-employment tax.

Gary has just divorced. He is asking his planner, Ruth, for recommendations of amending his financial plan given his newly single status. Which of the following recommendations should Ruth make? Gary should review the beneficiary designations on his life insurance policies to be certain the beneficiaries are in line with his wishes post-divorce. Ruth should ask for any documentation on property settlements and other court-ordered financial transactions. Ruth should inquire whether a qualified domestic relations order was issued by the court and obtain a copy to ascertain its effect on Gary's financial plan. Ruth should inform Gary that the transfer-for-value income tax rule makes the transfer of any life insurance policies a taxable event. A) I, II, and III B) I and IV C) III only D) II and IV

A) I, II, and III Statement IV is incorrect. The transfer-for-value income tax rule does not apply in situations when a life insurance policy is transferred from one spouse to another as a result of a property settlement.

Your client, Mary, is contemplating the formation of an S corporation for her small manufacturing business. What can you accurately tell Mary about forming an S corporation? Mary would have limited liability. The number of shareholders is limited to 100. Mary's death would require reorganization of the corporation. The corporation's items of income, deductions, and tax credits would flow through to Mary's income tax return. A) I, II, and IV B) II and III C) III only D) I and IV

A) I, II, and IV The S corporation form is similar to the partnership in operating as a conduit for tax purposes. It also is similar to a C corporation, since it features limited liability for shareholders. In addition, the death of a shareholder in an S corporation does not create a need to reorganize the business. Finally, unlike a C corporation, an S corporation may have no more than 100 shareholders. LO 5.2.2

Which of the following statements regarding the alternative minimum tax (AMT) or AMT planning are CORRECT? The AMT reduces the tax benefits from certain types of deductions and tax preferences allowable for regular income tax purposes. The starting point for determining alternative minimum taxable income (AMTI) is AGI as reported for regular income tax purposes. It is generally advantageous to defer the payment of real estate taxes to a future year when AMT will be paid in the current year. It is generally advantageous to accelerate ordinary income into years when AMT will be paid. A) I, III, and IV B) I, II, and IV C) III and IV D) I and III

A) I, III, and IV The IRS notes that the starting point for determining AMTI is taxable income as reported for regular income tax purposes on a taxpayer's IRS Form 1040. Because real estate taxes are not deductible for AMT purposes, it is generally advantageous to defer the payment of such taxes to a year when AMT will probably not be payable. Also, if AMT will be payable in the current year, it is generally advantageous to increase the amount of regular taxable income (e.g., by accelerating ordinary income into the current year) because the total tax payable will likely not be affected by doing so.

Your client, Sally, is considering investing in real estate as a way to diversify her investments. She has heard of active participation rental real estate, but is unsure of the requirements that must be met. What can you accurately tell her with respect to active participation rental real estate? The interest may not be held through a limited partnership. A deduction-equivalent tax credit of up to $25,000 is available. The taxpayer must hold a 10% or greater ownership interest. The taxpayer must make the major management decisions. A)I, III, and IV B)III and IV C)I and IV D)I and II

A) I, III, and IV The deduction for active participation rental real estate requires that the taxpayer participate in management in a bona fide sense; making the major management decisions. To qualify, the interest may not be held through a limited partnership, and the taxpayer must have at least a 10% or greater ownership interest in the property. The loss is deductible when computing the gross income. Active participation rental real estate generates an above-the-line deduction of up to $25,000 annually, not a credit.

You are working with new financial planning clients, Dan and Patrice Harden, on educational funding issues for their three dependent children. Dan and Patrice will file jointly this year, as they have always done, and anticipate an AGI of $110,000. Their oldest child, Ben, age 23, is in his first year of graduate school studying to be a pharmacist. Ben graduated from college in three years, so the American Opportunity Tax Credit was claimed for three years of Ben's education. Their middle child, Margaret, age 19, had a conviction for felony drug possession last year. She's "cleaned up her act" and will be a full-time student at a community college this year. Their youngest child, Francis, age 7, is a good kid who wants to be a doctor when he grows up. Which of the following statements concerning educational tax credits and incentives is CORRECT? Ben and Margaret would each qualify for the American Opportunity Tax Credit (AOTC). Ben and Margaret would each qualify for the Lifetime Learning Credit. Dan and Patrice may make a contribution to a Coverdell account for Francis. Only Ben would qualify for the Lifetime Learning Credit

A) II and III Ben and Margaret both qualify for the Lifetime Learning Credit. The Lifetime Learning Credit may be claimed for graduate studies, and may be claimed for a child with a felony drug conviction. Note that the Lifetime Learning Credit would allow a total of $10,000 of education expenses to be utilized on Dan and Patrice's tax return. (The Lifetime Learning Credit limitation of $10,000 is applied per return, whereas the AOTC limitation of $2,500 is per student.) The parent(s) who claims a child as a dependent is entitled to take the tax credit for the educational expenses of the child. The AGI is under the phaseout thresholds for the Lifetime Learning Credit. Even though the AOTC may generally be claimed for four years, Ben does not qualify for the AOTC because it may not be claimed for graduate work, only the first four years of undergraduate work. The AOTC may not be claimed for a child who has a felony drug conviction, so Margaret does not qualify for the AOTC. Dan and Patrice may make a Coverdell contribution for Francis, because their AGI does not exceed the phaseout limit of $190,000 to $220,000 for a married couple filing jointly. (The AGI limitations will be provided on the examination.)

During the current tax year, Jim purchased a warehouse for exclusive use in his manufacturing business. The cost of the property was $620,000, of which $100,000 was attributable to the land. Which of the following statements identify the proper treatment of the expenditure? A portion of the cost attributable to the building may be deducted under Section 179. The $100,000 attributable to the land must be capitalized and may not be depreciated. The $520,000 attributable to the building must be capitalized and depreciated. The entire $620,000 must be capitalized and depreciated. A) II and III B) IV only C) II only D) I and II Explanation

A) II and III Land is not a depreciable asset—only "wasting" assets are subject to depreciation. The building must be capitalized and depreciated over a period of 39 years. Section 179 generally does not apply to realty; it applies to tangible personalty used in the active conduct of a trade or business. LO 6.1.5

Which of the following are characteristics of a valid and enforceable premarital agreement? It may be orally executed by the parties that are affected. There should be a full and complete disclosure of each party's net worth prior to signing. It may be used to regulate an award of alimony upon divorce of the parties. There should be a written agreement with the willingly executed signatures of both parties.

A) II and IV To be valid, a premarital agreement must be in writing and contain a complete disclosure of each party's financial situation. It may not be used to regulate an award of alimony.

Which of the following statements regarding installment payments pursuant to a divorce is CORRECT? It is usually in the best interest of the recipient of the funds to spread the amount of payments over as many years as possible to take advantage of the time value of money. It is usually in the payee's best interest to immediately receive as much money as possible rather than receiving payments over a longer period. A)II only B)I only C)Neither I nor II D)Both I and II

A) II only A common issue that arises in the settlement of divorce proceedings is how payments are best structured in resolution of marital obligations. As a general rule, it is usually in the best interest of the payor to spread the amount of payments over as many years as possible to take advantage of the time value of money. Alternately, the payee's best interests are served by immediately receiving as much money as possible rather than receiving payments over a longer period.

As part of the property settlement after Lori and Gordon divorced, Gordon transferred ownership of a life insurance policy to Lori. Lori is the beneficiary of the policy, and Gordon is the insured. Which of the following statements regarding the property settlement is CORRECT? The transfer of the life insurance policy is subject to the transfer-for-value rule. The death proceeds of the policy will be income tax free at Gordon's death. A) II only B) Neither I nor II C) Both I and II D) I only

A) II only Statement I is incorrect; the transfer-for-value rule does not apply when a life insurance policy is transferred from one spouse to the other under a property settlement incident to divorce. Statement II is correct.

Fred runs a small business that has been in the family for over 50 years. The business has always used the cash basis method of accounting. Which of the following would be considered income to Fred last year? A bonus he declared for himself on December 31 of last year that is not payable until January 2 of the following year Dividends received on December 30 of last year Salary received during last year Interest on his money market account last year, but not posted until January 2 of the next year A) II, III, and IV B) I and II C) II and III D) I, II, and III

A) II, III, and IV For the cash method of accounting, all income actually received during the tax year is included in gross income (e.g., dividends, interest, wages). Income received constructively during the year, even though actual receipt is delayed, is includible for the tax year. Bonuses declared but not yet payable until the next tax year are not includible for the prior tax year because there was no constructive receipt.

Which of the following is subject to the self-employment tax? Distributive share of limited partnership operating income Flow-through of S corporation income Distributive share of general partnership operating income Interest or dividends from investments A) III only B) I, II, III, and IV C) I and III D) II and IV

A) III only The general partnership operating income is self-employment income. By definition, the other items of income are not subject to the self-employment tax.

Charley lent his friend, Richard, $17,000 for a down payment on a home in a no-interest loan early in the current year. Charley had investment income of $750, and Richard had investment income of $1,200 in the same year. The federal interest rate is 3.5%. Richard has been making payments each month. What recommendations do you make for accounting for the loan made to Richard by Charley? A) Imputed interest is calculated on the loan to Richard and is considered a gift to Richard from Charley. B) Charley must develop an amortization schedule using the federal rate of 3.5% to account for Richard's payments of principal and interest. C) Because Charley's investment income is less than $1,000 this year, no interest is imputed to the loan. D) Because this is a gift loan greater than $10,000 but less than or equal to $100,000, no interest will be imputed to the loan.

A) Imputed interest is calculated on the loan to Richard and is considered a gift to Richard from Charley. In a gift loan, the amount of the imputed interest constitutes a gift from the lender to the borrower. For gift loans greater than $10,000 and less than or equal to $100,000, no interest is imputed if the borrower's investment income for the year does not exceed $1,000. For a gift loan of more than $100,000, the prevailing federal rate of interest will be imputed. For this loan, Richard's investment income exceeds $1,000 and interest will be imputed.

To be considered a responsible person by the IRS, which one of the following is among the important factors? A) In charge of hiring and firing employees B) Shareholder of a similar company C) No authority to sign checks D) Impressive title

A) In charge of hiring and firing employees The determination of whether someone is a responsible person is a test of facts and circumstances. Some of the questions the IRS usually asks in determining responsibility are as follows: Was the individual an officer or director? Was the individual a shareholder? Was the individual a member of a board of directors? Did the individual have the authority to sign checks? Was the individual responsible for hiring and firing employees? Did the individual have actual authority or merely an impressive title?

Garret has the following items of income: $1,500 of interest income, $2,800 of qualified dividend income (he has not decided whether to have it taxed at the ordinary or capital gain rate), and a salary of $100,000. Which of these are classified as portfolio income? A)Interest income and dividend income B)Salary only C)Interest income, dividend income, and salary D)Interest income only

A) Interest income and dividend income

Which of the following statements regarding material participation is CORRECT? A)It usually refers to a taxpayer's main business. B)It only applies to portfolio income and losses. C)It is a much less rigorous standard than active participation. D)It only applies to real estate and associated rentals. PREV

A) It usually refers to a taxpayer's main business. Material participation is a much more rigorous standard than active participation. The statute provides that a taxpayer is materially participating only if he or she is involved in the operations of the business on a regular, continuous, and substantial basis.

Which one of the following types of audits is conducted on a random basis? A) National Research Program audit B) Targeted Program audit C) Discriminant Functions System Program audit D) Document Matching Program audit

A) National Research Program audit The National Research Program audit is the only random audit that the IRS conducts. The NRP is, in effect, the replacement for the old Taxpayer Compliance Measurement Program audit. The Discriminant Functions System Program (DIF) compares the information on each return to a set of norms, weighs each item, and ranks the return for audit-worthiness. The Document Matching Program allows the IRS to detect discrepancies between the amounts reported on a tax return and the amounts shown on information documents (e.g., W-2 forms, 1099 forms). Under the Targeted Program Audit, the IRS "targets" particular taxpayers in particular situations.

If an individual taxpayer files an Extension Form 4868 on a timely basis in 2021, the 2020 federal income tax return is due (ignoring weekends or holidays), at the latest, by A) October 15, 2021. B) August 15, 2021 C) July 15, 2021. D) April 15, 2021.

A) October 15, 2021. The Form 4868 provides an automatic six-month extension of time to file the Form 1040. The return would be due April 15 without regard to extensions. With the six-month extension, the due date is October 15.

Old Citywear, Inc., manufactures clothing in styles from the 1940s and 1950s. The company has been in existence four years and had the following taxable income or loss: Tax YearTaxable Income2018$5,0002019$15,0002020$15,0002021($25,000) Which of the following statements regarding Old Citywear's net operating loss in 2021 is CORRECT? A)Old Citywear may carry forward the $25,000 loss indefinitely. B)Old Citywear may carry back $5,000 of its 2021 NOL to 2020. C)Old Citywear may carry back $15,000 of its 2021 NOL to 2020. D)Because it cannot utilize all of the net operating loss in 2021, Old Citywear may carry back the loss to prior tax years.

A) Old Citywear may carry forward the $25,000 loss indefinitely. NOLs cannot be carried back but can be carried forward indefinitely following the loss year. The loss is limited in amount to 80% of the total of taxable income reported for the carryforward years.

Paul was divorced from his spouse, Pat, late in 2021. As part of the property settlement agreement, Paul agreed to transfer his interest in a residential rental property to Pat in exchange for release of marital claims. Paul's cost basis in this real estate tract was $50,000. The tract was appraised at a fair market value of $100,000 at the time of its transfer to Pat. Which of the following is an income tax implication of Paul's transfer of the real estate tract to Pat? A) Paul's basis in the real estate is carried over to Pat for income tax purposes. B) Pat receives a basis in the real estate equal to the fair market value at the time of transfer. C) Paul is allowed a deduction equal to the excess of the fair market value over his basis in the property. D) Paul must recognize the gain on the real estate at the time of transfer as ordinary income.

A) Paul's basis in the real estate is carried over to Pat for income tax purposes. When there is a transfer of property incident to divorce, the basis simply carries over to the other spouse. The transfer is not a taxable event. Pat will likely owe capital gains tax when disposing of the property.

Which of the following statements concerning alimony is CORRECT? A)Payments made with respect to jointly owned property are considered one-half alimony. B)No payments except cash can be considered alimony. C)Payments to maintain property used by the payee spouse, but owned by the payor spouse, qualify as alimony. D)Cash payment of the payee spouse's mortgage made by the payor spouse as required by the divorce or separation instrument qualifies as one-half alimony.

A) Payments made with respect to jointly owned property are considered one-half alimony. Cash payment of the payee spouse's mortgage, rent, tuition, or tax liability made by the payor spouse as required by the divorce or separation instrument may qualify as alimony. Payments to maintain property used by the payee spouse, but owned by the payor spouse, do not qualify as alimony, even if required under the instrument. Payments made with respect to jointly owned property are considered one-half alimony. These property-related expenditures may include mortgage payments, real estate taxes, and homeowners insurance. LO 4.2.2

Which one of the following is a statute of limitations that restricts the IRS in auditing a return? A) Six years if 25% of gross income is unreported B) Twelve years if 50% of gross income is unreported C) Seven years from the filing date of the return or due date, if later D) Ten years for failure to file or if a fraudulent return is filed

A) Six years if 25% of gross income is unreported After the statute of limitations has passed, except in cases of fraud, the IRS cannot audit a return. The statutes to be aware of are as follows: three years from the filing date of the return or due date if later, six years if 25% of gross income is unreported, and no statute of limitations for failure to file or if a fraudulent return is filed.

Which of the following may enable a direct participation program to provide specific tax advantages to the investors? A)Special allocations B)Partnership basis rules C)At-risk rules D)Passive activity loss rules

A) Special allocations The benefits that flow through from a partnership entity may be enhanced by the potential, under certain circumstances, "special allocation" of certain items of income, expense, gain, or loss. The passive activity loss rules state that passive losses may only be deducted against passive income (not a tax advantage); there are no partnership basis rules; and the at-risk rules are defined as the maximum deductible loss for an investment limited to the amount that the taxpayer-investor has at risk at the end of the current year. None of the other answers are direct participation programs, which concern business organizations that function as tax conduits. LO 7.1.1

Which one of the following steps occurs in the tax calculation process?

A) Tax liability minus tax credits equals refund or tax owed

Which one of the following is advice from the National Office of the IRS requests that can give both the taxpayer and a revenue agent an opportunity to resolve a dispute? A) Technical Advice Memoranda B) Notice C) Announcement D) Private Letter Rulings

A) Technical Advice Memoranda Technical Advice Memoranda normally take place during an audit or during the appeals process of the audit; they give both the taxpayer and the revenue agent an opportunity to resolve a dispute over a technical question. Private Letter Rulings are taxpayer guidance from the IRS that apply only to the particular taxpayer(s) asking for the ruling; they are not applicable to all taxpayers. A Notice is a public pronouncement that contains official guidance about regulations or interpretations of the Code. Announcements often summarize code sections in layman's terms, or notify taxpayers of impending deadlines; they have only short-term value. LO 8.1.1

For two years, Lisa Carson was able to pay the premiums on her whole life policy without borrowing. For the past two years, she has borrowed from the cash value of her whole life policy to pay the premiums. Last year, she paid $95 of interest on the funds she borrowed. What are the tax implications in this situation? A) The interest expense is not tax deductible. B) The interest is deductible because Lisa is in the business of continuing her insurance and the interest is deductible business interest expense. C) The interest expense is tax deductible because it does not exceed $100. D) The interest expense is not tax deductible because it does not exceed $100.

A) The interest expense is not tax deductible. The interest expense is not tax deductible because interest on a loan incurred to purchase personal life insurance protection is considered personal interest, which is not deductible. Personal loan interest is not tax deductible, regardless of whether the lender is a bank or a life insurance company.

Which of the following statements concerning the passive activity loss rules is NOT correct? A) The passive activity loss rules limitation is a permanent disallowance rule. B) The $25,000 offset allowance for the small real estate investor is not available to taxpayers whose AGI is $150,000 or more. C) Losses from one master limited partnership activity may only offset income from that particular activity; they cannot be used to offset income from any other passive activities. D) Losses from passive activities may not offset portfolio or active income except under limited circumstances.

A) The passive activity loss rules limitation is a permanent disallowance rule. The passive activity loss limitation is not a permanent disallowance rule. When a taxpayer disposes of his entire interest in a fully taxable transaction to an unrelated purchaser (not a related party), his suspended losses from that activity, including any losses incurred in the year of disposition, are generally deductible in full.

Bob and Bonnie were divorced in 2016. As a result of a court order, Bob pays Bonnie $700 per month in alimony. He makes each month's payment with a money order. Earlier this year, Bonnie moved in with Bob, and they now share a two-bedroom apartment. Which of the following statements is accurate concerning the alimony payments by Bob? A) The payments are not deductible because the taxpayers are living together at the time of payment. B) The payments are not deductible because the payments must be in cash. C) The payments are deductible because they are equal each year of the agreement. D) The payments are deductible because they are being made as a result of a court order.

A) The payments are not deductible because the taxpayers are living together at the time of payment. The requirement that the payments be in cash means that the payments may not be in the form of property. Payments are not necessarily deductible just because they are a result of a court order; there are other requirements that must be met as well. One of those requirements to have qualifying alimony is that the taxpayers must not be living together at the time of payment. The payments do not need to be equal each year. However, unequal and declining payments can trigger the alimony recapture (excess front-loading) rules.

Which of the following is most often considered an advantage of an S corporation? A)There is pass-through of ordinary loss. B)Two classes of shareholders are permitted. C)The number of shareholders is limited. D)The business may be a foreign corporation.

A) There is pass-through of ordinary loss. The ability to have losses flow through to the shareholders to offset other income is one of the primary advantages of the S corporation. The limitation on the number of shareholders is not considered an advantage. Only one class of stock is permitted, the business must be a domestic corporation, and there are restrictions on who may be a shareholder.

Which of the following would result in higher taxation of one party with a premarital agreement? A) Transfer for consideration is made B) Transfer for premarital is made C) Transfer under the agreement is treated as a gift D) Transfer under the agreement is treated as an estate

A) Transfer for consideration is made The income tax consequences of the premarital agreement depend in large part upon whether the transfer under the agreement is treated as a gift (where income tax is avoided) or as a transfer for consideration (which will probably result in the recognition of significant income by one party).

Which of the following statements best describes a tax benefit associated with an active participation rental real estate investment? A)Up to $25,000 in losses may be used to offset active or portfolio income for certain taxpayers. B)The first $25,000 of taxable income from the investment is nontaxable. C)The investment will generate portfolio rather than passive income. D)Unlimited losses may be used to offset active or portfolio income of any taxpayer. PREV

A) Up to $25,000 in losses may be used to offset active or portfolio income for certain taxpayers. Active participation rental real estate investment operating losses are deductible against active or portfolio income up to $25,000 per year, assuming the investor's AGI is $100,000 or less.

Jerry owns a dry-cleaning business. During the current year, Jerry purchased and placed into service $730,000 of equipment. He had taxable income of $745,000. Jerry is in the highest marginal income tax bracket this year, and expects to be in that bracket for two more years. After that, he plans to semi-retire, but keep the business open for another five years. He expects to drop into the lowest marginal bracket when he semi-retires. What advice would you give Jerry regarding the use of Section 179, bonus depreciation, and cost recovery deductions? A) Use the bonus depreciation provision. B) Forgo Section 179 and bonus depreciation and use the Modified Accelerated Cost Recovery System (MACRS) table. C) Forgo Section 179 and bonus depreciation and elect the straight-line method. D) Elect the maximum Section 179 and elect the straight-line method.

A) Use the bonus depreciation provision. The fact pattern indicates that Jerry is in the highest marginal bracket for three years, and then will be in the lowest marginal bracket after that. It makes sense to maximize the depreciation deduction this year when Jerry is in the highest marginal bracket. By using the bonus depreciation provision, the entire $730,000 may be deducted in the year of acquisition.

Maxine, an individual taxpayer, donated $100,000 in cash to a qualified public charity in Year 1. Her adjusted gross income was $150,000 in Year 1 and $150,000 in Year 2. She makes no donations to charity in Year 2. How much of a tax deduction will she be allowed for this gift in each of the two tax years? A)Year 1: $90,000; Year 2: $10,000 B)Year 1: $45,000; Year 2: $35,000 C)Year 1: $45,000; Year 2: $0 D)Year 1: $75,000; Year 2: $5,000

A) Year 1: $90,000; Year 2: $10,000 Individual cash donations to qualified public charities are limited to 60% of adjusted gross income, but excess amounts may be carried over in subsequent tax years. Sixty percent of Maxine's Year 1 AGI was $90,000. In Year 1, she can deduct $90,000 of the gift. In Year 2, she can deduct the remainder of $10,000.

Full imputed interest on a below-market loan (with the IRS providing accepted loan rates) will be paid with A)a gift loan of amounts more than $100,000. B)a gift loan of amounts more than $10,000. C)a gift loan of amounts more than $5,000. D)any gift loan given.

A) a gift loan of amounts more than $100,000. In a gift loan, the amount of the imputed interest constitutes a gift from the lender to the borrower. For gift loans greater than $10,000 and less than or equal to $100,000, no interest is imputed if the borrower's investment income for the year does not exceed $1,000. For a gift loan of more than $100,000, the prevailing federal rate of interest will be imputed.

The partner's tax basis in his interest in a partnership A) is increased by his share of income reported by the partnership. B) is decreased if additional capital is contributed. C) remains unchanged unless additional capital is contributed or distributions are made. D) remains unchanged until the interest is sold or otherwise disposed.

A) is increased by his share of income reported by the partnership. A partner's tax basis in a partnership interest is affected by items of income or loss, which are passed through to the partner on a proportionate basis. Items of income, as well as capital contributions, increase basis; deductions, as well as distributions, decrease basis. LO 7.1.1

All of the following are exclusion items for the purposes of calculating alternative minimum taxable income (AMTI) except A)the bargain element on exercised incentive stock options. B)the standard deduction amount. C)taxes taken as itemized deductions. D)miscellaneous itemized deductions.

A) the bargain element on exercised incentive stock options. The bargain element on exercised incentive stock options is a preference (deferral) item for AMT purposes and is not an exclusion item for calculating AMTI.

Jimmy will have an adjusted gross income of $275,000 for the current tax year. He would like to reduce his tax liability. Which of the following investments may reduce Jimmy's tax liability? A)An investment in an oil and gas working interest B)An investment in active participation rental real estate C)An investment in a publicly traded limited partnership generating losses D)An investment in a nonpublicly traded limited partnership generating losses

A)An investment in an oil and gas working interest

Which of the following dispositions of Section 1245 property would result in the immediate recapture of some or all of the previous depreciation deduction taken by the taxpayer? A)Installment sale B)Disposition in a tax-free transaction C)Disposition by gift D)Distribution by bequest PREV

A)Installment sale Section 1245 property includes all depreciable tangible personal property used in a trade or business. Section 1245 of the Tax Code requires any recognized gain to be treated as ordinary income to the extent of depreciation taken on the property and occurs when there is a taxable event regarding such property. An installment sale is an example of a triggering taxable event for purposes of Section 1245 recapture. LO 6.1.4

Carol, age 50, received a salary of $35,000 this year. In addition, she received a gift of $1,000 from her brother. She also made a contribution of $3,500 to her traditional IRA. She files as single, and in addition to her itemized deductions of $4,500, she had unreimbursed medical expenses from major surgery on her knees of $7,600. Which of the following best defines Carol's taxable income?

Adjusted gross income less the greater of the standard deduction or the amount of itemized deductions

Ken Brandt (a single taxpayer), age 28, holds the following securities: StockPurchase DateFair Market ValueCost BasisABC (300 shares)Oct. 3, 2003$12,200$5,500DEF (500 shares)Feb. 15, 2021$22,600$15,600GHI (100 shares)June 2, 2006$4,350$6,250LMN (700 shares)Dec. 9, 2020$19,360$28,560XYZ small-cap fund (500 shares)Oct. 20, 2011$1,200$3,700VWL (750 shares, §1244)July 17, 2005$4,050$115,600 BondsPurchase DateFair Market ValueCost BasisEE savings bondsMay 1, 2014$8,500$6,000 Assume Ken incurs $7,500 of college expenses in the current year. If Ken redeems the EE bonds to pay these expenses, what are the tax consequences?

All accrued interest is taxable in the current year The exclusion for interest on EE bonds redeemed to pay for qualifying higher-education expenses applies only to bonds purchased by an individual age 24 or older. Ken is 28 years old; the bonds were purchased approximately seven years ago, when Ken was approximately age 21. LO 2.4.1

Which of these statements regarding capital gains is CORRECT? A)A maximum rate of 28% applies to long-term gain on collectibles. B)Net long-term capital gains are subject to a 0% tax rate if a single filing taxpayer's taxable income is below $40,400 (2021). C)Net short-term gains are subject to a taxpayer's ordinary income tax rate. D)All of the above.

All of the above Net long-term capital gains (and qualified dividends) are subject to a 0% tax rate if the taxpayer's taxable income is below $40,400 (for a single taxpayer, in 2021). The breakpoints will be provided on the exam. Net short-term gains are subject to a taxpayer's ordinary income tax rate. A maximum rate of 28% applies to long-term gain on collectibles.

Dan and his spouse, Gina, were divorced almost four years ago. Under the terms of the decree, Dan must pay Gina 10% of his net business income each year for five consecutive years. Dan made alimony payments of $60,000 in his first post-separation year, $35,000 in the second year, and $5,000 in the third year. What is the tax result of the alimony payments made to Gina? A) None of the payments are deductible because the excess front-loading rules are violated. B) $47,500 of the payments must be recaptured due to the front-loading rules. C) All of the alimony payments are deductible because the front-loading rules do not apply to amounts tied to business profits. D) $15,000 of the payments is deductible because of the limit imposed by the front-loading rules.

All of the alimony payments are deductible because the front-loading rules do not apply to amounts tied to business profits. Payments that fluctuate because of a continuing liability to pay a fixed portion of income from business, property, or services are not subject to the excess front-loading rules.

Eight years ago, Joan Allen, a married taxpayer filing jointly, purchased U.S. Series EE savings bonds for $6,000. She titled the bonds jointly with her husband, Hank. During the current year, when Joan was 35 years old, they redeemed the bonds to help pay for Joan's graduate school tuition. The accrued value at the time of redemption was $8,000. Their AGI for 2021 is estimated to be $100,000. Assume Joan incurs $8,000 of tuition expenses during the year. What are the tax consequences upon the redemption of the bonds? A) All accrued interest is taxable in the current year. B) The interest is taxable at both state and federal levels. C) All the interest may be excluded. D) A portion of the interest may be excluded.

All the interest may be excluded. The EE bond exclusion (for educational purposes) is phased out (for married couples filing jointly) between $124,800 and $154,800 of AGI in 2021. There is no exclusion available when AGI exceeds $154,800. It is not necessary to memorize the exact phaseout amounts because they will be provided on the exam. To qualify for the exclusion, the bonds must be purchased by an individual age 24 or older and held in that person's name, or jointly with a spouse. EE bonds are not taxable at the state level.

Which one of the following summarizes code sections in layman's terms, but have only short-term value? A)Announcements B)Technical Advice Memoranda C)Private Letter Rulings D)Notices

Announcements Announcements often summarize code sections in layman's terms or notify taxpayers of impending deadlines; they have only short-term value. Technical Advice Memoranda normally take place during an audit or during the appeals process of the audit; they give both the taxpayer and the revenue agent an opportunity to resolve a dispute over a technical question. Private Letter Rulings are taxpayer guidance from the IRS that apply only to the particular taxpayer(s) asking for the ruling; they are not applicable to all taxpayers. A Notice is a public pronouncement that contains official guidance about regulations or interpretations of the Code.

Which one of the following often summarize code sections in layman's terms or notify taxpayers of impending deadlines? A)Notices B)Private Letter Rulings C)Technical Advice Memoranda D)Announcements

Announcements Announcements often summarize code sections in layman's terms or notify taxpayers of impending deadlines; they have only short-term value. Technical Advice Memoranda normally take place during an audit or during the appeals process of the audit; they give both the taxpayer and the revenue agent an opportunity to resolve a dispute over a technical question. Private Letter Rulings are taxpayer guidance from the IRS that apply only to the particular taxpayer(s) asking for the ruling; they are not applicable to all taxpayers. A Notice is a public pronouncement that contains official guidance about regulations or interpretations of the Code. LO 8.1.1

Ethel had the following from securities transactions during the current year: Long-term capital gain: $6,400 Long-term capital loss: $2,200 Short-term capital gain: $2,300 Short-term capital loss: $5,500 Which of the following describes the net capital gain or loss reportable by Ethel for the current tax year? A) $4,200 net long-term capital gain; $3,200 net short-term capital loss B) $1,000 net long-term capital gain C) $1,000 net long-term capital loss D) $900 net long-term capital gain; $100 net short-term capital loss

B) $1,000 net long-term capital gain Long-term transactions are netted together, as are short-term transactions. The net long-term capital gain is $4,200 ($6,400 - $2,200). The net short-term capital loss is $3,200 ($2,300 - $5,500). The net short-term capital loss is netted with the net long-term capital gain ($4,200 - $3,200) to result in a net long-term capital gain of $1,000. LO

Skip sold an automobile for $10,000 during the current tax year. The automobile had been used exclusively for business purposes. The cost basis was $18,000, which had been fully recovered through straight-line cost recovery deductions. The automobile was sold on an installment agreement, with a down payment of $1,000 and $2,000 principal payments beginning in the current year. What amount of gain must be recognized in the current year and next year, respectively? A)$1,000 and $2,000 B)$10,000 and $0 C)$3,000 and $2,000 D)$0 and $2,000

B) $10,000 and $0 In an installment sale, any cost recovery recapture, $10,000 in this situation, is recognized in the year of disposition. All gain in this situation is cost recovery recapture. Remember that with Section 1245 property (predominantly personalty), it does not matter whether straight-line, Section 179, or accelerated depreciation was used. All depreciation is subject to recapture as ordinary income. In this scenario, all future payments received are essentially tax free because all $10,000 of gain has been recognized in the year of disposition.

During the current tax year, Robin has a short-term capital loss of $13,000 from the sale of mutual funds. She also has a long-term capital gain of $6,200 from the sale of an antique clock and has unrecaptured Section 1250 income of $17,000. Robin is in the 35% marginal income tax bracket. What is the tax result from these transactions? A)$4,000 unrecaptured Section 1250 income taxed at 25% and $6,200 collectibles gain taxed at 28% B)$10,200 unrecaptured Section 1250 income taxed at 25% C)$14,200 of unrecaptured section 1250 income taxed at 25% D)$6,800 short-term capital loss carryforward and $17,000 unrecaptured Section 1250 income taxed at 25%

B) $10,200 unrecaptured Section 1250 income taxed at 25% The short-term capital loss is first used to offset the collectibles gain of $6,200. This leaves a short-term capital loss of $6,800. This is next used to offset the 25% gain. The $17,000 is offset by the $6,800 remaining capital loss. This leaves $10,200 of unrecaptured Section 1250 income, taxed at a maximum 25% rate. This is the most favorable manner of offsetting the capital losses against the capital gains.

Ron, age 43, and Sandy, age 41, are married with two children, Michael, age 12, and Victoria, age 8, who has been blind since her birth. Ron is an architect and general partner with XYZ partnership. Sandy is self-employed as an attorney and works out of a home office. Her home office is exclusively and regularly used for business, and the home office is her principal place of business. Their information for the tax year 2021 is as follows: Adjusted gross income: $217,300 Itemized deductions (including qualified residential mortgage interest, taxes paid, and charitable contributions): $33,000 Early in the current year, Sandy's father died. Sandy is the sole beneficiary of her father's entire estate. The estate is presently in the probate process. Sandy's mother, Lisa, age 68, has moved in with them but provides her own support. She was married to Sandy's father when he died earlier this year. This is Ron's second marriage. He makes monthly support payments to his former wife and his daughter. Because both Ron and Sandy are considered to be self-employed, they make quarterly estimated tax payments each year to cover both their income tax and self-employment tax obligations. Ron's investment in his XYZ partnership interest on December 31, 2019, was $12,000. As the result of a serious downturn in business, the partnership calculates Ron's share of the partnership's losses for 2021 will be $14,500. How much of this projected 2021 loss, if any, may Ron deduct on his income tax return for 2021? A)$0 B)$12,000 C)$2,500 D)$14,000

B) $12,000 Because this is not a passive activity, he may deduct (under the at-risk limitations) his share of the partnership loss to the extent of his investment in the partnership assuming he has at least that much in other income to offset the loss ($12,000).

Jim has the following items from four investments: Passive income from a publicly traded limited partnership$16,000Passive loss from a publicly traded limited partnership$(8,500)Passive income from a nonpublicly traded limited partnership$13,000Passive loss from a nonpublicly traded limited partnership$(20,000) What is the total amount of passive losses that may be deducted during the current year? A)$28,500 B)$13,000 C)$29,000 D)$20,000

B) $13,000 The passive loss from a nonpublicly traded limited partnership, $20,000, is deductible, but only up to the passive income from the same partnership; thus, only $13,000 of the $20,000 is currently deductible.

On April 1 of the current tax year, Elisha sold her principal residence for a total price of $501,000: $301,000 was in cash, with the buyer assuming a $200,000 mortgage on the house. Elisha purchased the house 15 years ago for $290,000, but she has an adjusted basis of $80,000. She has not made any improvements to the house. To assist in the sale of the residence, she incurred costs of $1,500 for repairs three weeks before the sale occurred. Realtor commissions of $31,000 resulted from the sale. On May 1 of the current tax year, Elisha bought a new residence for $260,000. What amount, if any, must be recognized on the sale of Elisha's residence? A)$388,500 B)$140,000 C)$390,000 D)$0

B) $140,000 The answer is computed as follows. Gain realized: Sale price$501,000Selling expenses(31,000)Total amount realized$470,000Less adjusted basis(80,000)Gain realized$390,000 Note that the repairs do not impact the basis in the residence. Also, the mortgage assumption by the buyer is part of the sales price. Gain recognized (amount subject to income tax): Gain realized$390,000Less exclusion250,000Gain recognized$140,000 LO 6.1.2

Your client has a salary of $80,000, dividends of $20,000, and limited partnership income of $15,000. This year, she invested in an equipment-leasing partnership. Her initial investment included $50,000 cash and a nonrecourse note for $60,000. What is the maximum tax deduction your client may take from this equipment-leasing investment this year? A) $50,000 B) $15,000 C) $45,000 D) $35,000

B) $15,000 Passive losses are only deductible against passive income. She has passive income of $15,000 from the limited partnership, thus she could deduct up to $15,000 of passive losses from the equipment-leasing limited partnership. LO

Patty has a $10,000 passive loss carryforward from Beta limited partnership, which is publicly traded. She also has a $15,000 passive loss carryforward from Alpha limited partnership, which is nonpublicly traded. In the current year, she has $6,000 of income from Beta. She also has $11,000 of income from Gamma LP. Gamma is not publicly traded. What is the total amount of passive losses that Patty may deduct during the current year? A) $6,000 B) $17,000 C) $11,000 D) $25,000

B) $17,000 Of the $10,000 passive loss carryforward from the Beta limited partnership, only $6,000 may be utilized in the current year due to the $6,000 of current year passive income. A total of $11,000 in losses from the Alpha limited partnership may be utilized against the $11,000 of income from the Gamma limited partnership in the current year because both are nonpublicly traded. Thus, the total of passive losses that are allowed for the current year is $17,000.

Irwin, a young financial planner, operates his practice as an LLC. His LLC net income is $100,000, and the taxable income on his single return is $140,000. The LLC paid no wages, and has no depreciable property. What is the amount of Irwin's qualified business income (QBI) deduction? A) $0, because he has no depreciable property B) $20,000 C) $28,000 D) $0, because he paid no wages

B) $20,000 For a specified service business (SSB), if the taxpayer's taxable income is below the phaseout range of $164,900 to $214,900 (single 2021), the QBI deduction is fully available. An SSB involves performance of services in the fields of health, law, accounting, actuarial science, performing arts, veterinary services, consulting, athletics, financial services, or brokerage services. For pass-through entities, the owner/taxpayer may be allowed a deduction equal to 20% of the lesser of the QBI, or 20% of the taxpayer's taxable income (reduced by net capital gain). Thus, the available deduction is 20% of the QBI of $100,000, or $20,000.

Willis has an active participation rental real estate activity. Last year, he had losses of $15,000 from the active participation real estate, and his AGI was $225,000. The $15,000 of losses were suspended due to his AGI. In the current year, Willis has an AGI of $90,000 and $6,000 of current losses from his real estate rental activity. What amount of loss may Willis deduct in the current year? A)$25,000 B)$21,000 C)$6,000 D)$15,000

B) $21,000 The suspended losses of $15,000 from a former year plus the current losses of $6,000 equals $21,000

Bill's 2020 income tax return, which was for a full year, showed an AGI of $140,000 and an income tax liability of $32,100. He estimates his 2021 income tax to be $38,000 and his total wage withholding to be $5,000. What minimum amount of estimated tax payments must Bill pay (in equal quarterly installments) for 2021? A) $41,800 B) $27,100 C) $34,200 D) $32,100

B) $27,100 The safe harbor for avoiding the underpayment penalty is the lesser of 90% of the current year tax liability or 100% of the prior year tax liability (110% if the prior year's AGI was over $150,000). 100% of the prior year tax liability is $32,100. 90% of the current year tax liability of $38,000 is $34,200. The smaller of these numbers, reduced by the $5,000 withholding, equals $27,100.

Matthew Brady, age 47, purchased a deferred annuity in January 1982 for $50,000. In the current year, when the surrender value was $125,000, Matthew took a nonperiodic distribution of $75,000. Which one of the following statements correctly describes the income tax consequences of the distribution? A) $75,000 is tax free. B) $50,000 is tax free, $25,000 is taxable. C) $50,000 is taxable, $25,000 is tax free. D) $75,000 is taxable income.

B) $50,000 is tax free, $25,000 is taxable. The pre-August 14, 1982, annuity retains first-in, first-out (FIFO) treatment. Thus, the basis of $50,000 is treated as being withdrawn first and is tax free. The remaining $25,000 is taxable. If this were a post-August 13, 1982, contract, it would be treated on a last-in, first-out (LIFO) basis.

Which of the following legally married couples may file a joint (MFJ) income tax return? David and Jack whose tax years began on the same day Marvin and Alicia who are legally separated on the last day of the calendar year Janice and Juan; Juan was a nonresident alien for only the first three months of the year Sylvia and Marilyn who married on New Year's Eve this year A)I and II B)I and IV C)III and IV D)II and IV

B) I and IV Only couple I and IV may file a joint income tax return. Marvin and Alicia were legally separated on the last day of the year and are ineligible. Because Juan was a nonresident alien for part of the year, Janice and Juan are also ineligible to use the MFJ filing status.

Jim, a single taxpayer, sells Section 1244 stock for a $250,000 loss during the tax year. The stock had been held for two years. Which one of the following is CORRECT regarding the treatment of the loss? A) $100,000 is treated as an ordinary loss; $150,000 is treated as long-term capital loss. B) $50,000 is treated as an ordinary loss; $200,000 is treated as long-term capital loss. C) $50,000 is treated as an ordinary loss; $200,000 is treated as a Section 1244 carryforward. D) $250,000 of the loss is fully deductible as a capital loss.

B) $50,000 is treated as an ordinary loss; $200,000 is treated as long-term capital loss. Loss on the sale of Section 1244 stock is deductible as an ordinary loss up to $100,000 per year on a jointly filed return, or $50,000 on any other return (which applies to Jim as a single taxpayer). Any Section 1244 loss in excess of the annual limits is treated as a capital loss—long term or short term, depending on the holding period. The $100,000/$150,000 option would be correct if Jim were a married taxpayer filing jointly.

Susan received 100 shares of stock as a gift from her uncle, Carl. Carl purchased the stock 15 years ago for $12 per share. Susan received the stock from Carl two months ago, when the fair market value of the stock was $15 per share, and she sold the stock this week for $19 per share. What is the amount and character of Susan's gain from the sale of the stock? A) $400 short-term capital gain B) $700 long-term capital gain C) $400 long-term capital gain D) $700 short-term capital gain

B) $700 long-term capital gain In the case of an asset received as a gift, where the fair market value on the date of the gift is greater than the donor's adjusted basis, the recipient has a carryover basis. In this case, Uncle Carl had purchased the stock for $12 per share and had gifted it to Susan when the fair market value was $15 per share. Susan subsequently sold the stock for $19 per share. Thus, the carryover basis from Uncle Carl would be $12 per share. In a situation where the recipient of the gift takes the donor's basis, the holding period is tacked. In other words, the donor's holding period is added to the donee's holding period. Thus, Susan is treated as holding the stock for over 15 years.

Amanda's share of general partnership net income is $60,000 in 2021. In addition, her distributive share of S corporation income is $10,000. What is Amanda's self-employment tax, if any, for 2021? Round your answer to the nearest dollar. A) $9,891 B) $8,478 C) $10,710 D) $0

B) $8,478 The distributive share of income from an S corporation is not subject to the self-employment tax. Actual earnings$60,000Less: 7.65%- (4,590)Net earnings from self-employment$55,410Multiply by× 15.3%Self-employment tax$8,478 LO 5.3.1

Kathy, age 70, is single, an employee of Expo Corporation, and lives in the state of Alabama. Her only sources of income this year are $80,000 of W-2 wages, $6,000 in capital gains, and $1,000 in interest on State of Alabama bonds. Based on this information, Kathy's adjusted gross income (AGI) for the current year is

B) $86,000. Kathy's AGI is all income from any source derived except for those items specifically excluded by the Tax Code. The W-2 wages and capital gains total $86,000. The municipal bond interest is excluded by law. Kathy's AGI is as follows: W-2 income$80,000Interest on State of Alabama bonds (tax exempt)0Capital gains6,000AGI$86,000

Assume that for each of the next five years, your client, Dan, will have the following: Active income: $100,000 Investment income: $50,000 Passive income: $0 Passive loss (from nonpublicly traded partnership): $25,000 Dan is considering an investment in a nonpublicly traded partnership that requires a $100,000 initial investment and will generate cash flow (pretax passive income) of $12,500 per year at the end of each year for the next five years. Upon liquidation at the end of the fifth year, Dan will receive a total of $100,000 after taxes. Dan's after-tax rate of return from other investments is 10% and his combined federal and state marginal tax bracket is 28%. What is Dan's net present value on the investment?

B) $9,477 This problem requires the calculation of the net present value of the cash flows as compared to the $100,000 initial cash outlay. The net present value of the $12,500 received at the end of each of the first four years is first calculated. The present value of these four payments is $39,623. The calculation of present value of the amount received at the end of year five of $112,500 ($12,500 + $100,000) using a 10% rate yields $69,854. The $69,854 combined with the $39,623 results in a combined present value of all payments of $109,477. Subtracting the initial cash outlay of $100,000 results in a net present value of $9,477. Note that no tax liability would result from the cash flows as they would be "sheltered" by the passive losses. LO 7.1.1

Sheila, a single taxpayer, has taxable income of $470,000. Included in the taxable income is $50,000 of qualified dividends. At what rate(s) will her qualified dividends be taxed? A) 15% only B) 15% and 20% C) 20% only D) 25%

B) 15% and 20% The qualified dividends straddle the $445,850 breakpoint (for 2021). Thus, a portion fall into the $40,401 to $445,850 range and are taxed at 15%. The dividends above the $445,850 breakpoint are taxed at 20%.

For a taxpayer with an AGI in excess of $150,000 for the prior tax year ($75,000 if married filing separately), the estimated tax penalty safe harbor is A) 120% of the prior year's tax liability or 80% of the current year's tax liability. B) 90% of the current year's tax liability or 110% of the prior year's tax liability. C) 90% of the prior year's tax liability or 110% of the current year's tax liability. Explanation

B) 90% of the current year's tax liability or 110% of the prior year's tax liability.

Your client, Elaine Dell, is near the highest tax bracket and is contemplating several investments. She is, however, concerned about minimization of her federal income tax liability on the income from the investment. Which of the following investments would produce income that would be taxed at the lowest potential tax rate? A) A certificate of deposit B) A utility stock with a high dividend yield C) A corporate bond fund D) A zero coupon bond

B) A utility stock with a high dividend yield Qualified dividends are generally taxed at a 15% rate (or 20% for taxpayers with higher income levels). All of the other options produce interest income, which is taxable as ordinary income, at the marginal rate of the taxpayer.

Personal expenses deductible from adjusted gross income most accurately describes which one of the following

B) Itemized deductions Itemized deductions are generally personal expenses (e.g., home mortgage interest, medical expenses) that are specifically allowed as a deduction from AGI. Schedule C (sole proprietorship) expenses and adjustments to income are both deductions for AGI, or above-the-line deductions

Which of the following statements regarding installment payments pursuant to a divorce is CORRECT? It is usually in the best interest of the payor to spread the amount of payments over as many years as possible to take advantage of the time value of money. It is usually in the payee's best interest to immediately receive as much money as possible rather than receiving payments over a longer period. A) I only B) Both I and II C) II only D) Neither I nor II

B) Both I and II A common issue that arises in the settlement of divorce proceedings is how payments are best structured in resolution of marital obligations. As a general rule, it is usually in the best interest of the payor to spread the amount of payments over as many years as possible to take advantage of the time value of money. Alternately, the payee's best interests are served by immediately receiving as much money as possible rather than receiving payments over a longer period.

Which of the following statements regarding the kiddie tax is CORRECT? The kiddie tax provision limits income shifting by preventing families from transferring large amounts of unearned income to children and making the shift effective for income tax purposes. If a child under the age of 19 has unearned income above a specified amount, the excess is taxed at the parents' marginal tax rates for the year, rather than at the child's marginal rate. A) II only B) Both I and II C) Neither I nor II D) I only

B) Both I and II Both statements are correct. The kiddie tax applies to unearned income for children under the age of 19 and full-time students until they reach age 24. LO 3.2.1

Harriet builds remote-control cars as a hobby. Which hobby expenses must she deduct from hobby income first? A) Cost recovery deductions B) Cost of goods sold C) Operating expenses D) None are deductible

B) Cost of goods sold Starting in 2018, due to the passage of the Tax Cuts and Jobs Act (TCJA), most hobby expenses are no longer deductible with the notable exception of cost of goods sold.

When spouses who are legally married use the married filing jointly filing status, what is their liability for the combined income tax owed? A) Each spouse owes 50% of the tax due. B) Each spouse has joint and several liability for payment of the entire tax. C) A nonworking spouse has no liability for the tax owed. D) Each spouse has liability for the tax due in proportion to their earnings reported on the return.

B) Each spouse has joint and several liability for payment of the entire tax.

Which one of the following is a tax return preparer failure that would initiate a tax penalty from the IRS? A) Failure to keep a copy of all returns prepared for at least the last 10 years B) Failure to sign a return as preparer and give their tax identification number on the return C) Failure to provide a taxpayer with a receipt for their services D) Failure to maintain a list of all returns prepared for the past five years

B) Failure to sign a return as preparer and give their tax identification number on the return The following actions would constitute a tax return preparer's failure subject to a tax penalty: failure to provide a taxpayer with a copy of their return, failure to keep a copy of all returns prepared for at least the last three years or to maintain a list of returns prepared, and failure to sign a return as preparer and give their tax identification number on the return.

Which form should a nonresident alien file, if they earned income during the taxable year and if not electing to be a resident? A)Form 1041 B)Form 1040NR C)Form 1040 D)Form 706

B) Form 1040NR If they earned U.S. income during the year, nonresident aliens must file Form 1040NR. However, if married to a U.S. citizen or resident alien, the nonresident alien can elect to be treated as a resident alien for tax purposes only. Tax status does not necessarily reflect immigration status. If this election is made, the couple must pay U.S. taxes on their worldwide income. In this situation, the nonresident alien spouse should obtain an Individual Tax Identification Number (ITIN). Depending on their individual situation and intentions, they may apply for a Social Security number with the Social Security Administration.

Adrian Brown owned 500 shares of XYZ growth and income fund. She has become increasingly dissatisfied with the performance of the fund and, upon the advice of a friend, decided to execute a "telephone transfer" and switch the balance in the fund to the XYZ intermediate bond fund. Which one of the following describes the tax effect of such a strategy? A) No gain or loss will be recognized by the taxpayer, and the basis in the new fund will be the same as that of the old fund. B) Gain or loss will be recognized by the taxpayer on the redemption of the old fund. C) Any loss will be recognized by the taxpayer, but any gain will be deferred through a reduction in the basis of the new fund. D) No gain or loss will be recognized by the taxpayer, but the basis of the new fund will be reduced by any deferred gain or increased by any unrecognized loss.

B) Gain or loss will be recognized by the taxpayer on the redemption of the old fund. A telephone transfer is the same as a sale or other taxable redemption of the fund. Therefore, gain or loss will be recognized based on the difference in the redemption proceeds and the basis in the shares redeemed. This is true even if the transfer is made between two funds in the same fund family.

Jamaal owns a professional service corporation. While capital is not a material income-producing factor, he does use certain equipment in his practice. Jamaal would like to assist his 18-year-old son, Jarod, in financing his education at a college located in another state. Which one of the following intrafamily transfer techniques would be most appropriate? A)Convert to S corporation status and give Jarod some of the stock. B)Gift the equipment to Jarod and then lease it back from him. C)Set up a short-term revocable trust for Jarod's benefit. D)List Jarod as an employee and pay him a salary, even though he cannot provide any meaningful services.

B) Gift the equipment to Jarod and then lease it back from him. The answer is gift the equipment to Jarod and then lease it back from him as the most appropriate intrafamily transfer technique.

Max is a widower who provides a home for himself and his dependent six-year-old daughter, Lucy. He has hired an individual to pick his daughter up from school each day, bring her home, cook dinner, and perform some housekeeping services until he gets home four hours later. He pays $1,600 per month for the service. How will this affect Max's income tax return? Max may be entitled to a child and dependent care credit. Max qualifies to list Lucy on his income tax return as a dependent. Because Lucy attends school during the day, the child or dependent care credit is not available. Max must allocate the $1,600 per month between child care and housekeeping services.

B) I and II School attendance does not affect the availability of the credit. The $1,600 in expenses incurred each month to enable Max to work outside the home do not have to be divided between child care and housekeeping services.

Which of the following taxpayers may owe the additional Medicare tax in 2021? Brad and Jane file jointly and have combined wages of $288,000. Terry's only income in 2021 is from his investments and totals $290,000. Jack has earned $150,000 in compensation from his employment at Bland Foods Inc. Lisa, whose filing status is head of household, is self-employed and has self-employment income of $225,000. A) IV only B) I and IV C) I only D) I, II, and III

B) I and IV Statements I and IV are correct. The additional Medicare tax rate is .9%. An individual is liable for the additional Medicare tax if the individual taxpayer's wages, other compensation, or self-employment income (combined with a spouse if filing as married filing jointly) exceeds the thresholds for the taxpayer's filing status of a combined income greater than $200,000 if single and $250,000 if married filing jointly.

Which of the following statements regarding Section 1033 involuntary conversions is CORRECT? For an owner-user, the replacement property must pass the functional use test. The taxpayer use test provides less flexibility than the functional use test. A) Neither I nor II B) I only C) Both I and II D) II only

B) I only Statement II is incorrect. The taxpayer use test provides more flexibility than the functional use test. LO 6.2.3

Which of the following statements is CORRECT regarding a minority non-employee shareholder in an S corporation? Reports income when the corporation has net income for a tax year Votes for the board of directors at the annual shareholders meeting Receives a K-1 annually to prepare a personal income tax return Reports on a personal income tax return a pro rata share of corporate profit or loss A) I and III B) I, II, III, and IV C) I, II, and III D) II and IV

B) I, II, III, and IV All of the statements are correct. The fact that the shareholder has a minority interest in the corporation has no bearing on the correct answers. The flow-through of a proportionate share of net income is reported on the K-1. The shareholder is allowed to vote for the board.

Which of the following statements correctly identify the requirements necessary to deduct $25,000 of losses from an active participation real estate program? The taxpayer must have at least a 10% ownership interest in the property. The taxpayer's modified adjusted gross income must not exceed $200,000. The taxpayer must make the major management decisions related to the property. The taxpayer's interest in the property may not be held as a limited partnership interest. A)II and IV B)I, III, and IV C)II, III, and IV D)I and III

B) I, III, and IV The answer is I, III, and IV. II is false because the phaseout range for the active participation real estate deduction is $100,000 to $150,000. LO 7.2.1

Which of the following itemized deductions would be adjustments to regular taxable income in arriving at alternative minimum taxable income (AMTI)? Casualty losses State income taxes paid Standard deduction Charitable donation made to the local university A) II, III, and IV B) II and III C) I and IV D) I and III

B) II and III Statement I is incorrect because casualty losses are deductible for both regular tax and AMT; no adjustment is necessary. Statement II is correct because state taxes are not deductible for AMT purposes. Statement III is correct because the standard deduction is a positive adjustment when calculating AMT income. Statement IV is incorrect because charitable contributions are deductible for both regular income tax and AMT; no adjustment is necessary.

Chris has been actively buying and selling science fiction memorabilia for many years. He has a booth at several conventions he attends each year. His tax professional has been tracking his profit and loss and reports that Chris made a profit in four of the last five years. Which of the following statements regarding Chris's profit and loss is CORRECT? The IRS will presume that Chris's activity is a hobby. If the hobby rules do not apply, Chris can use Schedule C to report the income and expenses from this activity. A)Both I and II B)II only C)I only D)Neither I nor II

B) II only Statement I is incorrect. An activity is presumed not to be a hobby if there are profits from the activity in any three of five consecutive tax years ending with the tax year in question (unless the IRS proves otherwise). Statement II is correct; if the hobby rules do not apply, Chris can report his income and expenses on Schedule C.

Alisha, a CFP® certificant and fee-only financial planner, has assisted Roger, a self-employed physician, in income tax return preparation and investment planning during the year. On which of the following schedules may Alisha's fee be deductible by Roger on his federal income tax return? Schedule A—itemized deductions Schedule C—profit or loss from business Schedule D—capital gains and losses A) I only B) II only C) I and II D) I, II, and III

B) II only The investment planning fees are not deductible (the Tax Cuts and Jobs Act, or TCJA, repealed the Tier II miscellaneous itemized deductions). This expense may not be added to the basis of securities. The tax preparation fee is partly nondeductible (the portion attributable to the individual return), and the portion of the fee for completing the Schedule C is a business expense that would be deducted on the Schedule C.

Your clients, Jane and Mark, are contemplating the purchase of a condominium to use as a rental property. They would manage the property themselves and anticipate that it would generate losses for the first few years, at least. Which of the following statements are CORRECT with respect to active participation rental real estate? The interest may be held through a limited partnership. A deduction-equivalent tax credit of up to $25,000 is available. The taxpayer must hold a 10% or greater ownership interest in the property. The taxpayer must participate in the management of the property in a bona fide sense.

B) III and IV III and IV are the only choices that are correct, by definition. The interest in the property may not, by definition, be held through a limited partnership, and up to $25,000 refers to losses that may be deducted, not credits that may be taken. The deduction-equivalent tax credits mentioned in option II are relevant with respect to low-income housing and historic rehabilitation passive activities. LO 7.2.3

Which one of the following best describes the role of a special allocation in a limited partnership? A)It allocates management responsibility to the general partners. B)It allows an allocation of items of income and expense that is not pro rata. C)It establishes the standards for allocating the proceeds of non-routine or "special" items of income. D)It requires all items to be distributed pro rata based on a partner's capital account balance.

B) It allows an allocation of items of income and expense that is not pro rata. A special allocation allows an allocation of items in a manner that differs from the "normal" pro rata allocation of deduction, income, credit, etc. LO 7.1.1

Jim is a single taxpayer. During the current year, he sold Section 1244 stock for $40,000. Jim had held the stock for three years. His basis in the stock was $130,000. What is the tax result from the sale of the stock? A)Jim has a long-term capital loss of $100,000. B)Jim has an ordinary loss of $50,000 and a long-term capital loss of $40,000. C)Jim has an ordinary loss of $50,000 this year and a carryforward of ordinary loss of $40,000. D)Jim has an ordinary loss of $100,000.

B) Jim has an ordinary loss of $50,000 and a long-term capital loss of $40,000. The loss on the sale, exchange, or worthlessness of Section 1244 stock may be treated as an ordinary loss up to $100,000 annually on a joint return or $50,000 annually on any other return. Thus, as a single taxpayer, Jim has a $50,000 ordinary loss. Any excess loss is a capital loss—short- or long-term depending on the holding period.

In 1991, John Idler purchased a single premium whole life insurance policy. In the current year his medical expenses are $15,000 and his AGI is $75,000. What is the tax implication to John if he borrows the interest from the policy's accumulated cash value to pay his current year's medical expenses? A) John will be required to report the amount borrowed as income, but he will not be allowed a medical expense deduction. B) John will be required to report the amount borrowed as income and will be allowed a medical expense deduction. C) John will not be required to report the amount borrowed as income and will not be allowed a medical expense deduction. D) John will not be required to report the amount borrowed as income, but he will be allowed a medical expense deduction.

B) John will be required to report the amount borrowed as income and will be allowed a medical expense deduction. Amounts borrowed on a single premium whole life policy issued on or after June 21, 1988 (a MEC), are taxable on a last-in, first-out basis; thus, the earnings would be taxable. A medical expense deduction will be allowed regardless of the source of the funds, since the payment would be for a valid medical expense.

Ron, age 43, and Sandy, age 41, are married with two children: Michael, age 12, and Victoria, age 8, who has been blind since her birth. Ron is an architect and general partner with XYZ partnership. Sandy is self-employed as an attorney and works out of a home office. Her home office is exclusively and regularly used for business, and the home office is her principal place of business. Their information for the tax year 2021 is as follows: Adjusted gross income: $217,300 Itemized deductions (including qualified residential mortgage interest, taxes paid, and charitable contributions): $33,000 Early in the current year, Sandy's father died. Sandy is the sole beneficiary of her father's entire estate. The estate is presently in the probate process. Sandy's mother, Lisa, age 68, has moved in with them but provides her own support. She was married to Sandy's father when he died earlier this year. This is Ron's second marriage. He makes monthly support payments to his former spouse and his daughter. Because both Ron and Sandy are considered to be self-employed, they make quarterly estimated tax payments each year to cover both their income tax and self-employment tax obligations. Based on the information provided in the case scenario, which of the following statements regarding Lisa's income tax filing status for 2021 is CORRECT? A) Lisa may file as head of household for 2021. B) Lisa may file married filing jointly for 2021. C) Lisa must file married filing separately for 2021. D) Lisa must file a single return for 2021.

B) Lisa may file married filing jointly for 2021. Because Lisa's spouse died earlier in the year, she may use married filing jointly status for 2021. She cannot use the head of household filing status because she does not maintain a household for a qualifying child or relative.

Which of the following statements is accurate with respect to a like-kind exchange? A) The amount of gain recognized will reduce the taxpayer's basis in the property received. B) No gain will be recognized unless the taxpayer receives boot. C) No gain will be recognized on the exchange of inventory. D) Gain recognized is equal to the gain realized on the exchange plus the boot received.

B) No gain will be recognized unless the taxpayer receives boot. In a like-kind exchange, the gain recognized is always the lesser of the gain realized or the boot received. If there is no boot received, there is no gain recognized. Inventory is not eligible for like-kind exchange treatment—thus, gain would be recognized. The basis in the acquired property is the FMV of the acquired property, reduced by the gain realized but not recognized (the deferred gain).

How is a shareholder's share of an S corporation's income and/or loss reported to the shareholder? A)On the K-1 of Form 1065 B)On the K-1 of Form 1120S C)None of these D)On Schedule C of Form 1040 PREV

B) On the K-1 of Form 1120S

Which one of the following is not a social objective of the federal taxation system?

B) Revenue raising Revenue raising is one of the three main purposes of the federal taxing system; it is not a social objective. Support of charitable organizations, preservation of our nation's historical buildings, and relief for certain child care expenses (the child care credit) are social objectives of the federal taxation system.

Ron, age 43, and Sandy, age 41, are married with two children, Michael, age 12, and Victoria, age 8, who has been blind since her birth. Ron is an architect and general partner with XYZ partnership. Sandy is self-employed as an attorney and works out of a home office. Her home office is exclusively and regularly used for business, and the home office is her principal place of business. Their information for the tax year 2021 is as follows: AGI: $217,300 Itemized deductions (including qualified residential mortgage interest, taxes paid, and charitable contributions): $33,000 Early in the current year, Sandy's father died. Sandy is the sole beneficiary of her father's entire estate. The estate is presently in the probate process. Sandy's mother, Lisa, age 68, has moved in with them but provides her own support. She was married to Sandy's father when he died earlier this year. This is Ron's second marriage. He makes monthly support payments to his former spouse and his daughter. Because both Ron and Sandy are considered to be self-employed, they make quarterly estimated tax payments each year to cover both their income tax and self-employment tax obligations. Based on the information provided in the case scenario for Ron and Sandy, which of the following statements regarding Sandy's home office deduction is CORRECT? A) The net income from the business use of Sandy's home must equal or exceed the business expenses (including depreciation). B) Sandy may deduct her qualifying home office expenses when calculating AGI. C) The amount Sandy may deduct this year for qualifying home office expenses is unlimited. D) Sandy is not eligible to deduct her home office expenses.

B) Sandy may deduct her qualifying home office expenses when calculating AGI. Because Sandy is self-employed, she can deduct her qualifying home office expenses when calculating AGI (as a line item on Schedule C). The amount of her home office expense deduction this year is limited to the net income from her business. A profit from the business is not one of the tests for qualification of a home office for tax deduction purposes. Rather, it is one of the limitations on the deductible amount after it qualifies. LO 5.4.1

John has had a very good year and has over $600,000 of taxable income, including a sizable amount of capital gains. He's thinking of selling a large block of stock to a neighbor at a price significantly below market value solely to recognize the loss. If a court disallows the loss on the sale of the stock because the sale was not bona fide and was made for the sole purpose of realizing a loss, which doctrine is being applied? A) Step transaction doctrine B) Sham transaction doctrine C) Assignment of income doctrine D) Clear reflection of income doctrine

B) Sham transaction doctrine A sale that is not bona fide and made for the sole purpose of realizing a loss from the transaction would be considered a sham transaction.

Which of the following forms of business would file a Schedule C to report the income or loss from the business? A) C corporation B) Sole proprietorship C) S corporation D) Limited partnership

B) Sole proprietorship The sole proprietorship is the business that files a Schedule C with the individual's Form 1040. The limited partnership files a Form 1065, and the C corporation files a Form 1120. The S corporation files an 1120-S. Note that a single-member LLC would also file a Schedule C, as it would be treated as a disregarded entity for income tax purposes.

Blake, a sole proprietor, is selling several business assets. He has been told by a friend that the items he is selling are not capital assets and are subject to the ordinary income tax rate. You are his financial planner and tell him that the gains on Section 1231 assets can be treated as capital gains for income tax purposes subject to certain rules. Which of the assets Blake sold are Section 1231 assets? A) A copyright on the theme song Blake's company uses in its advertising that Blake wrote B) The building and land sold when Blake's business moved to a new location C) Blake's inventory of electric guitars his business manufactures D) Accounts receivable

B) The building and land sold when Blake's business moved to a new location The building and land sold when Blake's business moved to a new location qualify under Section 1231 as depreciable personal or real property used in business or for the production of income. The building portion of the property was depreciable property. While they are not considered capital assets, under Section 1231 they are taxed using capital gain rates, subject to the Section 1245 and 1250 rules for depreciation recapture rules. Losses are always ordinary and not subject to the $3,000 ($1,500 for MFS) ordinary loss limitation. Accounts receivable, inventory, and copyrights and other creative works held by the creator are all ordinary assets that would result in ordinary income tax (not capital gain) if sold at a gain. LO 6.1.4

John, an accountant, is considering forming an S corporation for his practice. He will be the sole employee of the corporation. Which of the following statements accurately describes the income tax consequences of the proposed arrangement? A) The corporation would be considered a PSC, subject to graduated tax rates. B) The corporation would not be considered a personal service corporation (PSC), and the income would be subject to John's personal income tax rates. C) The corporation would be considered a PSC, subject to a flat 21% tax rate. D) The corporation would not be considered a PSC; therefore, the income would be subject to the graduated corporate income tax rates.

B) The corporation would not be considered a personal service corporation (PSC), and the income would be subject to John's personal income tax rates. The personal service corporation (PSC) classification for a C corporation was effectively repealed with the Tax Cuts and Jobs Act (TCJA). S corporations do not have corporate income tax rates. As an S corporation, all income would flow through to John to be taxed at his individual rates.

Marion donated a truck to the local food bank to use for picking up food donations. Marion had purchased the truck several years ago for $15,000, and it currently has a value of $3,400. Which of the following statements regarding the documentation Marion must have to support his charitable contribution of the truck is CORRECT? A) An appraisal must be attached to Marion's income tax return for the year of the donation. B) The documentation must have the description of the property, the name of the receiving charitable organization, the date of the contribution, and the amount of the donation. C) A noncash contribution under $5,000 needs no documentation to support the donation. D) A letter from the food bank thanking him for the donation of the truck is sufficient documentation.

B) The documentation must have the description of the property, the name of the receiving charitable organization, the date of the contribution, and the amount of the donation. The donor-taxpayer must have a canceled check, bank record, or a receipt from the donee organization to substantiate the deduction. The documentation must have the amount of cash or description of property, the name of the receiving charitable organization, the date of the contribution, and the amount of the donation. An appraisal is not required for noncash property over $500 and less than or equal to $5,000. However, taxpayers may wish to get an independent appraisal to support the deduction claimed.

Cash value life insurance is often structured like an investment vehicle. However cash value life insurance contains important features that shelter the inside buildup from taxation. Which of the following will NOT be considered when determining whether a policy can maintain its tax favored status? A) The death benefit B) The premium value test C) The cash value accumulation test D) The cash guideline premium test and corridor test

B) The premium value test Without a death benefit, a contract does not meet the legal definition of life insurance. There are currently two tests—only one of which must be met—in order to classify a product as life insurance for federal income tax purposes: (1) the cash value accumulation test and (2) the cash guideline premium test and corridor test. There is no premium value test.

Which one of the following reflects the CORRECT sequence of steps in the tax calculation process

B) Total income minus adjustments to income and standard or itemized deduction(s) equals federal taxable income. Total (gross) income minus adjustments to income equals adjusted gross income (AGI). AGI minus standard or itemized deduction(s) equals federal taxable income.

As part of a divorce decree, a judge has ordered Judy, 44, to give Alex, 36, half of her IRA. This transfer can be best described as A)subject to the 10% early withdrawal penalty. B)a domestic relations order (DRO). C)qualified domestic relief order. D)a qualified domestic relations order (QDRO).

B) a domestic relations order (DRO). This is properly known as a domestic relations order because an IRA is not a qualified plan. If Judy instead were ordered to withdraw from her 401(k), it would become a qualified domestic relations order.

Imputed interest on a below-market loan (with the IRS providing accepted loan rates) will be paid, unless the gift loan is A) between friends. B) less than $10,000 and the gift loan recipient has less than $1,000 in interest income. C) less than $5,000 and the loan recipient has no interest income. D) from a corporation to a shareholder.

B) less than $10,000 and the gift loan recipient has less than $1,000 in interest income. In a gift loan, the amount of the imputed interest constitutes a gift from the lender to the borrower. For gift loans greater than $10,000 and less than or equal to $100,000, no interest is imputed if the borrower's investment income for the year does not exceed $1,000. For a gift loan of more than $100,000, the prevailing federal rate of interest will be imputed.

All of the following statements regarding tax deduction limits on passive activity excess losses are correct except A)excess passive activity losses are the excess of otherwise allowable deductions from the taxpayer's passive activities over the amount of income from the taxpayer's passive activities. B)losses from one nonpublicly traded passive activity may not offset income from another nonpublicly traded passive activity. C)excess passive activity losses are disallowed on the taxpayer's current tax return and instead are deferred. D)excess passive activity losses are fully allowed in the year in which the taxpayer disposes of his entire interest in the passive activity in a taxable transaction.

B) losses from one nonpublicly traded passive activity may not offset income from another nonpublicly traded passive activity. The taxpayer's nonpublicly traded passive losses and nonpublicly traded passive income are aggregated for purposes of the limitation so that losses from one nonpublicly traded passive activity may offset income from another nonpublicly traded passive activity. LO 7.1.2

Yetunde has been selling sports memorabilia online for six years. It is not her primary employment, but she has been doing well with sales. While Yetunde will be itemizing her deductions this year, she believes she can report her income and expenses on Schedule C. Her gross sales from the sports memorabilia this year are $55,000. Her total expenses are $37,600, including cost of goods sold of $28,000. Yetunde has kept detailed records since she began selling memorabilia and can track her profit and loss for each year. In Year 1, she had a loss of $5,000; in Year 2, there was a loss of $2,000; in Year 3, she had a profit of $9,000; in Year 4, Yetunde had another loss of only $500; and Year 5 was a good year with a profit of $12,000. How should Yetunde report her sports memorabilia business for Year 6? A)Yetunde can use Schedule C to report the expenses of her business but must report the $55,000 as other income on the front of the Form 1040. B)Only cost of goods sold can be expensed on Schedule C; the rest of the business expenses are miscellaneous itemized deductions. C)Because this is not Yetunde's primary employment, the business is deemed a hobby and she should use hobby rules to report the income and expenses. D)Because Yetunde has a profit from the business in three of the last five years, the business is presumed not to be a hobby and she will report income and expenses using Schedule C.

Because Yetunde has a profit from the business in three of the last five years, the business is presumed not to be a hobby and she will report income and expenses using Schedule C. ecause Yetunde had a profit in Years 3, 5, and 6, the hobby rules do not apply and Yetunde can use Schedule C to report income and expenses from her business.

Which of the following statements regarding the rights of a limited partner is CORRECT? A limited partner has no right to take part in the management of the partnership. A limited partner is not subject to personal liability for partnership debts. A)I only B)Neither I nor II C)Both I and II D)II only

Both I and II

Martha and Max own Louisiana Properties, in which they manage their rental investment properties, both commercial and residential. Martha spends around 1,800 hours annually actively managing the business and it is her only business activity. Max spends 1,500 hours annually managing the business, usually during their summer and holiday rental seasons, and it is 70% of his business activity for the year. Which of the partners can fully deduct any real estate losses against active and/or portfolio income? Martha Max A)II only B)I only C)Neither I nor II D)Both I and II

Both I and II Both Martha and Max are considered real estate professionals because the activity comprises more than 50% of their personal services and they both participate in more than 750 hours annually. As real estate professionals, the partners may deduct any loss against active and/or portfolio income.

Which of the following statements regarding the at-risk rules and the passive activity loss limits is CORRECT? The at-risk rules are applied before the passive activity loss rules limits. If a loss is disallowed in any year because of the at-risk rules, the loss is suspended and taken in the first year that the at-risk amount is of an amount sufficient to absorb the loss. A)Neither I nor II B)I only C)Both I and II D)II only

Both I and II The at-risk rules are applied before the limits of the passive activity loss rules. They operate together such that, even if the taxpayer can avoid the restrictions of the at-risk rules, the passive activity loss rules will likely still preclude an income tax loss on an annual basis.

Which one of the following types of investors derives the greatest tax benefit from investing in preferred stocks? A) Government investors B) Mutual funds investors C) Corporate investors D) Nonprofit institutional investors

C) Corporate investors Because 50% of the preferred dividends received by a corporation are exempt from federal income taxes, a corporation gains a tax advantage. The government and nonprofit organizations pay no income taxes. Mutual funds are also exempt from taxation. (The Tax Cuts and Jobs Act, or TCJA, reduced the dividend-received deduction to 50%.) LO

Glen and Debbie both have significant net worth and are currently in the highest marginal income tax bracket. They have developed a process that allows them to neutralize toxic chemical waste. They want to form a business that will protect their net worth in case the business fails or it becomes involved in lawsuits. They expect the business to produce significant profits immediately, as they have agreements in the works with several large chemical companies. Also, they would like to share ownership with other family members as they get closer to retirement. Which business form would be most appropriate for Glen and Debbie at this time? A) S corporation B) Limited partnership C) General partnership D) C corporation

C corporation The S corporation (and the partnership entities) are conduits, and would cause the income to be added to their other significant individual income. The use of the C corporation would allow for protection from lawsuits or business failure. Also, the general partnership and the limited partnership both would cause a flow-through of the income, and the general partnership would not provide protection from personal liability. There is no indication that either party wants to be a general partner in a limited partnership. The use of a C corporation would be appropriate since it is a separate taxable entity, and the profits would not flow through. They could receive a reasonable salary, and the qualified dividends that are paid out would be subject to long-term capital gain rates.

Mark operates a sole proprietorship from his apartment. His gross income for the current tax year is $61,000. Business expenses not associated with his home office total $63,000. Expenses associated with the home office total $4,200. How much of the home office expense, if any, may Mark deduct for the current year? A) $4,200 B) $2,000 C) $0 D) $275

C) $0 The home office expense deduction is limited to the earned income from the business. In other words, the home office expense deduction can generally neither create nor add to a loss. In this situation, the $61,000 of gross income is reduced by the $63,000 of business expenses not associated with the home office, to leave no earned income. Thus, of the $4,200 of home office expenses, none would be deductible in the current year. Note that the entire $4,200 of home office expenses would be subject to a carryforward.

MSC, Inc. is a closely held C corporation that manufactures boilers. The company has been in business for over 45 years. MSC has active income this year of $250,000 and no passive or portfolio income. The company also leases equipment that generates passive losses of $120,000 per year. How much of the passive loss can the company use this year? A)$100,000 B)$25,000 C)$120,000 D)$0

C) $120,000 The company can deduct the entire $120,000 because it is a closely held C corporation that is not a personal service corporation. Passive losses may be used to offset active income, but not portfolio income.

Caroline has the following items of income to report in 2021: Salary of $65,000 from an S corporation for which Caroline is a 10% shareholder Her share of the net income from the same S corporation of $95,000 as reported on Schedule K-1 Schedule C net income of $15,000 from a small side business she owns How much of her income in 2021 will be subject to self-employment taxes? A)$95,000 B)$110,000 C)$15,000 D)$17,000

C) $15,000 Taxes are withheld from Caroline's salary at the S corporation in the same manner as any other employee of the S corporation. Income reported on the Schedule K-1 to shareholders of an S corporation is not considered self-employment income. Only the income from the small side business reported on Schedule C is subject to self-employment taxes.

Marcus purchased a diamond ring for $15,000 10 years ago. It was stolen in March this year. The ring was purchased to celebrate achieving a significant promotion at work. The FMV at the time of the theft was $20,000. The ring was insured, and after the deductible, Marcus received $19,000 from the insurance company. Marcus replaced the ring with a new one for $20,000. Under Section 1033, what is Marcus's new basis in the replacement ring? A) $20,000 B) $15,000 C) $16,000 D) $19,000

C) $16,000 Marcus's deferred gain on the new ring is $4,000. His new basis is the FMV of the property at acquisition minus the deferred gain ($20,000 − $4,000 = $16,000).

ABC Corporation has the following items of income and expense: Taxable income$300,000Federal income tax$80,000Dividends paid (all in current year)$40,000Accumulated earnings and profits at end of preceding tax year$160,000 Assume that XYZ Corporation is not a personal service corporation and cannot establish a valid business purpose for the excess accumulations. What is the amount, if any, of the accumulated earnings tax payable? A)$0 B)$6,000 C)$18,000 D)$26,000

C) $18,000 Explanation Taxable income$300,000Federal income tax(80,000)Dividends paid (all in current year)(40,000)Accumulated earnings credit ($250,000 - $160,000)1(90,000)Accumulated taxable income$90,00020%Accumulated earnings tax$18,000 1The accumulated earnings credit is the $250,000 accumulation limit minus accumulated earnings and profits at the end of the preceding tax year. LO 5.2.1

VUE Corp. entered into an equipment expansion in 2021. All of the equipment purchased totaled $215,000. Corporate taxable income before the election was $25,000. What is the maximum Section 179 expense election available to VUE Corp. in 2021? A)$0 B)$215,000 C)$25,000 D)$190,000

C) $25,000 The maximum Section 179 election amount available to VUE Corp. is $25,000. The election is limited to the lesser of the cost of the asset, the amount of business taxable income, or the annual Section 179 election amount for 2021 ($1,050,000).

Paul has the following items: Carryforward of prior year passive loss from: XYZ limited partnership (publicly traded)$(10,000)ABC limited partnership (nonpublicly traded)$(6,000) Current year passive income and loss from: XYZ limited partnership (publicly traded)$12,000GHI limited partnership (publicly traded)$(9,000)JKL limited partnership (nonpublicly traded)$18,000RST limited partnership (nonpublicly traded)$(14,000) What is the total amount of passive losses that Paul may deduct during the current year? A) $18,000 B) $14,000 C) $28,000 D) $30,000

C) $28,000 Publicly traded partnership (PTP) income may only be offset by prior year losses from the same partnership. Thus, the $10,000 XYZ carryforward is deductible. Nonpublicly traded income ($18,000) may be offset by current losses ($14,000) or carryforward losses ($6,000) from any nonpublicly traded activities. Thus, the $10,000 XYZ loss and the $18,000 nonpublicly traded loss total $28,000.

Ron has the following income for the current tax year: Schedule C net income: $12,000 Flow-through of general partnership operating income: $15,000 Wages: $80,000 Net income from rental of real estate: $10,000 What is the amount of Ron's self-employment tax? A) $1,696 B) $9,467 C) $3,815 D) $5,228

C) $3,815 The items included in the computation of self-employment income are the Schedule C income and the flow-through of general partnership operating income. The total of these items is $27,000. This is then reduced by 7.65% of this amount, and then multiplied by 15.3% to give us $3,815. Remember, the 7.65% subtraction always applies. Because the total of the self-employment income and the wages received are less than the wage base, no special computation is necessary.

Four years ago, Mark received a gift of 500 shares of common stock from his grandfather. The fair market value of the stock on the date of the gift was $335 per share. His grandfather had purchased the stock three years earlier at $425 per share. Mark sold this stock for $200 per share last week. What was Mark's basis in the stock when he sold it?

C) $335 per share When the fair market value on the date of the gift is less than the donor's basis in the asset and the sale price is less than the fair market value on the date of the gift, then the fair market value on the date of the gift is used as the donee's basis.

Eleven months ago, Lynnette received 1,000 shares of stock from her uncle, Joseph. Joseph purchased the stock eight years ago for $12 per share. The fair market value on the date of the gift to Lynnette was $9 per share, and she sold the stock today for $5 per share. What is the amount and character of Lynnette's loss from the sale of the stock? A) $7,000 long-term capital loss B) $3,000 long-term capital loss C) $4,000 short-term capital loss D) $3,000 short-term capital loss

C) $4,000 short-term capital loss There are two components to this question. What is the basis, and is there tacking of the holding period? When the fair market value on the date of the gift is less than the donor's basis in the asset, the donee's basis in the asset for purposes of determining a loss is the asset's FMV on the date of the gift. In this situation, the $9 per-share value on the date of the gift would be Lynnette's basis. The next issue is the "tacking" of the holding period. In a situation where the donee uses the FMV as the basis, there is no tacking of the holding period. In this situation, Lynnette used the FMV; thus, she uses her own holding period of 11 months. If the donee uses the donor's basis, then the holding period is tacked. In other words, the donor's holding period is added to ("tacked") the donee's holding period.

Which of the following statements correctly defines inside buildup as it refers to life insurance? A) During the insured's lifetime, the accumulations of cash value within a policy grow on a tax-annuitized basis. B) During the insured's lifetime, the accumulations of cash value within a policy grow on a tax-preferred basis. C) During the insured's lifetime, the accumulations of cash value within a policy grow on a tax-deferred basis. D) During the insured's lifetime, the accumulations of cash value within a policy grow on a tax-free basis.

C) During the insured's lifetime, the accumulations of cash value within a policy grow on a tax-deferred basis. Accumulations of cash value within a life insurance policy grow on a tax-deferred basis during the insured's lifetime.

You are a CFP® professional and are meeting with your client Brenda to monitor her ongoing financial status. Brenda owns a vacation home in New Mexico. She is now renting the property to others for the entire year except for 10 days during the summer when she and her family used it for their vacation. The gross rental income that Brenda received is $65,000. The expenses for the home for both the rental period and her personal use total $5,000. Brenda would like you to explain how this will change her income tax situation, in particular, how much of the rental expenses are deductible. After reviewing the documents she sent to you prior to the meeting, you have an answer for Brenda. How much of a deduction for rental expenses can Brenda take on her tax return? A)$5,000 B)$4,721 C)$4,863 D)$3,411

C) $4,863 Brenda can deduct the cost of renting the home if she occupies it for the greater of no more than 14 days per year or for 10% of the number of days the property is rented. Because Brenda occupies the house for only 10 days during the year, this test is satisfied. Even though this rental use exception is allowed, the deductible expenses related to the rental of the house are limited. Specifically, she can only deduct a portion of the actual rental expenses, which equals the number of days during the year that the house is rented to others divided by the total number of days that the house is used by either tenants or Brenda. Given 365 days per year, Brenda's tenants occupy the house all but 10 days, for a total of 355 days. She is allowed to deduct 97.26% (355 days ÷ 365 days) of the $5,000 rental expenses, which equals $4,863.

Which of the following benefits that Claudia has received from her employer can be excluded from taxation? A) A year-end bonus. B) A company car that she uses for personal vacations. C) $5,000 of graduate education assistance. D) An athletic membership at a local club valued at $1,500 per year.

C) $5,000 of graduate education assistance. Undergraduate and graduate education assistance is excluded from an employee's income in any one year period, up to a maximum of $5,250. The other options are fully taxable.

In the current year, Keith had passive losses of $19,000 from a real estate limited partnership purchased in 1985. He also had passive income of $7,000 from an oil and gas limited partnership. Both limited partnerships are not publicly traded. What is the total amount of passive losses that may be used to offset active, passive, and portfolio income in the current year? A)$8,200 B)$1,900 C)$7,000 D)$9,400

C) $7,000 The passive loss ($19,000) is deductible, but only up to the amount of passive income ($7,000) in the same year; thus, the answer is $7,000.

Jean leases a house from Nan. The rent is normally $850 per month. Nan accepts reduced monthly rent of $500 per month for the current year in addition to Jean providing repairs to the property. The repairs are worth $2,500 and cost $3,200. How much must Nan include in income due to the house rental? A)$9,200 B)$10,200 C)$8,500 D)$6,000 PREV

C) $8,500 The actual rent received of $6,000 plus the fair market value of the services provided, $2,500, are included in income.

What is the amount and character of the penalty for an employer who fails to withhold Social Security and federal income taxes from employee paychecks? A) 50%, and the responsible party needs to pay back what is owed plus a 50% penalty B) 100%, and the responsible party needs to pay back what is owed plus a 100% penalty C) 100%, but the responsible party needs only to pay back what is owed with no additional penalty D) 50%, but the responsible party needs only to pay back 50% of what is owed with no additional penalty

C) 100%, but the responsible party needs only to pay back what is owed with no additional penalty Employers are required to withhold amounts from an employee's paycheck for Social Security taxes and federal income taxes. If the employer fails to do so or fails to pay such amounts to the IRS, they or any other responsible person will be subject to the 100% penalty, which is simply having to pay 100% of the amount they should have collected, accounted for, and paid; they are not subject to any additional penalty. LO 8.2.1

What percentage, if any, of a taxpayer's self-employment tax may be deductible as an adjustment to income? A) 75% B) 25% C) 50% D) 0%

C) 50% Only half (50%) of a taxpayer's self-employment tax liability is deductible as an adjustment to income.

Which one of the following is CORRECT regarding an active participation rental real estate activity? A)A cumulative deduction of up to $25,000 is available over the life of the activity. B)A cumulative deduction-equivalent tax credit of up to $25,000 is available over the life of the activity. C)A deduction of up to $25,000 is available annually. D)A deduction-equivalent tax credit of up to $25,000 is available.

C) A deduction of up to $25,000 is available annually. In meeting several tests, an individual with active participation in real estate may deduct up to $25,000 of rental real estate losses against active and portfolio income in any one year. LO 7.2.3

Sally Franklin has AGI of $300,000. In addition, she currently has passive income of $150,000 and passive losses of $175,000—$150,000 of which she uses to offset the passive income and $25,000 of which is subject to disallowance. Which one of the following investments has the greatest potential for reducing Sally's tax liability?

C) A working interest in an oil and gas general partnership A working interest in an oil and gas partnership can provide unlimited loss deductions against other income. Congress considers this socially desirable to encourage investment in the industry. The caveat is investors in these instruments must also assume unlimited liability. Therefore, a working interest must be a general partnership rather than a limited partnership. The other answer choices cannot offset passive income

Which of the following methods of accounting is mandatory for businesses that maintain inventory and have gross receipts in excess of $26 million? A) Hybrid method of accounting B) IRS method of accounting C) Accrual method of accounting D) Cash method of accounting

C) Accrual method of accounting Under the cash method of accounting, a taxpayer generally reports income when any cash is collected (or the constructive receipt income tax doctrine applies) and reports expenses when any cash payment is made. The accrual method is the conceptual opposite of the cash method and requires recognition of taxable income in the same tax year it is reported on the taxpayer's financial statements; it is mandatory for businesses maintaining inventory and have gross receipts in excess of $26 million. The hybrid method of accounting is a combination of the accrual method and the cash method of accounting. There is no IRS method of accounting.

Which of the following forms does a C corporation file? A) Form 1065 B) Form 1120-S C) Form 1120 D) Form 1120-C

C) Form 1120 The C corporation files a tax return with the IRS—this is the Form 1120.

Which of the following statements regarding alimony paid under a 2021 divorce agreement is CORRECT? A) Alimony is deductible by the payor spouse, and includible in income by the payee spouse, to the extent that the payment is contingent on the status of the divorced couple's children. B) Alimony may be paid in either cash or property. C) Alimony payments must terminate on the death of the payee spouse. D) The divorced couple may be members of the same household at the time the alimony is paid.

C) Alimony payments must terminate on the death of the payee spouse. Payments to former spouses are no longer deductible and are considered alimony only if the payments are made in cash (and not property); the decree does not specify that the payments are not alimony for federal income tax purposes; the payor and payee are not members of the same household at the time that the payments are made; and there is no liability to make the payments for any period after the death of the payee.

Macy, age 22 and disabled, has been living with her older brother, Leon, since last December when their parents died in an auto accident. Leon has been providing all of her support as her share of the life insurance benefit was put into a trust for her college education. Leon is asking his planner about Macy's status as a qualifying relative because her life insurance benefit was $250,000. He hopes to use the head of household status this year. What does the planner tell him? A) Even though Leon supported Macy, Leon cannot list her as a dependent because her insurance benefit was so large. B) Only the trust can list Macy as a dependent on its return. C) Because Leon provided all of Macy's support this year, he may claim her as a dependent. D) Leon cannot list Macy as a dependent and claim head of household status.

C) Because Leon provided all of Macy's support this year, he may claim her as a dependent. Because Leon provided all of Macy's support this year, he can claim her as a dependent and claim head of household status.

Which of the following statements regarding limited partnerships is CORRECT? A limited partner is subject to the passive activity rules when accounting for income and losses from the limited partnership. The limited partner is liable to the creditors of the partnership only to the extent of that partner's contributed or promised cash or property. A) Neither I nor II B) II only C) Both I and II D) I only

C) Both I and II

Max and his mother, Lucy, live together in his home. Lucy is bedridden, and Max must pay a caregiver to provide meals and other aid during the day so he can leave the house and work. He pays $5,000 annually for this service. His mother has no income, and he is her full support. What tax relief may be available to Max for the expenses of caring for his elderly mother? A greater standard deduction amount Child and dependent care credit A) I only B) II only C) Both I and II D) Neither I nor II

C) Both I and II Max is entitled to both the standard deduction amount for the head of household filing status and the child and dependent care tax credit, subject to the limits based on Max's AGI.

Which of the following parties may be subject to IRS penalties? A) Individual taxpayer only B) Tax preparer only C) Both the individual taxpayer and the tax preparer D) The individual taxpayer when the tax preparer has closed their business

C) Both the individual taxpayer and the tax preparer Both clients and tax professionals alike may be held accountable for tax reporting errors. Financial planners should also be aware that a tax return preparer might be subject to certain penalties. These penalties include failure to provide a taxpayer with a copy of their return, failure to keep a copy of all returns prepared for at least the last three years or to maintain a list of returns prepared, and failure to sign a return as preparer and give their tax identification number on the return.

tom and Jeanette are married taxpayers filing jointly. Jeanette is the owner of a chain of successful car washes, from which she receives a substantial portion of her income in cash. Last year, the car washes had sales of $160,000, but Jeanette intentionally failed to report $12,000 of the sales that she received in cash. Which one of the following penalties is the Internal Revenue Service most likely to apply if it determines that Jeanette underreported her income? A) Underreporting penalty B) Substantial understatement penalty C) Civil fraud penalty D) Negligence penalty

C) Civil fraud penalty This is essentially taxpayer fraud that does not rise to the level of criminal fraud (most of the income was declared), but is willful. If imposed, the penalty is 75% of the portion of tax underpayment attributable to fraud.

Claudia will begin a private tutoring business for students for whom English is a second language and who need assistance preparing for the SAT. She will be using a room at the university library for the tutoring sessions, and except for some small initial costs for materials, she does not expect any expenses. If Claudia receives $9,000 in tutoring income this year, how does she account for it on her income tax return? A)Until Claudia can prove the tutoring is not a hobby (achieve a profit in three of the next five years), she will report gross tutoring income on the front of Form 1040. B)Because the tutoring is similar to the teaching she does at the university, Claudia is required to report it to the university, which will put the income on her W-2. C)Claudia must report the tutoring income and expenses on Schedule C of Form 1040. D)Because her expenses are negligible, Claudia is not allowed to deduct the expenses from her tutoring income.

C) Claudia must report the tutoring income and expenses on Schedule C of Form 1040. The tutoring business is engaged in for profit, not as a hobby. The income and expenses are reported on Schedule C of Form 1040.

Which of the following forms of business could NOT be a direct participation program (tax conduit)? A) S corporation B) Limited liability partnership (LLP) C) Closely held C corporation D) Limited liability company (LLC)

C) Closely held C corporation The tax advantages provided by direct participation programs are founded upon the principle that most types of business organizations function as tax conduits. Therefore, a closely held C corporation cannot qualify, as there is no flow through of gains and losses. LO 7.1.1

Which of the following is NOT an advantage of the cash basis method of accounting? A) Income is reported when it is received. B) Taxpayers can keep simple records. C) Constructive receipt serves to defer income. D) Taxpayers have more control over each year's income and expenses.

C) Constructive receipt serves to defer income constructive receipt, which serves to accelerate income, is not considered an advantage of the cash method of accounting.

Mike is involved in the business of breeding show horses on a part-time basis as a sole proprietor. He has shown a significant net loss in this business every year for the last six years, and he has used these losses to reduce the tax liability on the salary he has earned from his law practice. With which one of the following potential tax traps should Mike be most concerned? A) Assignment of income B) Substance over form C) Hobby loss D) Passive activity

C) Hobby loss The fact that this business has not been profitable and is merely a side business increases the likelihood it could be reclassified as a hobby loss. Mike has operated the business only part time, and he has not demonstrated that he is attempting to make it a profitable business. The fact that huge losses have occurred every year overrides the presumption that Mike is in the business to make a profit.

Landry has been baking pies and cakes for several years that are very popular when he sells them locally. He works full-time in the oil field and baking has become a hobby he enjoys. He is meeting with his new financial planner, Richard, to review his plans for retirement in 10 years. He explains that he will continue his baking after retirement but not necessarily full time. Richard has asked for as much documentation as possible about his current and prior years' sales and expenses for this hobby. Landry is confused about why it matters. What should Richard do for Landry as his planner? Richard should review Landry's previous income tax returns to ascertain if the hobby income was reported. When reviewing the documentation for the hobby, Richard is also determining whether or not the hobby could be classified as a business. Landry should file amended returns and deduct all of his hobby expenses from the hobby income before it is entered on his Form 1040 if Landry has not previously reported the hobby. A)II and III B)I only C)I and II D)I, II, and III

C) I and II Statement III is incorrect. If Landry has not previously reported the hobby income and expenses, Richard should advise him to file amended returns, reporting the income on the front of the Form 1040. Hobby expenses are not deductible except for allocated property taxes and mortgage interest.

Taxpayers owning certain resources may recover the cost of exhausting that natural resource (cost recovery) using which of the following methods? Cost depletion Percentage depletion Amortization over 15 years A)II only B)I and III C)I and II D)II and III

C) I and II Statements I and II are correct. Statement III is incorrect. Amortization is used for Section 197 intangible assets. LO 7.1.1

Which of the following incomes is not taxed under Social Security self-employment tax? Rental real estate income Small part-time repair shop income Shareholder's share of S corporation's income in excess of salary Income of an individual working as an independent contractor A)II and IV B)I, II, III, and IV C)I and III D)I, II, and III

C) I and III Income from rental property is not subject to Social Security tax, nor are dividends to S corporation shareholders/employees as long as the employees' compensation is considered reasonable.

Which of the following are requirements in order for a taxpayer who materially participates in a real property trade or business to be able to deduct any losses from the business? More than 50% of the individual's personal services during the tax year are performed in the real property trades or businesses in which the individual materially participates. More than 10% of the individual's net assets at the end of the tax year are invested in the real property trades or businesses in which the individual materially participates. The individual performs more than 750 hours of service in the real property trades or businesses in which the individual materially participates. A) I only B) I and II C) I and III D) II and III

C) I and III Options I and III are specific requirements that must be met in order for a taxpayer to able to deduct any losses from a real estate trade or business. The amount of the individual's net assets used in the business is not relevant.

Max is selling a truck that he uses in his business. He has taken $5,000 of depreciation on the truck and wants to use the installment sale method to sell the truck to Jerry for a down payment and an installment note over 36 months. He paid $40,000 for the truck and is selling it for $38,000. What are the tax consequences of this transaction? Max must recapture $3,000 of the Section 1245 depreciation taken as ordinary income in the year of the sale. Max has $5,000 of depreciation recapture taxed at the 25% tax rate. A) Neither I nor II B) II only C) I only D) Both I and II

C) I only Statement I is correct. Gain recaptured under Section 1245 (depreciable personal property used in a trade or business) is taxed as ordinary income and is not eligible for installment sale treatment. Therefore, these amounts are fully recognized (taxable) as ordinary income in the year of sale. Unrecaptured Section 1250 depreciation occurs only on depreciable real property (real estate) used in a trade or business.

Jack bought publicly traded stock seven years ago for $6,000. Its current value on the securities market is $11,000. He has donated this appreciated stock to a charity that provides housing for the homeless. What must Jack do to take the donation as a charitable deduction? Jack must have documentation from the charity substantiating the amount of the donation, the date donated, and the name of the charity. All donations of stock must have a qualified appraisal of the stock attached to the donor's income tax return. A) Neither I nor II B) II only C) I only D) Both I and II

C) I only Statement I is correct. In additional, the taxpayer must be in receipt of this documentation by the due date of the return or when the return is filed. Statement II is incorrect because it reads "all" donations of stock must have a qualified appraisal. A qualified appraisal is not required for closely held stock if the amount donated is less than $10,000. The appraisal itself is not attached to the tax return.

Brandon failed to file a federal income tax return for last year's tax liability by April 15 of the current year. He filed his tax return October 30 of the current year and remitted the tax that was due. Which of the following statements is CORRECT? Brandon will owe interest to the IRS on the unpaid tax liability that runs from April 15 of the current year until the tax liability is paid in full. Because he took so long to file, the 90%/100% payment criteria does not apply. Brandon will be assessed the failure-to-file penalty. Depending on the facts and circumstances surrounding the return filed, Brandon could be assessed a negligence penalty. A) I, II, III, and IV B) III and IV C) I, III, and IV D) I only

C) I, III, and IV Statement II is incorrect. The 90%/100% underpayment criteria will still apply even though Brandon did not file in a timely manner. All of the other statements are correct.

Which of the following statements regarding IRS audits is CORRECT? A correspondence audit is usually performed through the mail because the disputed tax issue is minor. The IRS is not required to disclose the criteria it uses in selecting returns for audit. An office audit is usually restricted in scope to a specific item or items and is performed at the IRS office by an office auditor. A field audit is an examination of numerous items and is usually performed on the premises of the taxpayer (such as a business office) by a revenue agent. A)I and IV B)I, II, III, and IV C)I, III, and IV D)III only

C) I, III, and IV Statements I, III, and IV are correct. Statement II is incorrect. The IRS is required to disclose the criteria it uses in selecting returns for audit and to take steps to protect the confidentiality of personal and financial information.

Bill, an engineer, is contemplating forming an S corporation for his practice. He will be the sole employee of the corporation. Which of the following statements accurately describe the income tax consequences of such an arrangement? All of the flow-through of net income from his S corporation is subject to the self-employment tax. Bill must draw a reasonable salary as an employee of the S corporation. The net income of the corporation will be taxed at Bill's individual tax rate. The net income of the corporation will be subject to a flat 21% tax rate. A)I, II, and IV B)II and IV C)II and III D)I and IV

C) II and III The S corporation income will flow through to Bill and be taxed at his marginal rate. The flow-through is not subject to the self-employment tax. However, Bill must receive reasonable wages or salary, or the IRS will reclassify part of the net income as salary and impose penalties.

A taxpayer invested in a real estate limited partnership several years ago. There is a special allocation in effect in the partnership. He is concerned about the deductibility of the losses that are flowing from the partnership. Which of the following rules or doctrines may limit the availability of income tax benefits from his limited partnership investment? Direct participation program Special allocation rules At-risk rule Passive activity loss rule

C) II, III, and IV Special allocation rules, such as the substantial economic effect doctrine, limit the ability to utilize special allocations in a partnership. The at-risk rule limits the ability to utilize leverage by attacking the use of nonrecourse financing. The passive activity loss rule limits the ability to deduct losses from activities in which the taxpayer does not materially participate. The direct participation program simply refers to a tax conduit.

Which of the following rules or doctrines may limit the availability of income tax benefits from a particular investment? Tax conduit The substantial economic effect doctrine The at-risk rule The passive activity loss rule A)II and III B)I and IV C)II, III, and IV D)I and III

C) II, III, and IV The substantial economic effect doctrine limits the ability to use special allocations in a partnership. The at-risk rule limits the ability to use leverage by attacking the use of nonrecourse financing. The passive activity loss rule limits the ability to deduct losses from activities in which the taxpayer does not materially participate. LO 7.1.2

Which one of the following is NOT a group of the IRS Regulations? A) Interpretive B) Procedural C) Implementative D) Legislative

C) Implementative Regulations can be classified into three groups: (1) legislative, (2) interpretive, and (3) procedural.

Which one of the following provides the greatest federal revenue from IRS taxation? A) Corporate income taxes B) Gift taxes C) Individual income taxes D) Estate taxes

C) Individual income taxes The three main sources of federal tax revenue are individual income taxes, corporate income taxes, and payroll taxes. Individual income tax accounts for approximately 40% of the total tax revenue collected by the federal government. The federal estate tax and gift tax actually compose only a small percentage of annual tax revenues.

Which one of the following best describes the role of a special allocation in a limited partnership? A) It establishes the standards for allocating the proceeds of non-routine or "special" items of income. B) It requires all items to be distributed pro rata based on a partner's capital account balance. C) It allows an allocation of items of income and expense that is not pro rata. D) It allocates management responsibility to the general partners.

C) It allows an allocation of items of income and expense that is not pro rata. A special allocation allows an allocation of items in a manner that differs from the "normal" pro rata allocation of deduction, income, credit, etc.

Lacey is a single taxpayer. Lacey's aunt, Blanche, has come to live with her after it was determined she could no longer live independently. She is physically challenged and needs full-time care. Blanche's parents set up a trust for her that supplies all of her support, including paying for in-home care and all medical bills. Lacey wants to know how this will affect her income tax situation. What does the planner tell her? A)Lacey's filing status changes to head of household. B)Lacey may list Blanche as a dependent on her income tax return. C)Lacey and Blanche will both file as single. D)Blanche is a qualifying relative.

C) Lacey and Blanche will both file as single. Because the trust pays for all of Blanche's support, Lacey's income tax return is unaffected. Both Lacey and Blanche will file as single.

Which one of the following statements is CORRECT with respect to capital gains and losses? A) Excess capital losses are carried forward for up to five years. B) Net capital gains are always taxed at a flat rate of 15%. C) Net capital losses are deductible up to $3,000 annually. D) Net capital gains are always taxed at a maximum rate of 28%.

C) Net capital losses are deductible up to $3,000 annually. Net long-term capital gains (LTCG) (from other than unrecaptured Section 1250 income and collectibles) are taxed at rates of 0%, 15%, or 20%. For married couples filing jointly, the 0% long-term capital gain rate ends at $80,800 of taxable income. For long-term capital gains falling between the $80,800 breakpoint and $1,600 of taxable income (again, for married couples filing jointly), the rate is 15%. For long-term capital gains falling into taxable income levels above $1,600 (MFJ), the rate is 20%. The table shows the breakpoints for LTCG and qualified dividend preferential rates. LTCG Rates Based on Taxable Income Filing Status0% rate15% rate20% rateSingleUnder $40,$40,-$445,850Over $4450Head of householdUnder $500$5100-$473,750Over $4,750Married filing jointlyUnder $80,0$80,-$501,600Over $1,600Estates and trustsUnder $2,$2,0-$13,250Over $13,0 Special rates apply to the sale of real estate or collectibles—25% (the maximum rate for gain attributable to straight-line depreciation on real estate), or 28% (maximum rate in the case of collectibles). Net capital losses, the capital losses remaining after netting against capital gains, are deductible up to $3,000 per year with an indefinite carryforward.

Cindy has adjusted gross income (AGI) of $350,000. Included in the AGI is passive income of $40,000 and passive losses of $55,000, $40,000 of which she uses to offset the passive income and $15,000 of which is subject to carryforward. Which one of the following activities has the greatest potential for reducing Cindy's tax liability? A) Investing in a real estate partnership in which she will not materially participate that is producing passive losses B) Investing in "active participation" rental real estate that is producing a loss C) None of these options will reduce Cindy's tax liability D) Investing in an oil and gas limited partnership that is generating losses

C) None of these options will reduce Cindy's tax liability The active participation deduction is eliminated at $150,000 of AGI. The oil and gas limited partnership and the equipment-leasing limited partnership would produce more passive losses that are nondeductible. Therefore, none of the options are viable. LO 7.1.2

Which of the following is a permissible method for determining the business part of a residence for purposes of the home office expense deduction? A) Relative use method of allocation B) Amount of utility use compared to that of the whole house C) Number of rooms in the house used for business compared to total rooms in the house D) Accrual method of allocation

C) Number of rooms in the house used for business compared to total rooms in the house Both the relative square footage of the home office area and the relative number of rooms may be used in determining the business part of a home. The amount of utility use can be a deduction, but is not used in determining the business part of a home. There is no recognized method known as the relative use method of allocation.

How can passive activity losses from an ongoing nonpublicly traded partnership, such as a RELP, be deductible from other taxable income? A)Passive losses from one partnership can only be offset by passive gains from the same partnership. B)Passive activity losses are deductible against portfolio gains. C)Passive losses can offset passive gains. D)Passive losses can only be carried forward against future passive gains.

C) Passive losses can offset passive gains. Generally, passive activity losses can only be used to offset passive activity income. Losses from nonpublicly traded partnerships (non-PTP) can be used to offset gains from other nonpublicly traded partnerships. Losses from master limited partnerships can only offset gains from the same master limited partnership. Losses from a non-PTP may be carried forward, but it is not a requirement. Passive losses cannot be used to offset portfolio income.

Which of the following is NOT a requirement to be met in order to deduct alimony payments for a divorce finalized in 2018? A)The parties must not file a joint tax return at the time of payment. B)Payments must be in cash. C)Payments must be equal in each year of the agreement. D)The legal document or state law must require that payments will stop after the recipient spouse dies.

C) Payments must be equal in each year of the agreement. All of the options are requirements except the payments must be equal each year of the agreement. However, unequal and declining payments can trigger the alimony recapture rules.

Which one of the following is a publication of specific taxpayer guidance from the IRS? A) Revenue Procedures B) Revenue Rulings C) Private Letter Rulings D) Regulations

C) Private Letter Rulings Private Letter Rulings are taxpayer guidance from the IRS that apply only to the particular taxpayer(s) asking for the ruling; they are not applicable to all taxpayers. The primary purpose of the regulations is to explain and interpret particular IRS Code sections. LO 8.1.1

Sam has had persistent car trouble and decided to buy a new truck to make certain he gets to work on time. He received an interest-free loan of $5,000 for the needed down payment from his employer this year on January 1. The federal interest rate was 4% this year. Sam's only income this year was his salary of $40,000 and interest income of $75. Which of the following statements regarding this loan is CORRECT? A)Because this is a compensation-related loan, it results in compensation expense for Sam's employer and compensation income for Sam of $200. B)Sam has nondeductible interest expense of $200. C)Sam has no compensation income this year from interest imputed on the loan. D)Sam's employer must recognize $75 of interest income this year because of the loan to Sam.

C) Sam has no compensation income this year from interest imputed on the loan. In a compensation-related below-market loan of $10,000 or less, no interest is imputed and no compensation results. LO 3.3.1

During the current tax year, Rod purchased a building for exclusive use in his manufacturing business. The cost of the property was $422,000, of which $122,000 was attributable to the land. Which of the following statements identifies the proper treatment of the expenditure? A) The $122,000 must be capitalized and may be depreciated. B) The $300,000 attributable to the building may be currently deductible. C) The $122,000 must be capitalized and may not be depreciated. D) The cost attributable to the building may be deducted under Section 179.

C) The $122,000 must be capitalized and may not be depreciated. The land may not be depreciated, as only "wasting" assets are subject to depreciation. The Section 179 expense election generally applies to personalty only, and is not available for most real estate. The cost of the building may not be currently deducted; it must be capitalized and depreciated because it has a useful life of over one year. LO 6.1.1

Jena owns and operates a string of retail electronics stores with approximately $30 million of sales annually. Approximately 20% of her sales are with extended credit terms. What method of tax accounting is most appropriate for Jena's business? A) The installment sale method, to spread the gain over a longer time frame B) The cash method, because it provides flexibility in the timing of income and expenses C) The accrual method, because inventory is such a large component of the business D) The hybrid method, because the business involves both inventory and service

C) The accrual method, because inventory is such a large component of the business The accrual method of accounting generally is mandatory when inventory constitutes a significant income-producing factor. Thus, the cash method is incorrect. The hybrid method is incorrect because there is no indication that service constitutes a significant portion of the business. Also, the installment sale method is not available for sales of inventory, or sales with revolving credit terms. With annual sales of $30 million, the accrual method exception does not apply. If she had annual sales of under $26 million (2021), she could still use the cash method, even though she has inventory.

Marco had an individual AMT credit two years ago and has not yet used it. Which of the following is correct regarding Marco's AMT credit? A)The credit must be applied against the AMT liability in the next year there is AMT to be paid. B)The credit can be carried forward indefinitely to be applied against a future alternative minimum tax liability. C)The credit can be carried forward indefinitely to be applied against a regular income tax liability. D)The credit must be applied against the regular income tax liability in the year the credit is created.

C) The credit can be carried forward indefinitely to be applied against a regular income tax liability. The AMT credit created in any one year may be used as a credit against regular income tax in a future year. It may be carried forward indefinitely.

Ann Hamilton owns 500 shares in the XYZ S&P 500 Index Fund. The basis of her investment in this fund is $4,500, while the fair market value is only $2,000. She wants to sell her shares to "lock in" the $2,500 loss, but she is considering buying 500 shares of the GRC Small-Cap Index ETF the following week because she believes that the value is going to increase significantly over a longer period. As her planner, what can you accurately tell Ann about this scenario? A) The basis in the newly acquired shares would be the amount paid for those shares, increased by the $2,500 disallowed loss. B) If the loss were disallowed, the basis in the newly acquired shares would be decreased by the disallowed loss. C) The loss would be a fully deductible capital loss. D) She should wait a minimum of 61 days after the sale to repurchase the shares so that the loss may be recognized.

C) The loss would be a fully deductible capital loss. The wash sale rule disallows a loss if substantially identical securities are purchased prior to 30 days after the sale that resulted in the loss. The basis of the acquired securities is increased by the amount of the disallowed loss. The S&P 500 mutual fund should not be substantially identical to the small-cap ETF because the funds track very different indices and because of the difference in the way ETFs trade compared with mutual funds.

Mary has owned her principal residence for over six years. Two years ago, she married John, who immediately moved into the residence. John has never used the Section 121 exclusion. If Mary sells the residence this year and John and Mary file a joint return, which of the following statements is CORRECT with respect to the availability of the Section 121 exclusion? A) The maximum exclusion is $250,000 because that is the maximum exclusion for an individual who was single when the residence was purchased. B) The maximum exclusion is $250,000 because John is not an owner of the residence. C) The maximum exclusion is $500,000 because Mary has at least two years of ownership, and both spouses meet the use requirement. D) The maximum exclusion is $500,000 because that is the amount always available for married taxpayers who file jointly.

C) The maximum exclusion is $500,000 because Mary has at least two years of ownership, and both spouses meet the use requirement. Currently, Section 121 allows for a gain exclusion, of up to $500,000 for taxpayers married filing jointly, to any taxpayer who satisfies certain tests, known as the ownership test and the use test. To satisfy the ownership test, the home must have been owned and used as a principal residence for at least two of the five years preceding the date of sale. (Note: These years do not have to be consecutive; they only have to add up to at least two years.) Either spouse can meet the ownership test, but both must meet the use (two-out-of-five-year) test. This is likely not difficult for most married couples (applies even to those living in the house and then getting married), but it can be burdensome for individuals who are divorced or in the process of a divorce.

Which one of the following is an exception to the general rule that life insurance proceeds are excluded from income?

C) The transfer for value rule The only exception to the general rule that life insurance proceeds are excluded from income is the transfer for value rule, which applies when a life insurance contract is transferred for valuable consideration.

Tim Jones is single, 21 years old, and in his third year of college. He has an AGI of $35,000 and receives no support from his parents. The college is a Title IV institution where students are eligible to receive federal financial aid, and Tim is pursuing an undergraduate degree in criminal justice. When Tim was 13, his parents established a Uniform Transfers to Minors Act (UTMA) for him, and funded it with EE savings bonds. When Tim was a freshman, he was convicted of a felony drug possession charge. Which one of the following is CORRECT regarding Tim's situation? A) Tim could use both the American Opportunity Tax Credit and the Lifetime Learning Credit in the same year. B) Tim qualifies for the American Opportunity Tax Credit. C) Tim qualifies for the Lifetime Learning Credit. D) Tim may redeem the EE bonds potentially tax free if the proceeds are used for his qualifying education expenses

C) Tim qualifies for the Lifetime Learning Credit. Tim qualifies for the Lifetime Learning Credit. His AGI is under the phaseout range. He is pursuing a degree at an eligible institution. The felony drug conviction would preclude the use of the American Opportunity Tax Credit but not the Lifetime Learning Credit. There is no exclusion available for EE bonds unless they are held by the individual who purchases the bonds or unless they are held jointly with a spouse. A bond that has been gifted to another taxpayer does not qualify for the exclusion. The American Opportunity Tax Credit and the Lifetime Learning Credit may not be claimed in the same year for the same student. LO 2.4.1

Which one of the following reflects the CORRECT sequence of steps in the tax calculation process?

C) Total income minus adjustments to income equals AGI. Total (gross) income minus adjustments to income equals adjusted gross income (AGI). AGI minus standard or itemized deduction(s) equals federal taxable income.

Which one of the following steps is CORRECT concerning the tax calculation process?

C) Total tax liability minus withholding and/or estimated tax payments equals refund or tax owed Tax liability minus tax credits plus additional taxes owed equals total tax liability. Then total tax liability minus withholding and/or estimated tax payments made equals refund or tax amount owed. Employment eligibility is submitted via Form I-9 and is not considered part of the 1040 calculation

A small business with under $20 million in gross receipts and no inventory should most likely use which accounting method? A) Cash B) Hybrid C) LIFO D) Accrual

Cash A typical small business would use the cash method of accounting. Due to cost and complexity, the accrual and hybrid methods would NOT be recommended. A business is not required to use the accrual method until it has at least $26 million in gross receipts (2021). A business that has no inventory would not use LIFO.

Bradley, Inc., sold some land on June 30 of the current year for a $15,000 gain. The land was originally purchased three years ago and was classified as a Section 1231 asset. This was the only asset sale for this year. In the previous year, Bradley, Inc., had an $8,000 net Section 1231 loss. For the current year, Bradley's net Section 1231 gain is treated as A)a $15,000 ordinary loss. B)a $8,000 long-term capital gain and a $7,000 ordinary loss. C)a $7,000 long-term capital gain and a $8,000 ordinary gain. D)a $15,000 ordinary gain.

C) a $7,000 long-term capital gain and a $8,000 ordinary gain. When the taxpayer has a net Section 1231 gain for the year, the lookback rule may recapture some or all of the net gain as ordinary income. To the extent the lookback rule does not apply, the net gain is treated as a long-term capital gain. When determining how much of the gain is treated as capital gain and how much must be treated as ordinary gain, the taxpayer is looking back to the five previous taxable years, not including the current taxable year. LO 6.1.4

Beth's husband died in Year 1. Assume that Beth does not remarry and continues to maintain a home for herself and her dependent child during Year 2, Year 3, and Year 4, providing full support for her child throughout those years. For Year 4, Beth's filing status will be

C) head of household. Beth's Year 4 filing status is head of household. Qualifying widow filing status is only available for 2 years following the death of a spouse (Year 2 and Year 3

All of the following are tax avoidance techniques except A) investing in tax-free municipal bonds. B) utilizing tax credits such as qualifying child care expenses. C) investing in zero-coupon U.S. Treasury bonds. D) owning, rather than renting, a residence to benefit from claiming a home mortgage interest deduction.

C) investing in zero-coupon U.S. Treasury bonds Taxes must be paid on accrued interest on zero-coupon Treasury bonds even though no cash income is received. The other choices are legitimate tax avoidance techniques.

All of the following statements regarding tax deduction limits on passive activity excess losses are correct except A)excess passive activity losses are fully allowed in the year in which the taxpayer disposes of his or her entire interest in the passive activity in a taxable transaction. B)excess passive activity losses are the excess of otherwise allowable deductions from the taxpayer's passive activities over the amount of income from the taxpayer's passive activities. C)losses from one nonpublicly traded passive activity may not offset income from another nonpublicly traded passive activity. D)excess passive activity losses are disallowed on the taxpayer's current tax return, but, instead, are deferred. PREV

C) losses from one nonpublicly traded passive activity may not offset income from another nonpublicly traded passive activity. The taxpayer's nonpublicly traded passive losses and nonpublicly traded passive income are aggregated for purposes of the limitation, so that losses from one nonpublicly traded passive activity may offset income from another nonpublicly traded passive activity. LO 7.2.2

With respect to the at-risk rules, qualified nonrecourse financing is a debt A) that is convertible into an equity interest. B) from a loan provided by a related person at "below market" terms. C) secured by specific real property. D) for which at least one individual is personally liable.

C) secured by specific real property. A partner's share of nonrecourse financing established basis in the partnership, but is not treated as an amount at risk. Remember that nonrecourse financing is debt that is secured by the property, but for which no individual has personal liability. LO 7.1.2

All of the following statements regarding the installment method of reporting gain from a disposition of property are correct except A) an installment sale is a sale of property in which the seller receives at least one payment after the year of sale. B) the payments received under an installment sale may each include capital gains, return of capital, and interest. C) the installment sale method may be used for securities sold in the secondary market. D) the installment method permits the seller to spread out the taxable gain over more than one year. Explanation

C) the installment sale method may be used for securities sold in the secondary market. The installment sale method cannot be used for inventory or securities traded in the secondary market. LO 6.2.4

Which of the following is NOT a step in the tax calculation process?

Calculate federal tax on total income. The following are involved in the income tax computation: subtracting adjustments to income from total income to get adjusted gross income, subtracting tax withholdings from total tax liability, and deducting the greater of itemized deductions or the standard deduction from AGI to arrive at taxable income. Credits are applied to tax liability. The calculation of federal tax is on federal taxable income.

Which of the following is CORRECT with respect to the alternative minimum tax? A)Net capital gains are subject to the AMT rates of 26% or 28%. B)All itemized deductions except home mortgage interest and property taxes are added back in the AMT computation. C)The exercise of an incentive stock option always causes an AMT preference item to be added into the AMT calculation. D)Charitable contributions, investment interest expense, and casualty losses are allowable itemized deductions for AMT purposes.

Charitable contributions, investment interest expense, and casualty losses are allowable itemized deductions for AMT purposes. Qualified housing interest, charitable contributions, investment interest expense, casualty losses, and gambling losses (to the extent of gambling winnings) are among the allowable itemized deductions for AMT purposes. State and local income taxes and property taxes are not allowed for AMT purposes. Net capital gains (by definition—long-term capital gains) and qualified dividends are subject to the preferential rates (15% or 20%) even for AMT purposes. The preference item for the bargain element on the exercise of an ISO may be avoided by utilizing a disqualifying disposition of stock acquired with the ISO. A disqualifying disposition is a sale of the stock in the same year as exercise of the option.

John is a bartender who makes a significant amount of his income from tips. During the current year, John intentionally underreported by 20% the amount of tips that he received. Which one of the following penalties, potentially applicable to John, would most likely apply? A)Late payment B)Underpayment of estimated tax C)Negligence D)Civil fraud

Civil fraud Negligence is the failure to make a reasonable attempt to comply with the provisions of the Code. The penalty for negligence is 20%. Civil fraud is knowing and showing intentional disregard of the Code and Regulations. The penalty for civil fraud is 75% of the amount underreported due to fraud by the taxpayer. Because the failure to report was intentional, the civil fraud penalty should apply.

Which of the following is necessary to include in a premarital agreement? A) Financial resources and net worth of the less wealthy party B) Intentions to facilitate a divorce C) Financial resources and net worth of the more wealthy party D) Financial resources and net worth of both parties

D) Financial resources and net worth of both parties To be valid, a premarital agreement must be in writing and contain a complete disclosure of each party's financial situation. It may not be used to regulate an award of alimony.

Which of the following statements regarding the use of life insurance inside a retirement plan is CORRECT? A) If the employee dies prematurely, the survivors will receive no benefits. B) The premiums paid are a taxable benefit to the employer. C) The premiums paid are NOT a taxable benefit to the employee. D) The premiums paid are a taxable benefit to the employee.

D) The premiums paid are a taxable benefit to the employee. The premiums paid are a taxable benefit to the employee. The main benefit to the employee is in the event of their premature demise, their survivors will still receive ample retirement benefits.

Ron, age 43, and Sandy, age 41, are married with two children: Michael, age 12, and Victoria, age 8, who has been blind since her birth. Ron is an architect and general partner with XYZ partnership. Sandy is self-employed as an attorney and works out of a home office. Her home office is exclusively and regularly used for business, and the home office is her principal place of business. Their information for the tax year 2021 is as follows: Adjusted gross income: $217,300 Itemized deductions (including qualified residential mortgage interest, taxes paid, and charitable contributions): $33,000 Early in the current year, Sandy's father died. Sandy is the sole beneficiary of her father's entire estate. The estate is presently in the probate process. Sandy's mother, Lisa, age 68, has moved in with them but provides her own support. She was married to Sandy's father when he died earlier this year. This is Ron's second marriage. He makes monthly support payments to his former spouse and his daughter. Because both Ron and Sandy are considered to be self-employed, they make quarterly estimated tax payments each year to cover both their income tax and self-employment tax obligations. Ron gave a painting to the local art museum in the current year. The painting had a fair market value of $34,000. He paid $16,500 for it five months ago. The museum will display the painting among its collection. Based on the information provided for Ron and Sandy, what is Ron's charitable contribution deduction? A)$13,800 B)$34,000 C)$23,000 D)$16,500

D) $16,500 Because the artwork is appreciated property held for less than one year, its sale would result in a short-term capital gain. Consequently, the deduction is limited to the lesser of the fair market value or the basis of the gifted property.

During the current year, Susan Snow received $8,000 of unemployment compensation, $4,500 of workers' compensation benefits, and $10,000 compensation for lost wages in the settlement of a lawsuit against her former employer. What amount, if any, must Susan report as income?

D) $18,000 The $8,000 of unemployment compensation and $10,000 compensation for lost wages in the settlement of a lawsuit against her former employer are included in income. Both are essentially a replacement for wages. The workers' compensation is nontaxable because it is received for injuries received on the job

Paul died recently. At the time of his death, he had $12,000 in suspended losses from a limited partnership, a passive activity. His basis in the limited partnership was zero, and the step-up in basis at death was $10,000. What is the amount of suspended losses that are deductible? A) $0 B) $12,000 C) $10,000 D) $2,000

D) $2,000 The answer is $2,000, the suspended losses ($12,000) minus the basis ($10,000).

Jane owns a printing business. She wants to trade her old copiers for new fax machines. In the contemplated exchange, Jane will pay $750 in cash. Additional information related to the transaction is given as follows: The copiers have an adjusted basis of $1,500. The copiers have a fair market value of $1,000. The fax machines have a fair market value of $1,750. What is Jane's recognized gain or loss in this exchange? A) $0 B) ($250) C) $500 D) ($500)

D) ($500) Jane is paying $750 plus the adjusted basis of $1,500 ($2,250); compared to the fair market value of the property received of $1,750, thus yielding a $500 loss. There is no loss recognized in a like-kind exchange. This exchange is simply treated as a sale of the asset. A loss on a Section 1231 asset may be recognized in the year of the loss. The Tax Cuts and Jobs Act (TCJA) restricted the like-kind exchange rules to real estate only. Personalty does not qualify for like-kind exchange treatment.

Kappa Corporation has the following items of income and expense: Taxable income: $310,000 Federal income tax paid: $80,000 Dividends paid: $20,000 Accumulated earnings and profits at the end of the preceding year: $90,000 Kappa is an engineering firm with 100% of the stock owned by its three employee-shareholders. The corporation cannot establish a valid business purpose for excess accumulations. How much accumulated earnings tax is payable by Kappa?

D) $30,000 The accumulated earnings tax applies to corporate accumulated earnings in excess of $250,000, generally, for which there is no valid business reason for accumulating the funds. For a personal service corporation (PSC), the accumulations limit is $150,000. Engineering is one of the personal services specifically listed for determining a PSC. Expenses otherwise not allowed in computing the corporate income tax are allowed in computing the accumulated earnings tax, as is summarized here. (Note that the accumulated earnings credit of $60,000 is the difference between the $150,000 safe harbor and the $90,000 of accumulations at the beginning of the year.) The accumulated earnings tax rate is 20%, the highest rate that may apply to qualified dividends. Taxable income$310,000Federal income tax(80,000)Dividends paid(20,000)$210,000Less: accumulated earnings credit(60,000)Excess accumulations$150,000× 20%$30,000

In February, Jeremy purchased a new computer (five-year property) for use in his business. The cost of the computer was $4,300, while freight and setup charges totaled $600. What is the first-year cost recovery deduction using the straight-line method? A)$430 B)$790 C)$860 D)$490

D) $490 The freight and setup charges of $600 must be capitalized (i.e., added to the cost of the computer) to give a total basis of $4,900. The straight-line rate for five-year property is 20% (100% divided by 5), but the half-year convention limits the deduction to 50% in the year of acquisition. Thus, $4,900 times 10% equals $490. Note that bonus depreciation is not used, as the fact pattern states to use straight-line. LO 6.1.3

On December 20, 2004, Jody moved into a condominium that she owns and had rented to tenants since July 1, 1997. Her cost basis in the condo was $238,440. Jody took depreciation deductions totaling $54,000 for the period that she rented the property. After moving in, she used the residence as her principal residence. Jody sells the property on August 1, 2021, for $538,000. Jody is in the highest marginal income tax bracket for the current year. What is the amount and character of the recognized gain resulting from the sale? A) $353,560 "regular" long-term capital gain B) $54,000 of ordinary income; $49,560 of "regular" long-term capital gain C) $54,000 of unrecaptured Section 1250 income; $299,560 of "regular" long-term capital gain D) $54,000 of unrecaptured Section 1250 income; $49,560 of "regular" long-term capital gain Explanation

D) $54,000 of unrecaptured Section 1250 income; $49,560 of "regular" long-term capital gain Jody's gain realized (the actual economic gain) from the sale is $353,560 ($538,000 of sales proceeds reduced by the adjusted basis of $184,400). Of this $353,560 of gain, the first $54,000 is recognized as unrecaptured Section 1250 gain, taxed at 25%. Unrecaptured Section 1250 gain is the gain created by the straight-line depreciation. This leaves $299,560 of gain to account for. Jody used the condo as her principal residence for two full years—thus, she is eligible to exclude $250,000 under Section 121. This leaves $49,560 of long-term capital gain to be recognized at a 20% rate (because she is in the highest marginal income tax bracket, her taxable income exceeds the $445,850 breakpoint for the 20% LTCG rate). The recognized gain is the gain on which Jody will pay taxes. Note that the nonqualified use provision does not come into play here as there was no nonqualified use after 2008. LO 6.2.5

What is Bobby's total self-employment tax on his Schedule C net income of $50,000? Round your answer to the nearest dollar. A) $3,825 B) $7,650 C) $3,533 D) $7,065

D) $7,065 Bobby's total self-employment tax is $7,065 ($50,000 × 0.9235 = $46,175; $46,175 × 0.1530 = $7,065).

Bob passed away during the current year. He had suspended losses from a limited partnership activity of $25,000. Bob's basis in the partnership was $1,000 and the fair market value at the time of his death was $18,000. What amount of passive losses, if any, is deductible on Bob's final income tax return? A) $14,000 B) $17,000 C) $0 D) $8,000

D) $8,000 The suspended passive losses are "freed up" and deductible only to the extent that the losses exceed the step-up in basis. In this situation, the step-up in basis equals $17,000 (from $1,000 to $18,000). The losses of $25,000 exceed the step-up amount by $8,000.

In the current year, George invested $100,000 for a 20% partnership interest in an activity in which he is a material participant. The partnership reported a loss of $400,000 in the current year and a loss of $200,000 in the following year. George's share of the partnership's loss was $80,000 in the current year and $40,000 in the following year. How much of the loss from the partnership, if any, can George deduct in the current and following year? A)$80,000 in the current year and $0 in the following year B)$80,000 in the current year and $40,000 in the following year C)$0 in both the current and the following year D)$80,000 in the current year and $20,000 in the following year

D) $80,000 in the current year and $20,000 in the following year

Paul and Mary form an equal partnership to produce hammers for the military. They each have a 50% profit and loss sharing agreement. Paul contributes cash of $50,000 and property with a fair market value of $50,000 and an adjusted basis of $20,000. Mary contributes cash of $25,000 and property with a fair market value of $75,000 and an adjusted basis of $40,000. A bank gives the partnership a recourse loan of $30,000 and a nonrecourse loan of $25,000. What is the amount of Paul and Mary's basis in the partnership, respectively? A)$127,500 for Paul, $127,500 for Mary B)$70,000 for Paul, $65,000 for Mary C)$85,000 for Paul, $80,000 for Mary D)$97,500 for Paul, $92,500 for Mary

D) $97,500 for Paul, $92,500 for Mary The basis of a partnership interest is measured by the amount of cash contributed, increased by the adjusted basis of property contributed, and further increased by the partner's share of both recourse and nonrecourse financing.

Mavis and Bennet are married and file jointly. Mavis earns $30,000 and Bennet earns $70,000. What percentage of the tax liability does each spouse owe? A)Bennet earns the most, so he owes 100% B)Depends on the percentage of income C)50% for each spouse D)100% for each spouse

D) 100% for each spouse Under the rule of joint and several liability, each spouse owes 100% of the tax liability, no matter who earned the income or in what proportion.

If Rachel files single with gross income of $90,000 and taxable income of $76,000, what is her effective tax rate? Refer to the 2021 tax table provided in your course references

D) 16.41% 22%($76,000 ‒ $40,526) + $4,664 = $7,804 + $4,664 = $12,468 ÷ $76,000 = 16.41%.

Three investors wish to start a manufacturing business. The business is expected to generate a large income, which it will reinvest for many years. John has substantial assets that he plans to contribute to the business. John is also concerned about showing too much business income on his personal return. Which business structure would be most appropriate for the business? A)A limited partnership with John as the limited partner B)An S corporation with all three as equal shareholders C)A business trust with all three as equal interests D)A C corporation with all three as shareholdersD) A C corporation with all three as shareholders

D) A C corporation with all three as shareholders A C corporation is the only listed entity that is not a conduit. Since John is concerned about showing too much business income on his personal return, a separate taxable entity is appropriate.

Erma, age 40, anticipates an adjusted gross income of $177,000 for the current tax year. All her income is attributable to active and portfolio income. She would like to acquire an investment that would reduce her tax liability without exposing her to personal liability. Which one of the following investments is the most appropriate for Erma? A)A master limited partnership that will produce passive losses B) An oil and gas working interest that will produce losses C) An "active participation" investment in rental real estate that will produce losses D)A historic rehabilitation real estate limited partnership that will produce rehabilitation tax credits

D) A historic rehabilitation real estate limited partnership that will produce rehabilitation tax credits Erma's AGI is too high to claim the active participation real estate exception. Losses from a master limited partnership can only be deducted against income generated by the same partnership in another tax year. The oil and gas working interest will generate unlimited liability. Thus by process of elimination, the correct answer is historic rehabilitation credit.

Which of the following may be allowed as a like-kind exchange? A)An apartment building located in San Diego exchanged for an apartment building located in Acapulco, Mexico B)Farmland exchanged for farming equipment C)A heifer exchanged for a bull D)A parking lot exchanged for a shopping center

D) A parking lot exchanged for a shopping center An exchange of U.S. realty for foreign realty is not considered like-kind. The like-kind requirements were changed under TCJA and now limits exchanges to realty for realty. The like-kind requirement does not mean that the property transferred must be identical to the property received; it merely requires that realty be exchanged for realty.

Amanda and Matt were divorced in January 2021 and have one 11-year-old son. The court ruled that Amanda must pay Matt $4,000 per month to cover both alimony and child support. The divorce decree states that 70% of each payment is allocated for child support, and these payments must be made for seven years. Based on this information, which of the following statements is CORRECT? A)Amanda can deduct $2,800 per month as child support. B)Amanda can deduct the $4,000 per month as child support and alimony payment. C)Amanda can deduct $1,200 per month as alimony. D)Amanda cannot deduct the alimony or the child support she pays Matt.

D) Amanda cannot deduct the alimony or the child support she pays Matt. Child support is not deductible, and, beginning in 2019, alimony payments are no longer deductible by the payor.

Lauren owns a vacation home that she also rents to others during the year. The house was rented to unrelated parties for 11 full weeks during the current year. Lauren used the house 16 days for her vacation during the year. After properly dividing the expenses between rental and personal use, it was determined that a loss was incurred as follows: Gross rental income$6,400Less:Allocated mortgage interest and property taxes($7,000)Other allocated expenses($1,000)Net rental loss($1,600) Which of the following statements regarding the treatment of the rental income and expenses on Lauren's federal income tax return for the current year is CORRECT? A)A $1,600 loss should be reported. B)The $7,000 rental portion of mortgage interest and taxes cannot be deducted. C)Because the house was used only 20.8% personally by Lauren, all expenses allocated to personal use may be deducted. D)Because Lauren used the house for 16 days, the mortgage interest and property taxes are deductible as a rental expense to the extent of the gross rental income of the property.

D) Because Lauren used the house for 16 days, the mortgage interest and property taxes are deductible as a rental expense to the extent of the gross rental income of the property. A vacation home can be classified as a personal residence, a rental property, or a mixed-use property. A personal residence is a property that is rented out for less than 15 days per year. Because the beach house was rented for 11 weeks, it will be classified as either a rental property or a mixed-use property. The determination is based on the number of days the taxpayer used the residence for personal use. To qualify as rental property, the personal use cannot exceed the greater of 14 days per year or 10% of rental days. The property was used for personal use for 16 days, so the vacation home will be considered a mixed-use property. Expenses incurred on a mixed-use property must be allocated between rental use and personal use. The rental expenses on a mixed-use property are only deductible to the extent of rental income received (i.e., the taxpayer cannot claim a loss). The expenses not deducted in the current year may be carried forward.

Which one of the following is NOT a federal taxation function of the economic objective? A) Restricting spending through greater taxation B) Reduction of taxes during a recession to stimulate the economy C) Promoting full employment D) Charitable deduction

D) Charitable deduction The reduction of taxes in order to stimulate the economy is due to the economic objective, as are restricting spending and promoting full employment. Social objectives of the federal taxation system include the charitable deduction, excluding life insurance proceeds from taxation, and renovation of a historic home.

Danielle created a revocable trust for her two minor sons. She named her bank as trustee. The trust property earned $30,000 in the first year and had taxable income of $28,000 after deducting expenses. This income was left to accumulate for future distributions to be made to each son equally when the youngest son attains age 18. To which of the following will the income of the trust be taxable? A) The oldest son after attaining age 18, then the sons equally after the youngest son attains age 18 B) Both sons equally C) The trust D) Danielle

D) Danielle The trust income will be taxed to the grantor, as the trust is revocable. A revocable trust is treated as a grantor trust.

Which of the following is CORRECT regarding the LIFO method of accounting for inventory? A)During periods of declining inventory prices, the cost of goods sold (COGS) will be higher. B)During periods of declining inventory prices, lower taxable income will result. C)During periods of increasing inventory prices, higher taxable income will result. D)During periods of increasing inventory prices, lower taxable income will result.

D) During periods of increasing inventory prices, lower taxable income will result. Under the LIFO method, when prices are increasing, COGS will be higher; thus, taxable income will be lower.

During 2021, Judy, a sole proprietor, purchased new equipment (seven-year property) for her manufacturing business at a cost of $600,000. Judy is in a 12% marginal income tax bracket this year, and expects to be in that bracket for two more years. She is extremely confident that she will be in the highest marginal bracket after that. What advice would you give Judy regarding the use of bonus depreciation and cost recovery deductions? A) Use the maximum bonus depreciation and use the Modified Accelerated Cost Recovery System (MACRS) table. B) Use the maximum bonus depreciation and elect the straight-line method. C) Forgo bonus depreciation and use the Modified Accelerated Cost Recovery System (MACRS) table. D) Forgo bonus depreciation and elect the straight-line method.

D) Forgo bonus depreciation and elect the straight-line method. The fact pattern indicates that Judy is in the lowest marginal bracket for three years, and will be in the highest marginal bracket after that. It makes no sense to maximize the depreciation deduction in years when Judy is in the lowest marginal brackets. By forgoing bonus depreciation and using straight-line, more deductions are pushed into the last five years of the depreciation schedule, when Judy will be in the highest marginal bracket. Remember that because of the half-year convention, seven-year property is depreciated over eight years. Under TCJA, 100% bonus depreciation is allowed for all personalty. In other words, 100% of the cost is deducted in the first year. LO 6.1.3

Max has taxable income of $80,000 from wages and will list his mother as a dependent on his tax return. Max will take the standard deduction this year. He wants to file on as simple an income tax form as he can. Which form will fit Max the best? A)Form 1040NR B)Form 990 C)Form 709 D)Form 1040

D) Form 1040 Max can use the 1040. 990 is for nonprofits. 709 is for gift tax. 1040NR is for nonresident aliens.

Which of the following forms does an S corporation file? A)Form 1120-C B)Form 1065 C)Form 1120 D)Form 1120-S

D) Form 1120-S The S corporation files an information return with the IRS—this is the Form 1120-S. The S corporation provides a K-1 to each shareholder to show the flow-through of income, deductions, and credits.

Which of the following statements regarding home equity loan interest paid by a couple who files an income tax return as married filing jointly is NOT correct?

D) Home equity loan interest is not deductible.

Michael was divorced after 12 years of marriage and recently died. He had two dependent children, ages four and six, who are cared for by their mother. He was currently, but not fully, insured under Social Security at the time of his death. The benefits that his survivors are entitled to include which of the following? A lump sum death benefit of $255 A children's benefit A divorced spouse's benefit A parent's benefit for deceased workers' parents who are over the age of 62 A)III and IV B)I, II, and III C)I, II, III, and IV D)I and II

D) I and II A lump sum death benefit of $255 is payable to the surviving spouse or children of the deceased worker if he was fully or currently insured. The children's benefit is payable because Michael was either currently or fully insured. The mother of the children would be entitled to a benefit because she is caring for Michael's children who are under the age of 16. However, that is not called the divorced spouse benefit. It is the surviving spouse caring for a dependent child benefit. The parents are not entitled because Michael was not fully insured. Statements III and IV are benefits only available under fully insured status.

When filing as married filing separately, which of the following are true? If one spouse itemizes deductions, the other spouse must also itemize. The couple avoids joint and several liability for the other spouse's return. Married filing separately is usually more advantageous for a couple than married filing jointly. A)III only B)I, II, and III C)II only D)I and II

D) I and II As an alternative to filing jointly, married individuals may file separate returns. This is known as married filing separately (MFS), and, in a few cases, the individuals may derive some tax benefit. However, the filing of returns MFS is usually not advantageous for married individuals.

Which of the following must be true for someone to be claimed as a dependent for another taxpayer? A dependent may not have more than $4,300 (2021) of gross income. The taxpayer must provide over 75% of the dependent's support. A person who dies during the year may be identified as a dependent. Social Security payments are always included in the dependent's gross income. A) III and IV B) II and IV C) I and II D) I and III

D) I and III A dependent may not have more than $4,300 (2021) of gross income. Social Security income is excluded from the test if that is the elder's only source of income. The taxpayer must also provide over 50% of the dependent's support to claim them. Coincidentally, as long as all the tests are met, a person who dies during the year may still be identified as a dependent.

In 2021, which of the following is a method of calculating total self-employment tax where net income from self-employment is below the taxable wage base? Calculate self-employment income; multiply by 0.9235 (1 − 0.0765); and multiply the result by 15.3%. Calculate self-employment income; multiply by 15.3%; and subtract 7.65%. Calculate self-employment income; subtract 7.65%; and multiply the result by 15.3%. Calculate self-employment income; multiply by 15.3%. A)II and IV B)I and II C)III only D)I and III

D) I and III to calculate the self-employment tax in 2021 where net income from self-employment is below the taxable wage base, use the following steps: Step 1: Calculate self-employment income. Step 2: Subtract 7.65% or multiply by 0.9235 (1 − 0.0765). Step 3: Multiply the resulting product by 15.3%. LO 5.3.1

Which of the following taxpayers may use the married filing jointly filing status? A married couple, even though one spouse did not have any income during the tax year A married couple that is legally separated on the last day of the tax year if they share custody of a dependent child A married couple that is legally separated on the last day of the tax year A married couple that is not legally separated on the last day of the tax year A) I, II, and IV B) II and IV C) I, II, and III D) I and IV

D) I and IV A married couple may file a joint return even though one spouse has no income or deductions if they are not legally separated or divorced on the last day of the tax year.

Which of the following taxpayers may use the married filing jointly filing status? A married couple, even though one spouse did not have any income during the tax year A married couple that is legally separated on the last day of the tax year if they share custody of a dependent child A married couple that is legally separated on the last day of the tax year A married couple that is not legally separated on the last day of the tax year A)II and IV B)I, II, and IV C)I, II, and III D)I and IV

D) I and IV A married couple may file a joint return even though one spouse has no income or deductions if they are not legally separated or divorced on the last day of the tax year.

Your client, Joe, has active income of $300,000 per year and substantial unused passive losses from a nonpublicly traded limited partnership. He would like to find an investment that would allow him to utilize his passive losses. Which of the following are the most appropriate investments for Joe? An active participation rental real estate activity generating income A master limited partnership (MLP) generating income Certificates of deposit generating portfolio income A nonpublicly traded limited partnership generating income A) III and IV B) I, II, and IV C) I and II D) I and IV

D) I and IV Income from active participation rental activities is considered passive income. The nonpublicly traded limited partnership losses may not offset income from a publicly traded partnership (the MLP) or portfolio income. LO 7.2.2

Which of the following statements regarding passive activity losses is CORRECT? When determining the amount of suspended loss that may be used against income, the at-risk rules are applied before the passive activity loss rules. If a loss is not allowed because of the at-risk limitations, the loss is a suspended loss eligible for deduction as a disposition of a passive activity. A)Both I and II B)Neither I nor II C)II only D)I only

D) I only Statement I is correct. Statement II is incorrect. If a loss is not allowed because of the at-risk limitations, the loss is a suspended loss and is not eligible for deduction as a disposition of a passive activity.

Your clients, John and Mary Voight, spoke recently to their insurance agent regarding the purchase of a single premium whole life policy. The agent indicated that the policy would be a modified endowment contract. The Voights were unsure what that meant. Which of the following describe a modified endowment contract? Meets the requirements of a life insurance contract under state law Was entered into (or substantially modified) on or after June 21, 1988 Fails to meet the "seven pay test" Meets the guideline premium and corridor test or the cash value accumulation test A)II and III B)II, III, and IV C)I, II, and III D)I, II, III, and IV

D) I, II, III, and IV Statements I through IV are all elements of the IRC definition of a modified endowment contract. The contract fails to meet the seven-pay test if the cumulative amount paid under the contract at any time in the first seven years is greater than the seven net level annual premiums that would have been paid under a seven-pay, paid-up contract.

Your clients, Joseph and Jane, have read many articles in financial publications about the alternative minimum tax (AMT) and are concerned that some of their investments and activities may cause AMT problems. Which of the following are preference items or adjustments for purposes of the individual AMT? Interest from qualified private-activity municipal bonds issued in 2008 Bargain element on the exercise of an incentive stock option Excess of percentage depletion over the property's adjusted basis Cost depletion deductions A) I and IV B) II, III, and IV C) I, II, III, and IV D) I, II, and III

D) I, II, and III By definition, the only listed item that is not an AMT preference item or adjustment is the cost depletion deduction. Note that interest on private-activity municipal bonds issued in 2009 and 2010 is not a preference item for the AMT.

Which of the following apply to the passive activity loss rules? Deductible passive losses are limited to the passive gains in other passive activities. Any unused passive losses may be carried forward against future passive gains. When the passive activity property is disposed of, any unused passive losses can be deducted against passive gains, portfolio, or active income. Passive loss rules apply only to real estate transactions. A) I and III B) II, III, and IV C) II and IV D) I, II, and III

D) I, II, and III Passive loss rules apply to all passive activities, not just real estate transactions. Passive losses may be deducted against passive gains. If the investor has excess passive losses, the losses are carried forward and may be used in future years to offset future passive gains. LO 7.1.1

Which of the following are allowable itemized deductions for purposes of computing the alternative minimum tax? Charitable deductions Qualified housing interest Gambling losses to the extent of gambling winnings Property taxes A) II, III, and IV B) I and III C) I and II D) I, II, and III

D) I, II, and III Statement IV, property taxes, is the only itemized deduction listed that is not allowed for AMT purposes. State and local income taxes are also disallowed.

Carol owns and operates a retail electronics store with annual sales of approximately $2 million. The store also specializes in repairing computers and other small electronics. Approximately 30% of her sales are with extended credit terms. What method of tax accounting may be used for Carol's business? The accrual method, because inventory is such a large component of the business The cash method, because it provides flexibility in the timing of income and expenses The hybrid method, because the business involves both inventory and service The installment sale method, to spread the gain over a longer time frame A) III only B) I, II, III, and IV C) I only D) I, II, and III

D) I, II, and III The cash method may be used because average annual gross receipts are under $26 million for the prior tax years. (This limit was increased by the Tax Cuts and Jobs Act, or TCJA.) The accrual method may always be used. The hybrid method may be used because the business involves both inventory and service. The installment method may not be used, as dealers are not allowed to use the installment method.

Which of the following business forms may be tax conduits? General partnership General partnership Limited partnership Publicly traded C corporation S corporation A)II, III, and IV B)I, II, and III C)I and IV D)I, II, and IV

D) I, II, and IV General partnerships, limited partnerships, and S corporations may all be tax conduits.

To which of the following do the passive activity loss rules apply? Individuals C corporations that are not closely held Closely held C corporations Estates A) I only B) I and III C) II and IV D) I, III, and IV

D) I, III, and IV Of the listed choices, option II is the only option to which the passive activity loss rules do not apply. The passive activity loss rules specifically do not apply to C corporations that are not closely held.

What important information could you tell your nonresident client (married to a U.S. citizen) about electing to be treated as a resident alien for tax purposes? You will need to pay taxes only on your U.S. income. You will need to pay taxes on your worldwide income. This will create an immigration benefit. You should obtain an individual taxpayer identification number (ITIN) or Social Security number.

D) II and IV If married to a U.S. citizen or resident alien, the nonresident alien can elect to be treated as a resident alien for tax purposes only. Tax status does not necessarily reflect immigration status. If this election is made, the couple must pay U.S. taxes on their worldwide income. In this situation, the nonresident alien spouse should obtain an Individual Tax Identification Number (ITIN). Depending on their individual situation and intentions, they may apply for a Social Security number with the Social Security Administration.

Under the passive activity loss rules, passive losses can be deducted against which of the following? Active income Passive income Portfolio income A)III only B)I, II, and III C)I and III D)II only

D) II only Statement II is correct. Statements I and III are incorrect. Passive losses cannot be deducted against either active income or portfolio income.

Which of the following activities are considered passive activities? A real estate broker spending 1,500 hours on her real estate activities, this being 80% of her personal services for the year A taxpayer who owns an apartment building using a property management to assist him in managing the property A taxpayer investing in a real estate limited partnership (RELP) A taxpayer owning 8% of an inherited condo that is rented to others during the year A) I, II, III, and IV B) I only C) II and III D) II, III, and IV

D) II, III, and IV Statements II, III, and IV are correct. Only the real estate professional's activities are not considered passive. In statement IV, the taxpayer owns less than 10% of the activity, so it is considered passive. This taxpayer is barred from using the real estate exception for losses because the taxpayer owns less than 10% of the activity.

Which of the following are deductions from AGI? Qualifying alimony paid to a former spouse Child support paid to a former spouse Investment interest expense incurred by an individual State and local income taxes paid A)I, II, and III B)II and IV C)III only D)III and IV

D) III and IV Child support is not deductible. Alimony, if it meets certain tests, is an above the line deduction for AGI. Investment interest expense and State and Local Taxes are below the line deductions from AGI.

Which one of the following is a characteristic of a fixed annuity contract? A) Fixed annuity contracts are not tax advantaged, unlike other annuity contracts. B) The buyer may choose among a handful of investment options. C) The annuitant pays now for future fixed or variable payments. D) If a corporation owns the annuity contract, the earnings are not tax deferred

D) If a corporation owns the annuity contract, the earnings are not tax deferred. With a fixed annuity contract, there is no ability to select the investment options; the payments are fixed. Fixed annuity contracts are generally tax advantaged (tax deferred), unless a corporation owns the annuity contract, in which case the earnings are currently taxable. Such is also the case with a variable annuity.

As part of their divorce, Phil must pay Jane $10,000 on January 1 for five years in a property settlement agreement. Phil offers to pay her a lump sum today of $40,000. Assume a discount rate of 10%. Assume all payments are to be made at the beginning of each year. Which of the following is true? A)Jane should demand Phil increase his payments to account for income tax. B)Jane should make a counteroffer and ask Phil to simply surrender his sports sedan, which originally cost $50,000 five years ago. C)Jane should accept this offer. D)Jane should reject this offer.

D) Jane should reject this offer. The present value of the installment option is $41,699. Jane would therefore be better off accepting the payments over the next five years. As a wasting asset, it is highly unlikely Jane could sell the car for its original fair market value of $50,000. Property settlements incident to a divorce are not subject to income tax. The 10BII+ calculator keystrokes are [Begin Mode], PMT = $10,000, I = 10%, N = 5, PV = $41,699. LO 4.2.2

Alex established a 2503(c) trust for his daughter, Julie, when she entered college four years ago. Alex decided to name his attorney as trustee and give Julie the right to revoke the trust at age 23, when she finished college. Julie did not revoke the trust and chose to allow the trust to continue until she is age 30. Which of the following correctly identifies the taxpayer, if any, who must pay tax on the trust income? A) The attorney as trustee B) Alex, because this is required by law C) The trust, because it is irrevocable and a separate taxable entity D) Julie, because she allowed the trust to continue past age 23 Explanation Because Julie waited past age 23 when she had the right to revoke the trust, she is responsible for taxes on the trust given to her. LO 3.1.1

D) Julie, because she allowed the trust to continue past age 23 Because Julie waited past age 23 when she had the right to revoke the trust, she is responsible for taxes on the trust given to her.

Jason and Sarah are accountants and are considering starting a business together. They will both be general partners. Jason and Sarah prefer the partnership form but are concerned about liability issues. Specifically, neither wants to be liable for the acts of the other. Which of the following forms would best suit Jason and Sarah? A)S corporation B)Limited partnership C)General partnership D)Limited liability partnership

D) Limited liability partnership In an LLP, the general partners are not liable for the acts of other partners. In addition, some states will protect the general partners from the claims that arise from obligations of the partnership, but this typically extends only to claims arising out of tort law, not contract law.

Alex owns a vacation home that he rents, on average, 98 days each year. Alex stays in the home for 6 weeks during the fall every year. Which category does this vacation home fall under? A)Rental use B)Because Alex stays in the home for 6 weeks in each year, the home is classified as his primary residence C)Primarily personal use D)Mixed use

D) Mixed use Explanation A vacation home is classified as mixed use if the vacation home is rented for at least 15 days a year and it is also used for personal use for more than the greater of 14 days per year or 10% of the rental days. LO 7.2.4

effrey and Karen have given cash gifts to their children over the years. In addition, in 2021 Mark, age 13, earns $2,500 in salary. Jennifer, age 19, who attends community college for approximately three months per year, earns $2,200 in dividends and capital gains. Nancy, age 12, earns $2,950 in dividends and interest. Steven, age 10, earns $900 in dividends and interest. Whose income is subject to the tax at the parents' marginal rate? A) Jennifer's and Nancy's B) Nancy's and Mark's C) Steven's D) Nancy's

D) Nancy's Nancy is the only child up to and including age 18 with unearned income in excess of $2,200 for 2021. Earned income is not subject to taxation at the parental rate. Jennifer is not subject because she is not a full-time student. The kiddie tax applies to children under 19 years of age. It also applies to children under age 24 if they are full-time students. The kiddie tax does not apply if the child's earned income exceeds one-half of the child's support. A full-time student is an individual who is a full-time student for at least five calendar months during the tax year.

Which of the following statements regarding replacement property in a Section 1033 exchange is CORRECT? In the taxpayer use test, the taxpayer's use of the replacement property and of the involuntarily converted property must be the same. For the functional use test, the owner-investor's properties must be used in similar endeavors as the previously held properties. A)I only B)Both I and II C)II only D)Neither I nor II

D) Neither I nor II Neither statement is correct. In the functional use test, the taxpayer's use of the replacement property and of the involuntarily converted property must be the same. In the taxpayer use test, the owner-investor's properties must be used in similar endeavors as the previously held properties.

What could you advise a client, concerning taxation, if a premarital agreement initiates a transfer of funds? You would be better off with a transfer for consideration, which would likely not be taxed. If a gift transfer is made, one spouse would likely have considerable income added. A)Both I and II B)II only C)I only D)Neither I nor II

D) Neither I nor II The income tax consequences of the premarital agreement depend in large part upon whether the transfer under the agreement is treated as a gift (where income tax is avoided) or as a transfer for consideration (which will probably result in the recognition of significant income by one party). LO 4.1.2

This year, Ken sold several securities that left him with the following types of gains and losses: Long-term capital gain: $8,000 Short-term capital gain: $1,800 Long-term capital loss: $2,200 Short-term capital loss: $1,000 What is the net capital gain or loss on Ken's security sales? A)Net long-term loss of $1,400 B)Net long-term gain of $2,320 and net short-term gain of $800 C)Net long-term gain of $2,640 D)Net long-term gain of $5,800 and net short-term gain of $800

D) Net long-term gain of $5,800 and net short-term gain of $800 The long-term gain and loss are netted, leaving a long-term gain of $5,800. Short-term gains and losses are netted, leaving a short-term gain of $800. These are left separate due to the disparate tax treatment of short-term versus long-term gains.

Ken Brandt (a single taxpayer), age 28, holds the following securities. StockPurchase DateFair Market ValueCost BasisABC (300 shares)October 3, 2003$12,200$5,500DEF (500 shares)February 15, 2021$22,600$15,600GHI (100 shares)June 2, 2006$4,350$6,250LMN (700 shares)December 9, 2020$19,360$28,560XYZ small-cap fund (500 shares)October 20, 2011$1,200$3,700VWL (750 shares, §1244 stock)July 17, 2005$4,050$115,600 BondsPurchase DateFair Market ValueCost BasisEE savings bondsMay 1, 2012$8,500$6,000 If Ken sells the VWL stock at its fair market value, what is the result? A)The $111,500 is an ordinary loss, fully deductible against other income. B)The $111,550 loss is a long-term capital loss, deductible up to $3,000 per year. C)Of the $111,550 loss, $100,000 is an ordinary loss, and $11,500 is a long-term capital loss. D)Of the $111,550 loss, $50,000 is an ordinary loss, and $61,550 is a long-term capital loss.

D) Of the $111,550 loss, $50,000 is an ordinary loss, and $61,550 is a long-term capital loss. The annual ordinary loss on a sale of Section 1244 stock is limited to $50,000 for single taxpayers or for married taxpayers filing separately. On a joint return, $100,000 of ordinary loss may be deducted. The remainder of the loss (the excess over the ordinary loss) is treated as a capital loss (long term, in this situation).

Your client is contemplating the exchange of two parcels of investment land for two similar parcels. Given the following details of the proposed transactions, compute the amount of recognized gain and loss, if any, on both parcels if your client does the exchanges. Parcel A: There were 10 acres of land acquired 15 years ago with a current basis of $50,000. In exchange, your client will receive 8 acres of land (FMV $80,000) and $20,000 of cash. Parcel B: There were 20 acres of land acquired 2 years ago with a current basis of $100,000. In exchange, your client will receive 12 acres of land (FMV $75,000) and $10,000 of cash. A) Parcel A recognized gain: $20,000; Parcel B recognized loss: $15,000 B) Parcel A recognized gain: $50,000; Parcel B recognized loss: $10,000 C) Parcel A recognized gain: $20,000; Parcel B recognized loss: $10,000 D) Parcel A recognized gain: $20,000; Parcel B recognized loss: $0

D) Parcel A recognized gain: $20,000; Parcel B recognized loss: $0 The realized gain in Parcel A is $50,000 and the recognized gain (the lesser of the gain realized or the boot received) is $20,000. The realized loss in Parcel B is $15,000. However, there is no loss recognized (deducted) in a like-kind exchange.

Which of the following statements concerning alimony is CORRECT? A) Payments made with respect to jointly owned property are considered full alimony. B) No payments except cash can be considered alimony. C) Cash payment of the payee spouse's mortgage made by the payor spouse as required by the divorce or separation instrument qualify as one-half alimony. D) Payments to maintain property used by the payee spouse but owned by the payor spouse do not qualify as alimony.

D) Payments to maintain property used by the payee spouse but owned by the payor spouse do not qualify as alimony. Cash payment of the payee spouse's mortgage, rent, tuition, or tax liability made by the payor spouse as required by the divorce or separation instrument may qualify as alimony. Payments to maintain property used by the payee spouse, but owned by the payor spouse, do not qualify as alimony, even if required under the instrument. Payments made with respect to jointly owned property are considered one-half alimony. These property-related expenditures may include mortgage payments, real estate taxes, and homeowners insurance.

Which one of the following is NOT subject to the Medicare contribution tax? A) Long-term capital gains B) Qualified dividends C) Income from a nonperiodic distribution from an annuity D) Qualified Roth distributions

D) Qualified Roth distributions Qualified Roth distributions are not subject to the Medicare contribution tax. Only taxable items, such as net capital gains, net rental income, annuity income and dividends, for example, are subject to the Medicare contribution tax.

Ruth and Doug divorced last year. They have two children ages seven and nine. Their divorce decree states that Ruth has custody of both children. There is no written agreement for listing the children as dependents on Ruth's or Doug's income tax returns. However, Doug provides 75% of the child support, amounting to $15,000 per year. Based on this information, which parent is entitled to list the children as dependents for income tax purposes? A) Doug, because he provides over one-half of the child support B) Ruth, because the court awarded her custody of the children C) Doug, because he provides at least $1,200 per year for the children's support and Ruth cannot not prove she contributes more than this amount D) Ruth, because she has custody and there is no written agreement stating Doug could list the children on his return

D) Ruth, because she has custody and there is no written agreement stating Doug could list the children on his return The parent with custody for a greater portion of the year is treated as providing more than one-half of a child's support. In these circumstances, however, Ruth could potentially sign IRS Form 8332 which would constitute a written agreement that would allow Doug to claim their two children as dependents on his tax return in the current year.

Which of the following statements about S corporations is CORRECT? A) S corporations may have nonresident aliens as shareholders. B) S corporation status is automatic if there are fewer than 100 shareholders. C) S corporations are prohibited from earning passive income. D) S corporations are prohibited from having more than one class of stock.

D) S corporations are prohibited from having more than one class of stock. An S corporation may have only one class of stock, although differences in voting rights are allowed within that one class of stock. An S corporation may have no more than 100 shareholders; however, the corporation must elect S status by filing a Form 2553—the treatment is not automatic. An S corporation may have passive income, although excess passive income may trigger the sting tax if the corporation had been a C corporation with earnings & profits (E&P). Nonresident aliens may not be shareholders in an S corporation.

Which one of the following is NOT a main source of federal tax revenue? A)Corporate income taxation B)Payroll taxation C)Individual income taxation D)Sales taxation

D) Sales taxation The three main sources of federal tax revenue are individual income taxes, corporate income taxes, and payroll taxes; sales taxation is used by states, not the federal government.

Which of the following married couples may file their federal income tax return using the married filing jointly (MFJ) status? A) Terry and Edie divorced last year but did not move into separate homes until November of this year. B) Mark and Beth are both self-employed and have different fiscal years to accommodate their businesses. C) Paul and Josie will be married next year on New Year's Day. D) Sara has income for the tax year but Jack does not.

D) Sara has income for the tax year but Jack does not.] Because Mark and Beth have different fiscal years to accommodate their businesses, their tax years do not begin on the same date, and they may not file as MFJ. In order to use the MFJ status, a couple must be legally married on the final day of the tax year.

Which one of the following may enable a direct participation program to provide specific tax advantages to the investors? A)At-risk rules B)Passive activity loss rules C)Partnership basis rules D)Special allocations

D) Special allocations

Which of the following is NOT a step in the tax calculation process?

D) Subtract exclusions from AGI. The following are involved in the income tax computation: subtracting adjustments to income from total income to get AGI, and deducting the greater of itemized deductions or the standard deduction from AGI to arrive at taxable income. Subtracting exclusions from AGI is not a step in the tax calculation process. Excluded amounts simply do not show up as income on the return

Which one of the following is NOT a main source of federal tax revenue? A) Corporate income taxation B) Individual income taxation C) Payroll taxation D) Taxation of taxpayers living abroad

D) Taxation of taxpayers living abroad The three main sources of federal tax revenue are individual income taxes, corporate income taxes, and payroll taxes; taxation of taxpayers living abroad is not a main source of federal tax revenue. LO 8.1.2

Jean's mother dies and leaves her house to Jean this year. The house is valued at $40,000. Jean rents the house to tenants for $1,000 per month following her mother's death. What amount must Jean include in her annual gross income for this year? A) $40,000 plus the amount of rental income she receives B) The lesser of the FMV of the house or the amount of rental income she receives C) $40,000 D) The amount of rental income she receives

D) The amount of rental income she receives Jean receives the house on a tax-free basis because inheritances and gifts are not taxable to the recipient. However, any income generated by the house, such as rental income, is subject to taxation.

If a vacation home is rented for 14 days or less during the year, which one of the following statements is CORRECT? A) Typically, only a small amount of cost recovery deductions is allowed for the year. B) Repair expenses attributed to the rental activity are deductible. C) A portion of the rental income may be nontaxable. D) The full amount of home mortgage interest is permitted as an itemized deduction.

D) The full amount of home mortgage interest is permitted as an itemized deduction. If property is rented fewer than 15 days per year, the full amount of home mortgage interest, taxes, and casualty losses are permitted as an itemized deduction (not expenses, though); in addition, rental income can be excluded from gross income.

Which of the following statements regarding S corporations is CORRECT? A)Any individual, estate, corporation, or trust may be an S corporation shareholder. B)An S corporation must pay self-employment tax on its ordinary income. C)An S corporation may have more than one class of stock. D)The maximum number of shareholders an S corporation may have is 100.

D) The maximum number of shareholders an S corporation may have is 100. Only certain types of trusts, individuals, and estates may be shareholders of an S corporation. They are limited to one class of stock, and neither the corporation nor its owners pay self-employment tax. The maximum number of shareholders is 100.

On April 1 of the current tax year, Susan sold her principal residence for a total price of $501,000; $301,000 was in cash, with the buyer assuming a $200,000 mortgage on the house. Susan purchased the house 15 years ago for $290,000, but she has an adjusted basis of $80,000 due to a Section 1034 rollover. She has not made any improvements to the house. To assist in the sale of the residence, she incurred costs of $1,500 for repairs three weeks before the sale occurred. Realtor commissions of $31,000 resulted from the sale. On May 1 of the current tax year, Susan bought a new residence for $260,000. Assume that Susan is considering renting out her new residence for two weeks (14 days) during the upcoming tax year. However, she is unsure of the income tax consequences. Which one of the following statements is CORRECT? A) The rental income may or may not be includible in income, depending on the amount. B) The rental income is includible in full in gross income. C) The rental income is includible in income, but mortgage interest and property taxes allocable to the rental are deductible for AGI. D) The rental income is not includible in income.

D) The rental income is not includible in income. Income from rentals for fewer than 15 days during the year are not required to be included in gross income. However, no deductions attributable solely to the rental are allowed, either. The home mortgage interest and property taxes are still deductible in full as itemized deductions.

Which one of the following statements correctly describes the income tax consequences upon the disposition of a passive activity? A)If a taxpayer sells 20% of the interest in an activity, then 20% of the suspended losses are deductible against any other income. B)The suspended losses are deductible in full if more than 50% of the interest in the activity is sold. C)The suspended losses are nondeductible. D)The suspended losses are deductible against any income upon the sale of the entire interest.

D) The suspended losses are deductible against any income upon the sale of the entire interest. The suspended losses are deductible against any income upon the sale of the entire interest.

A taxpayer currently is being audited by the IRS, and the agent has proposed a tax deficiency with which the taxpayer does not agree. The client has asked you to research the issue. Which one of the following sources is considered to be the most authoritative and, accordingly, would have the highest precedential value in defending the taxpayer's position to the IRS? A) Revenue Ruling B) Private Letter Ruling C) Revenue Procedure D) Treasury Regulations

D) Treasury Regulations Treasury Regulations have the full force and effect of law. A Private Letter Ruling is never precedential, and Revenue Rulings and Revenue Procedures are merely administrative interpretations of the statutory tax law with lower authority than Regulations.

When does the alternative minimum tax (AMT) apply to a taxpayer? A) When the taxpayer has losses from passive activities or other business functions B) When the taxpayer has not withheld or prepaid a sufficient proportion of actual income tax liability C) Anytime a person is involved in passive activities D) When the AMT calculation results in a tax liability that is greater than that resulting from the regular income tax calculation

D) When the AMT calculation results in a tax liability that is greater than that resulting from the regular income tax calculation The AMT is paid when the AMT tax calculation produces a higher amount of tax than the regular tax calculation and the AMT payable is the difference between the two.

The marginal tax rate is obtained by

D) finding the tax bracket of the taxable income amount. The marginal tax rate is found by finding the tax bracket that contains the taxable income amount; it is the amount at which all subsequent taxable amounts will be taxed (until entering the next tax bracket). The effective tax rate is calculated by dividing the calculated tax by taxable income.

All of the following are examples of self-employment income for purposes of the self-employment tax except A) net Schedule C income. B) income from a sole proprietorship. C) net Schedule F income. D) salary paid to an S corporation shareholder.

D) salary paid to an S corporation shareholder. Wages and salary paid to an S corporation shareholder are not self-employment income.

Marvin has all of the following items. All of them are AMT preference items except A) exclusion of gain on the sale of certain qualified small business corporation stock. B) percentage depletion in excess of adjusted basis on a mining property. C) tax-exempt interest on certain private-activity bonds. D) tax-exempt income from a State of Iowa municipal revenue bond.

D) tax-exempt income from a State of Iowa municipal revenue bond. Tax-exempt income from a municipal revenue bond is not a preference for AMT. All of the other items are preferences for AMT purposes.

In November 2019, George's rental real estate was condemned by the city under the eminent domain statute to allow for a new overpass to be built. He received the condemnation payment from the city later that month. If there were a gain on the conversion, when would the replacement period end? A)November 30, 2021 B)November 30, 2022 C)December 31, 2022 D)December 31, 2021

December 31, 2022 The replacement period for condemned real estate used in a trade or business or held for investment purposes ends on the last day of the third taxable year following the year in which any part of the gain on the condemnation is realized.

According to the Internal Revenue Code, which of the following statements regarding gross income is CORRECT?

Gross income consists of all income except for those items that are specifically excluded by the Internal Revenue Code.

Jacob has an apartment building in Atlanta that he would like to exchange. Which of the following assets could Jacob receive in a like-kind exchange? Farmland Interest in a low-income housing limited partnership Parking lot An apartment building in Tahiti

I and III In a like-kind exchange, only real estate may be exchanged for real estate. The like-kind exchange rules specifically prohibit the exchange of U.S. realty for foreign realty.

Which of the following is a requirement for the IRS to assure protection of taxpayer rights? The IRS must disclose the criteria it uses in selecting returns for audit. The IRS must take steps to protect the confidentiality of personal and financial information. The IRS must follow due process when imposing tax levies. The IRS must provide clear explanations in any tax notices or inquiries. A)IV only B)I, II, III, and IV C)II and IV D)III only

I, II, III, and IV All of these are requirements the IRS must follow to assure taxpayer rights.

After arriving at adjusted gross income (AGI), which of the following is(are) deductible to arrive at taxable income? Additional standard deduction Standard deduction Itemized deductions if greater than the combined standard deductions

I, II, and III

Which of the following tax factors may limit the availability of tax benefits from a limited partnership? "Passive loss" rules At-risk rules Alternative minimum tax A)I only B)I and II C)I, II, and III D)III only

I, II, and III

Under the installment sale method, when must a seller-lender recognize all of the gain remaining in an installment note? When an installment sale is canceled When the lender makes a gift of the installment note to the debtor When the lender sells the installment note to a third party A)II and III B)I only C)I, II, and III D)III only

I, II, and III All of these statements are correct.

Which of the following statements correctly identify the requirements necessary to deduct $25,000 of losses from an active participation real estate program? The property cannot be used as a vacation home for more than 14 days or 10% of the number of days during the year that the home was rented at a fair rental price., whichever is greater The taxpayers can file as MFS. The taxpayer must make the major management decisions related to the property. The taxpayer's interest in the property may not be held as a limited partnership interest. A)II and IV B)II, III, and IV C)I, III, and IV D)I and III

I, III, and IV If the taxpayers file with the MFS status, the deduction is limited to $12,500. All other statements are true.

Jimmy will have an adjusted gross income of $275,000 for the current tax year. He would like to reduce his tax liability without exposing himself to personal liability. Which of the following investments would be appropriate for Jimmy? An investment in active participation rental real estate An investment in a newly formed low-income housing limited partnership An investment in an oil and gas working interest An investment in a newly formed historic rehabilitation limited partnership A)II, III, and IV B)II and III C)III and IV D)II only PREV

II only Jimmy should invest in a newly formed low-income housing limited partnership. Active participation in real estate, historic rehabilitation, and investing in oil and gas are subject to personal liability.

Which of the following activities is treated as a rental activity under the passive activity rules? Property rental where average customer use is 6 days Property rental where average customer use is more than 30 days and no significant services are provided Property rental where extraordinary services are provided on behalf of the owners Property rental where the property is customarily made available during defined business hours for the nonexclusive use of customers A)II, III, and IV B)II and III C)I only D)II only

II only Only Statement II is treated as a rental activity according to IRS regulations. If a rental is 30 days or less and significant personal services are provided, the activity is a service rather than a rental activity.

Which of the following statements regarding filing status is CORRECT? The single filing status is used by unmarried and divorced taxpayers, but not those who are legally separated and who do not qualify for any other filing status. Taxpayers who file married filing jointly have joint and several liability for the payment of the income taxes due.

II only Statement I is incorrect. Taxpayers who are unmarried, legally separated, or divorced individuals who do not qualify for any other filing status generally use the single filing status. Statement II is correct.

Which of the following is NOT self-employment income? Net Schedule C income The distributive share of income paid to a limited partner Part-time earnings of an individual (e.g., a professional) Board of directors fees A)I, II, III, and IV B)I and II C)III and IV D)II only

II only The distributive share of income paid to a limited partner is not considered self-employment income, but would be income received under the passive activity rules.

Which of the following statements regarding the kiddie tax is CORRECT? The standard deduction for a child with both earned and unearned income is always earned income plus $350. The child's tax rate is 10% for all income received. The excess of unearned income above $2,200 is tax to the child at the parents' top marginal tax rate. The kiddie tax provision limits the effectiveness of income shifting, wherein families are prevented from transferring large amounts of unearned income to children and making the shift effective for income tax purposes. A)I, II, and III B)III only C)III and IV D)I only

III and IV Statements I and II are incorrect. The standard deduction for a child with both earned and unearned income is the greater of $1,100 (2021) or earned income plus $350, but it is limited to the standard deduction for a single taxpayer, $12,550 in 2021. The child's tax rate is determined by the amount of taxable income the child has. Statements III and IV are correct.

Five years ago, Tom bought 10,000 shares at $10 per share in an intermediate-term bond fund. Today, the shares are worth $200,000 and are paying a nonqualified dividend of $8,000 per year. Tom feels that the stock will continue to appreciate at a rate of 5% per year, including the dividend. Tom wants to establish a college education fund for his two daughters, ages 18 and 9. Neither child has any earned income. Which of the following statements is true? If Tom gives 2,500 shares to his 18-year-old daughter, all income from the 2,500 shares will be taxed in her income tax bracket. If Tom gives 2,500 shares to his 9-year-old daughter, all dividends from the 2,500 shares will be taxed at her marginal rate. Two years from now, if Tom's older daughter sells her 2,500 shares at $30 per share, Tom will need to report the gain as a long-term capital gain on his personal income tax return. All interest income received by his 9-year-old daughter that exceeds $2,200 in 2021 will be taxed at the parents' marginal tax rate.

IV only Statement IV is the only correct statement. The kiddie tax applies to children under 19 years of age. It also applies to children under age 24 if they are full-time students. The kiddie tax does not apply if the child's earned income exceeds one-half of the child's support. Thus, I and II are incorrect. There is no requirement that the proceeds of a future sale be reported on the donor's return.

Marcus purchased a diamond ring for $15,000 10 years ago. It was stolen in March of this year. The ring was purchased to celebrate achieving a significant promotion at work. The FMV at the time of the theft was $20,000. The ring was insured, and, after the deductible, Marcus received $19,000 from the insurance company in October of this year. Under Section 1033, how does Marcus account for the $4,000 realized gain on his income taxes? A)Marcus must purchase the qualified replacement property before the end of this year for his realized gain to not be recognized. B)If Marcus purchases another ring for $19,000 or more, his realized gain is not recognized. C)Marcus has three years from the end of this year to replace the property without having to recognize any realized gain. D)Marcus must use the taxpayer-use test in determining whether the replacement property qualifies under Section Under 1033.

If Marcus purchases another ring for $19,000 or more, his realized gain is not recognized. If the amount reinvested in the replacement property equals or exceeds the amount realized, the realized gain is not recognized. Normally, the taxpayer has two years from the end of the taxable year in which any gain is realized from an involuntary conversion (e.g., theft) to replace the property. However, if a condemnation of real property by a governmental authority is the reason for the conversion, this period is extended to three years from the end of the taxable year in which any gain is realized. Because this was a personal use item, Marcus will apply the functional-use test, not the taxpayer-use test.

Which of the following is NOT one of the methods by which an S corporation can be terminated? A)A majority vote of the shareholders B)When it fails to meet the requirements of a small business corporation C)If it earns more than $10 million in gross receipts in any one year D)If gross income for 3 years in a row is of a certain type that exceeds a certain share of total income

If it earns more than $10 million in gross receipts in any one year The only time that earnings can revoke the S status is when more than 25% of gross receipts for three successive years come from certain types of passive income and the corporation has accumulated earnings and profits from its operations prior to the S election. LO 7.1.1

Which one of the following statements is CORRECT with respect to a second, or vacation, home? A)If the home is rented for fewer than 15 days, the rental income is taxable, and the rental expenses are deductible. B)If the home is rented for fewer than 15 days, the rental income is not taxable. C)If the residence test is met, the home mortgage interest is not deductible. D)If the residence test is not met, then any rental income is taxable, but the expenses attributable to the rental are nondeductible.

If the home is rented for fewer than 15 days, the rental income is not taxable. If property is rented fewer than 15 days per year, the taxpayer can exclude rental income from gross income, but expenses are nondeductible, except mortgage interest, taxes, and casualty losses (Schedule A).

Which of the following is the main reason for a premarital agreement? A)It financially protects the parties in a divorce, particularly the one who has more assets. B)It financially protects the parties if the death of one occurs, particularly the one who has fewer assets. C)It financially protects the parties if the death of one occurs, particularly the one who has more assets. D)It financially protects the parties in a divorce, particularly the one who has fewer assets. Explanation

It financially protects the parties in a divorce, particularly the one who has more assets. A premarital agreement financially protects the parties in a divorce, particularly the one who has more assets.

Colin and Lucy are meeting with their financial planner, Jorge, before the tax year ends to discuss changes in their situation. In March of this year, Lucy's father, Curtis, was diagnosed with dementia and required placement in a nursing home. Colin and Lucy paid for about half of his expenses until his death in December of this year. They would like to know what tax relief they may have for supporting Curtis this year. What does Jorge tell the couple? A)The couple is assured of at least being able to list Curtis on their tax return this year. B)The couple can take a partial tax credit for Curtis this year. C)Because they paid for only half of his nursing home expenses for Curtis, the couple is not entitled to the dependent care credit on their tax return. D)Jorge should ask for documentation to establish Curtis' income, the amounts spent on his own support, and how much the couple spent before he can provide any recommendations.

Jorge should ask for documentation to establish Curtis' income, the amounts spent on his own support, and how much the couple spent before he can provide any recommendations. Jorge cannot provide any answers to Colin and Lucy until he has more documentation to support his recommendations.

Don and Paul are married. They adopted an infant daughter in December of last year. They have consulted you, a CFP® professional, for advice on how to proceed when filing their federal income tax return this year. What should you recommend as their filing status this year for their federal return?

Married filing jointly Having a dependent does not change the filing status for a married couple.

In which of the following situations may the installment method of accounting be used? A)Mike sells a classic convertible automobile to a coworker for an $8,000 gain. The sales price will be paid to Mike in a lump sum next year. B)Elena is a used car dealer and reports her gains using the installment method if the buyer does not pay cash. C)Mary sells a vacant lot to her friend for a loss. Mary will receive payments over four years. D)Kalila sells IBM stock to her neighbor. Kalila will receive half of the sale price this year and half next year.

Mike sells a classic convertible automobile to a coworker for an $8,000 gain. The sales price will be paid to Mike in a lump sum next year A casual sale of an asset does qualify for installment sale treatment. An installment sale is a sale in which a payment will be received in a year later than the installment sale. In this situation, there would be no gain recognized in the current year, and all gain would be recognized next year when Mike receives the payment for the auto. The installment method of accounting cannot however be used to report the sale of inventory, such as car dealerships. LO 6.2.4

Which one of the following is NOT a goal of the federal income tax system? A) Monetary policy B) Economic growth C) Inflation control D) Price stability Explanation The Federal Reserve System controls monetary policy. Income tax policy is used to influence all of the other goals. LO 8.1.2

Monetary Policy The Federal Reserve System controls monetary policy. Income tax policy is used to influence all of the other goals.

Ken (a single taxpayer), age 28, holds the following securities: StockPurchase DateFair Market ValueCost BasisABC (300 shares)Oct. 3, 2003$12,200$5,500DEF (500 shares)Feb. 15, 2021$22,600$15,600GHI (100 shares)June 2, 2006$4,350$6,250LMN (700 shares)Dec. 9, 2020$19,360$28,560XYZ small-cap fund (500 shares)Oct. 20, 2011$1,200$3,700VWL (750 shares, §1244 stock)July 17, 2005$4,050$115,600 BondsPurchase DateFair Market ValueCost BasisEE savings bondsMay 1, 2012$8,500$6,000 If Ken sells ABC, DEF, GHI, and LMN on July 1, 2021, at their fair market values, what is the net capital gain or loss (ignoring commissions) from the security sales? A)Net long-term gain of $2,600 B)Net short-term gain of $2,600 C)Net long-term gain of $4,800 and net short-term loss of $2,200 D)Net short-term loss of $2,500 and net long-term gain of $5,100

Net long-term gain of $2,600 The long-term items (ABC and GHI) are netted, leaving a long-term capital gain of $4,800. The short-term items (DEF and LMN) are netted, leaving a short-term capital loss of $2,200. These are netted, leaving a net long-term capital gain of $2,600.

Ken (a single taxpayer), age 28, holds the following securities: StockPurchase DateFair Market ValueCost BasisABC (300 shares)Oct. 3, 2003$12,200$5,500DEF (500 shares)Feb. 15, 2021$22,600$15,600GHI (100 shares)June 2, 2006$4,350$6,250LMN (700 shares)Dec. 9, 2020$19,360$28,560XYZ small-cap fund (500 shares)Oct. 20, 2011$1,200$3,700VWL (750 shares, §1244 stock)July 17, 2005$4,050$115,600 BondsPurchase DateFair Market ValueCost BasisEE savings bondsMay 1, 2012$8,500$6,000 If Ken sells ABC, DEF, GHI, and LMN on July 1, 2021, at their fair market values, what is the net capital gain or loss (ignoring commissions) from the security sales? A)Net short-term loss of $2,500 and net long-term gain of $5,100 B)Net long-term gain of $2,600 C)Net short-term gain of $2,600 D)Net long-term gain of $4,800 and net short-term loss of $2,200

Net long-term gain of $2,600 The long-term items (ABC and GHI) are netted, leaving a long-term capital gain of $4,800. The short-term items (DEF and LMN) are netted, leaving a short-term capital loss of $2,200. These are netted, leaving a net long-term capital gain of $2,600.

Francine and Marshall have three children: Bill, Curt, and Rachel. For 2021 Bill, age 11, has $1,250 of interest income. Curt, age 13, has $2,950 of salary from a part-time job. Rachel, age 19, a part-time student for four months of the year, has $5,100 of dividends and capital gains. Whose income is subject to income tax at the parents' tax brackets? A)Bill B)Bill and Rachel C)Curt D)None of the children

None of the children The parents' marginal tax rates apply to unearned income above $2,200 for 2021. Rachel has unearned income above $2,200. However, because Rachel is 19 and is not a full-time student, she is not subject to the kiddie tax rules. The kiddie tax stops applying in the year that the child turns 19, if the child is not a full-time student. The parents' tax rate does not apply to earned income, so Curt is not subject to the kiddie tax. The kiddie tax applies to children who are under 19 years of age, or who are under 24 if a full-time student. The kiddie tax does not apply to a child who is married and files a joint return for the tax year, or if the child has earned income that exceeds half of his support. Also, the kiddie tax applies only where the child has at least one living parent. (A full-time student is an individual who is a full-time student for at least five calendar months during the tax year.)

Which of the following is a public pronouncement that contains official guidance about regulations or interpretations of the Internal Revenue Code (IRC)? A) Private Letter Ruling B) Revenue Procedure C) Revenue Ruling D) Notice

Notice A notice is a public pronouncement that contains official guidance about regulations or interpretations of the Code. The guidance is often substantial but again, ultimately it only points to higher regulations. Notices in and of themselves do not carry the weight of law. LO 8.1.1

How can passive activity losses be deductible from other taxable income? A)Passive losses can only be offset by passive gains. B)Passive activity losses are deductible against portfolio gains. C)Passive losses can only be carried forward against future passive gains. D)Passive losses can offset passive gains, and a phased-out $25,000 deduction ($12,500 for MFS) for rental real estate applies.

Passive losses can offset passive gains, and a phased-out $25,000 deduction ($12,500 for MFS) for rental real estate applies. Generally, passive activity losses can only be used to offset passive activity income. An exception for the deduction of passive losses is for rental real estate. A phased-out deduction of $25,000 ($12,500 for MFS) of rental real estate losses is allowed against a taxpayer's other nonpassive income. If AGI is greater than $100,000 ($50,000 for MFS), however, there is a reduction of 50 cents for each dollar over $100,000 ($50,000 for MFS), which then terminates at $150,000 ($75,000 for MFS) of AGI.

George, whose wife died last November, filed a joint tax return for last year. He did not remarry after his wife's death and has continued to maintain his home for his two dependent children. In the preparation of his tax return for this year, what is George's filing status?

Qualifying widower George filed a joint return in the year of his wife's death. He can file as a qualifying widower (also known as surviving spouse) for the two years following his wife's death if he continues to maintain a home for his dependent children

Which one of the following is a taxation function of the economic objective? A)Renovation of a historic home B)Reduction of taxes to stimulate the economy C)Charitable deduction D)Excluding life insurance proceeds from taxation

Reduction of taxes to stimulate the economy The reduction of taxes in order to stimulate the economy is due to the economic objective. Social objectives of the federal taxation system include the charitable deduction, excluding life insurance proceeds from taxation, and renovation of a historic home

To be considered a responsible person by the IRS, which one of the following is among the important factors? A)No authority to sign checks B)Shareholder of the company C)Relative of a board member D)Not in charge of hiring and firing employees

Shareholder of the company The determination of whether someone is a responsible person is a test of facts and circumstances. Some of the questions the IRS usually asks in determining responsibility are as follows: Was the individual an officer or director? Was the individual a shareholder? Was the individual a member of a board of directors? Did the individual have the authority to sign checks? Was the individual responsible for hiring and firing employees? Did the individual have actual authority or merely an impressive title?

Jack was divorced on March 30 of the current year and has not remarried as of the last day of the tax year. He lives alone in his condo. His ex-wife, Mary, has custody of their son Jack Jr. What is Jack's filing status for the current tax year

Single A taxpayer who is unmarried, legally separated, or divorced and does not qualify for any other filing status must use the single filing status. Jack does not have custody of his son and does not qualify for head of household status

Which one of the following objectives of the federal taxation system would include the addition of bonus depreciation to the tax code? A)Economic objective B)Social objective C)Writing tax code D)Revenue raising

Social objective As a social objective, JGTRRA and TCJA significantly increased the Section 179 expense limit and the amount of depreciation deductions (bonus depreciation) that may be claimed in the first year in an attempt to stimulate purchases of business assets. The first, and perhaps most important, goal of the economic objective is price stability. Revenue raising through corporate, individual, and payroll taxes is an important objective of the federal taxation system. LO 8.1.2

Courtney and Della are considering obtaining a home equity line of credit of $50,000. They will use some of the proceeds to make needed improvements to their personal residence. Della is concerned about the deductibility of the interest. Which of the following statements is(are) CORRECT?

Statement I is correct. Home equity loan interest is not deductible on a taxpayer's income tax return to the extent it is used for other than home acquisition or improvements for the home that secure the mortgage. The interest on the funds used for home improvements is deductible

If a taxpayer deducted medical expenses on a prior year's tax return and receives a reimbursement of the expenses from her insurance company in the following year, which one of the following rules or doctrines might cause the reimbursement to be included in income? A)Tax benefit rule B)Assignment of income doctrine C)Ownership attribution rules D)Constructive receipt doctrine

Tax benefit rule The tax benefit rule applies to situations in which a particular tax benefit exists. For instance, if an insurance reimbursement is received for medical expenses, it is generally nontaxable. However, if the reimbursement was received in the year after a medical expense deduction was taken and a tax benefit was received for that deduction, then the reimbursement is converted into taxable income in that subsequent year. LO

Anne is the owner of 1,000 shares of the GRQ Mutual Fund Group. Recently, because of favorable market conditions, she sold 1,000 shares of the fund for a market price well in excess of her adjusted cost basis. She is now interested in computing her gain on these shares. Anne uses the specific identification method to determine basis. Which of the following is an income tax implication for Anne in the sale of her mutual fund shares? A)The gain is computed by subtracting the documented basis of identified shares from their sale price. B)The gain is computed using the excess of the fair market value over the cost of those shares first purchased. C)The gain is computed according to the average cost of all shares as compared to the total sale proceeds. D)The gain is computed using the excess of the fair market value over the cost of those shares last purchased.

The gain is computed by subtracting the documented basis of identified shares from their sale price When using the specific identification method of computing basis in the sale of mutual fund shares, it is necessary to identify the shares being sold based on their purchase date. The documented basis of those shares is then used to compute the gain (or loss) realized.

Which one of the following reflects the CORRECT sequence of steps in the tax calculation process?

Total (gross) income minus adjustments to income equals adjusted gross income (AGI). AGI minus standard or itemized deduction(s) equals federal taxable income.

Pam is considering renting out her vacation home in Vail for two weeks during ski season. She lives in the home for approximately two months of the year. She has asked you to explain the income tax consequences. Which one of the following statements is CORRECT? A)The rental income is includible in full in gross income, and the mortgage interest and property tax are 2/52 deductible for AGI. B)The rental income is includible in income, but mortgage interest and property taxes allocable to the rental are not deductible for AGI. C)The rental income is not includible in income, and the mortgage interest and property taxes are fully deductible as itemized deductions. D)The rental income may or may not be includible in income depending on the amount.

The rental income is not includible in income, and the mortgage interest and property taxes are fully deductible as itemized deductions.

Which one of the following statements correctly describes the income tax consequences upon the disposition of a passive activity? A)The suspended losses are nondeductible. B)The suspended losses are deductible in full if more than 50% of the interest in the activity is sold. C)The suspended losses are deductible against any income upon the sale of the entire interest. D)If a taxpayer sells 20% of the interest in an activity, then 20% of the suspended losses are deductible against any other income.

The suspended losses are deductible against any income upon the sale of the entire interest.

Jim purchased a limited partnership interest many years ago for $50,000. He has $26,000 of suspended losses from the partnership. His basis in the partnership was $15,000 and he recently sold his entire partnership interest for $20,000. Which one of the following statements correctly describes the proper tax treatment of the sale? A)There is a $30,000 loss resulting from the sale of the partnership interest, and the $11,000 of suspended losses are deductible in full. B)There is a $5,000 gain resulting from the sale of the partnership interest, and $11,000 of suspended losses is deductible in full. C)There is a $30,000 loss resulting from the sale of the partnership interest, and the suspended losses are deductible in full. D)There is a $5,000 gain resulting from the sale of the partnership interest, and the suspended losses are deductible in full.

There is a $5,000 gain resulting from the sale of the partnership interest, and the suspended losses are deductible in full. The gain or loss is the difference between the sale price and the adjusted basis of the partnership interest—$5,000 in this instance. Upon the taxable disposition (typically the sale) of the passive activity, all suspended losses are deductible in full against any other income. LO 7.1.2

Which of the following sources of information would be the most authoritative when conducting research on an income tax issue? A)Technical Advice Memorandum B)Treasury Regulation C)CPA's opinion letter D)Revenue Procedure

Treasury Regulation

The substitute basis of the qualifying asset received in a like-kind exchange is the asset's A)fair market value increased by the gain realized but not recognized. B)basis reduced by the gain realized but not recognized. C)fair market value reduced by the gain realized but not recognized. D)basis increased by the gain realized but not recognized.

fair market value reduced by the gain realized but not recognized. The substitute basis of a qualifying asset received in a like-kind exchange is the asset's fair market value reduced by the gain realized but not recognized. The deferred gain reduces the basis of the acquired asset, such that when that asset is sold, there is a larger gain recognized.

A revenue ruling is A)a judicial interpretation of specific circumstances related to a taxpayer. B)a court's general administrative interpretation of the Internal Revenue Code and regulations. C)an administrative interpretation of statutory tax law that is generally related to specific circumstances of fact. D)a judicial interpretation of an Internal Revenue Code provision or a Treasury regulation.

an administrative interpretation of statutory tax law that is generally related to specific circumstances of fact. A revenue ruling is an administrative interpretation of statutory tax law that is generally related to specific circumstances of fact.

Which of the following statements is CORRECT? Targeted program audits make up approximately 25% of each year's total of returns audited. The general statute of limitations for audits is three years from the filing date of the return, but six years if 25% of gross income is unreported. A)II only B)I only C)Both I and II D)Neither I nor II PREV

both 1 and 2 Targeted program audits make up approximately 25% of each year's total of returns audited. The general statute of limitations for audits is three years from the filing date of the return (or due date, if later), but six years if 25% of gross income is unreported. LO 8.2.2

As part of their divorce decree, Judy, age 44, was forced to split her IRA with Alex, age 36. Alex received a check in April for $100,000 from the IRA custodian. Alex put the proceeds into a one-year CD account at the local bank. As a result, Alex will A) have to pay ordinary income tax on the $100,000. B) have to pay ordinary income tax on the $100,000 plus a 10% early withdrawal penalty. C) not have to pay ordinary income tax on the $100,000. D) not have to pay income tax, but Judy will owe gift tax.

have to pay ordinary income tax on the $100,000. Although the transfer itself is tax free, Alex should roll the assets distributed from Judy's IRA within 60 days into his own IRA or retirement plan. Because he failed to do so, he owes ordinary income tax on the entire distribution. When incident to a divorce, domestic relations orders (DROs) are however not subject to gift tax or early withdrawal penalties.

What, if any, is the primary difference in tax treatment between a general partnership and a limited partnership? A)Limited partnerships are taxed as corporations, while general partnerships are taxed as partnerships. B)None of these. C)The limited partners only receive capital distributions, while the general partners receive only ordinary income distributions. D)The limited partners are treated only as capital investors, whereas the active partners receive both ordinary and capital distributions.

none of these As long as the limited partnership is classified as a partnership (and not a C corporation) for tax purposes, the taxation is no different than it would be if the organization were a general partnership. That is, the partnership issues a K-1 to all of the partners for their distributive share of items of income and loss. Limited partners generally must treat net income or loss from the partnership as passive.

Which business entity is never subject to federal income taxation? A)S corporation B)C corporation C)Individual D)Partnership PREV

partnership Partnerships are never subject to federal income tax. They are pass-through entities. C corporations and individuals are separately taxed, and certain S corporation transactions are taxed at the entity level.

The effective tax rate is obtained by dividing the amount of tax paid by

taxable income The effective tax rate is found by dividing total tax by taxable income.

All of the following statements regarding accounting methods are correct except A)under the hybrid method of accounting, a taxpayer may account for some items using the accrual method and some items using the cash method. B)once an accounting method has been adopted, it cannot be changed without approval from the IRS. C)individuals and sole proprietorships may use the cash method of accounting. D)the cash method of accounting is generally required when the sale of merchandise is an income-producing factor.

the cash method of accounting is generally required when the sale of merchandise is an income-producing factor. The accrual method of accounting is required when the sale of merchandise is an income-producing factor.

If a court disallows a loss on the sale of an asset because the sale was not bona fide and was made for the sole purpose of realizing a loss, the court is applying A)the clear reflection of income doctrine. B)the assignment of income doctrine. C)the sham transaction doctrine. D)the step transaction doctrine.

the sham transaction doctrine. A sale that is not bona fide and made for the sole purpose of realizing a loss from the transaction would be considered a sham.


Conjuntos de estudio relacionados

Microsoft AZ-900 Cloud Concepts - Principles of cloud computing

View Set

Health Benefits of Physical Activity

View Set

A&P Lab- Week Two: Exercises 4 & 5 (questions)

View Set

Thomas Jefferson (3rd President)

View Set

infection control/prevention prepu lvls 5-6

View Set

Chapter 8 Cardiorespiratory Fitness Training

View Set

Chapter 15: Files, Streams and Object Serialization

View Set

Chapter 2: Social and Cultural Diversity

View Set

Chapter 13 The Heart and Heart Disease

View Set

Module 6 Computer Concepts Exam, Computer Concepts Module 8, Computer Concept Module 9, Module 10 Computer Concepts Exam

View Set